18 Psychiatry
18 Psychiatry
1
likely present in the patient?
(or)
What abnormality of affect is most likely present in a patient who exhibits uncontrollable laughing and
sudden tears without any apparent reason?
A. Flat/Blunt affect
B. Labile affect
C. Elevation of mood
D. Depressed mood
----------------------------------------
6. What is the term that refers to an inability to express or understand emotions?
A. Avolition
B. Altruism
C. Alexithymia
D. Anhedonia
----------------------------------------
7. In a clinical setting, a patient is observed to have a tendency to provide excessive and unnecessary
details while communicating. This phenomenon is known as:
A. Flight of ideas
B. Prolixity
C. Circumstantiality
D. Perseveration
----------------------------------------
8. What is the term for the communication style characterized by rapid speech, excessive details,
occasional sidetracking, but eventually returning to the main topic and answering the question?
(or)
During a psychiatric evaluation, a patient displays a communication style characterized by speaking
rapidly, providing excessive details, and occasionally getting sidetracked. However, they eventually
return to the main topic and answer the question. This phenomenon is known as:
A. Prolixity
B. Flight of ideas
C. Circumstantiality
D. Tangentiality
----------------------------------------
9. What is the term used for a person who maintains a steadfast belief that they are under constant
surveillance by a government agency, even after reassurances from loved ones?
(or)
A 35-year-old woman firmly believes that she is being constantly monitored by a secret government
organization. Despite reassurances from her family and friends, she remains convinced of this belief.
Page 2
2
This is an example of:
A. Hallucination
B. Obsession
C. Delusion
D. Paranoia
----------------------------------------
10. What is the term for the scenario in which a woman firmly believes that her colleagues are plotting
against her and trying to harm her, even when reassured by colleagues and presented with evidence to
the contrary?
(or)
A 35-year-old woman is convinced that her colleagues at work are constantly plotting against her and
trying to harm her. Despite reassurances from her colleagues and evidence to the contrary, she firmly
holds onto this belief. This scenario is an example of:
A. Delusion of Persecution
B. Delusion of Grandiosity
C. Delusion of Reference
D. Delusion of Nihilism
----------------------------------------
11. What is the psychological condition represented by a 60-year-old male patient with depression who
believes that his body has decayed, lacks vital organs, and refuses to eat or take care of himself
because he thinks he is already dead?
(or)
A 60-year-old male patient with depression complains that his body has decayed and he no longer has
vital organs. He refuses to eat or engage in self-care activities because he believes that he is already
dead. This scenario is most consistent with:
A. Cotard syndrome
B. Delusion of Reference
C. Delusion of ill Health
D. Delusion of Persecution
----------------------------------------
12. Which type of delusion is most likely represented by a patient who believes insects are crawling
under his skin, causing him physical discomfort and harm?
(or)
A 35-year-old male patient visits a psychiatrist with a complaint of insects crawling under his skin. He
firmly believes that these insects are causing him physical discomfort and harm. Which type of delusion
does this scenario most likely represent?
A. Delusion of persecution
B. Delusion of infestation/parasitosis
C. Delusion of dysmorphophobia
Page 3
3
D. Delusion of foul body odor/halitosis
----------------------------------------
13. A 50-year-old man visits a psychiatrist with complaints of extreme jealousy and suspicion regarding
his wife's loyalty. Despite no evidence, the patient firmly believes that his wife is having multiple affairs
and becomes highly distressed. He constantly monitors her activities, interrogates her, and accuses her
of being unfaithful. The patient has a history of alcohol abuse. Which of the following best describes the
patient's condition?
(or)
What best describes a man with extreme jealousy, suspicion of his wife's loyalty despite no evidence,
constant monitoring, interrogation, and accusations, along with a history of alcohol abuse?
A. Delusion of persecution
B. Othello's Syndrome
C. Delusion of grandiosity
D. Delusion of reference
----------------------------------------
14. Which of the following statements about bizarre and non-bizarre delusions is incorrect?
(or)
Which of the following statements about bizarre and non-bizarre delusions is incorrect?
A. Bizarre delusions are scientifically impossible or culturally implausible.
B. Non-bizarre delusions are false beliefs that are actually possible.
C. Bizarre delusions often involve beliefs that are socially accepted and commonly shared.
D. Non-bizarre delusions may be unlikely but still within the realm of possibility.
----------------------------------------
15. What is the best description of the symptom disorganized speech, showing a lack of logical
connection between thoughts, and sentences that appear unrelated and incoherent?
(or)
A 28-year-old female presents with disorganized speech characterized by a lack of logical connection
between thoughts. Her sentences often seem unrelated and do not follow a coherent train of thought.
Which of the following best describes this symptom?
A. Loosening of associations
B. Word salad
C. Neologism
D. Tangentiality
----------------------------------------
16. What term best describes the phenomenon exhibited by a 35- year-old male who while speaking,
switches topics abruptly and displays continuous speech flow with no pauses or breaks?
(or)
What term best describes the phenomenon exhibited by a 35- year-old male who while speaking,
switches topics abruptly and displays continuous speech flow with no pauses or breaks?
Page 4
4
A. Derailment
B. Thought blocking
C. Flight of ideas
D. Neologism
----------------------------------------
17. What is the most likely thought disorder in a patient during a psychiatric evaluation, where he starts
discussing unrelated topics without directly answering the initial question about his hobbies?
(or)
A 30-year-old male patient is being interviewed during a psychiatric evaluation. When asked about his
hobbies, he starts discussing his recent vacation, and the weather conditions at the destination, and
then suddenly mentions his love for gardening. Despite the initial question about hobbies, he never
directly answers it. What is the most likely thought disorder exhibited by this patient?
A. Loosening of association
B. Derailment/Knight's move thinking
C. Tangentiality
D. Neologism
----------------------------------------
18. In a psychiatric evaluation, a patient uses the word "flibberdygibbet" to describe a common
household object. This is an example of:
(or)
In a psychiatric evaluation, a patient uses the word "flibberdygibbet" to describe a common household
object. This is an example of:
A. Loosening of association
B. Neologism
C. Derailment
D. Tangentiality
----------------------------------------
19. Which of the following best represents word salad in a clinical scenario?
(or)
Which of the following best represents Word salad in a clinical scenario?
A. The patient expresses his preference for blue shoes and talks about the weather.
B. The patient describes his daily routine in a clear and organized manner.
C. The patient utters random words and phrases that do not make any sense
D. The patient discusses his hobbies and interests in a logical and structured manner.
----------------------------------------
20. Which of the following scenarios best represents Vorbeireden?
(or)
Page 5
5
Which of the following scenarios best represents Vorbeireden?
A. The patient is asked about their favorite color and they respond with their favorite book.
B. The patient is asked about their occupation and they provide a detailed description of their family's
occupation.
C. The patient is asked about their age and they reply with their school’s name.
D. The patient is asked about their hobbies and they talk about their injury of the hand occurred few
days back
----------------------------------------
21. A patient reports that he experiences a sense of external influence on his thoughts, as he is having
thoughts of going out to watch a movie, which according to him is being put in his mind from outside.
Which of the following terms best describes this phenomenon?
(or)
What term best describes the phenomenon where a patient reports experiencing a sense of external
influence on their thoughts?
A. Thought Insertion
B. Thought Withdrawal
C. Thought Broadcast
D. Thought Obsession
----------------------------------------
22. A patient wakes up in the middle of the night and claims to see a large spider crawling on the
ceiling. Upon closer examination, it is discovered that it was actually a wall hanging. This is an example
of:
(or)
What is the term for the phenomenon where a patient claims to see a large spider crawling on the
ceiling, but upon closer examination, it was actually a wall hanging.?
A. Delusion
B. Hallucination
C. Illusion
D. Disorientation
----------------------------------------
23. What best describes the experiences reported by a patient with schizophrenia who hears voices
commenting on their actions and discussing their life, insists that the voices are real, and cannot be
controlled?
(or)
A 25-year-old patient with a history of schizophrenia is sitting in the clinic. During the assessment, the
patient reports hearing voices that are not present. The voices are commenting on the patient's actions
and discussing the patient's life. The patient insists that these voices are real and cannot be controlled.
Which of the following best describes the experiences reported by the patient?
A. Hallucination
B. Illusion
Page 6
6
C. Pseudo-hallucination
D. Delusion
----------------------------------------
24. Which of the following statements is true regarding visual hallucinations?
(or)
Which of the following statements is true regarding visual hallucinations?
A. Visual hallucinations are perceptions of objects that are not real but seem real to the patient.
B. Visual hallucinations only occur in schizophrenia and not in organic mental disorders.
C. Visual hallucinations are experienced by the patient as fleeting flashes of light only.
D. Visual hallucinations are primarily observed in auditory processing disorders.
----------------------------------------
25. What is the term used to describe the phenomenon where a patient experiences a visual perception
triggered by a different sensory stimulus?
(or)
A patient reports experiencing a visual perception that is triggered by a different sensory stimulus. This
phenomenon is known as:
A. Illusion
B. Hallucination
C. Pseudo-hallucination
D. Reflex hallucination
----------------------------------------
26. Which of the following best describes the phenomenon of autoscopy?
(or)
Which of the following best describes the phenomenon of autoscopy?
A. Seeing someone else's reflection in the mirror.
B. Feeling disconnected from one's body.
C. Experiencing the presence of another person in the room.
D. Seeing one's own self outside the body.
----------------------------------------
27. Which of the following statements is true regarding hypnagogic and hypnopompic hallucinations?
(or)
Which of the following statements is true regarding hypnagogic and hypnopompic hallucinations?
A. Hypnagogic hallucinations are typically visual, while hypnopompic hallucinations are primarily
auditory.
B. Hypnagogic hallucinations occur during the transition from wakefulness to sleep, while hypnopompic
hallucinations occur during the transition from sleep to wakefulness.
Page 7
7
C. Hypnagogic hallucinations occur during rapid eye movement (REM) sleep, while hypnopompic
hallucinations occur during non-REM sleep.
D. Hypnagogic hallucinations are a common feature of sleep paralysis, while hypnopompic
hallucinations are more common in nightmares.
----------------------------------------
28. A 45-year-old woman presents to the psychiatric clinic with complaints of hearing voices for the past
six months. She reports that she often hears two distinct voices conversing with each other, making
derogatory comments about her. These voices seem to come from nowhere and are not related to any
external stimuli. The patient feels distressed and anxious due to these experiences, as she believes the
voices are real and directed specifically towards her. Upon further assessment, there are no observable
abnormalities in her hearing and no evidence of any actual conversations occurring around her.
(or)
What term is used to describe the phenomenon experienced by a woman, where she hears two distinct
voices making derogatory comments about her, despite no observable abnormalities in her hearing and
no evidence of actual conversations occurring around her?
A. Schizophrenia
B. Bipolar disorder
C. Major depressive disorder
D. Delusion
----------------------------------------
29. What is the primary purpose of the Digit Repetition Test in neuropsychological assessment? A) B)
C) D)
(or)
What is the primary purpose of the Digit Repetition Test in neuropsychological assessment?
A. Assessing visual memory
B. Evaluating attention and working memory
C. Measuring verbal fluency
D. Gauging fine motor skills
----------------------------------------
30. How many consecutive subtractions, starting from 100 and subtracting 7 each time, typically imply
intact concentration?
(or)
A clinician is evaluating a patient's concentration abilities. The patient is given a mental task to assess
their capacity to perform continuous subtraction. Starting with the number 100, the patient is asked to
subtract 7 from each answer repeatedly. How many successful subtractions typically indicate intact
concentration?
A. 3 Subtractions
B. 5 Subtractions
C. 7 Subtractions
D. 9 Subtractions
Page 8
8
----------------------------------------
31. A 55-year-old patient presents with difficulty recalling recent events, such as conversations with
family members, appointments, and where they placed everyday items like keys. However, he can
vividly recall childhood memories, including the names of friends and teachers from elementary school.
Which type of memory is most likely affected in this patient?
(or)
Which type of memory is likely affected in a 55-year-old patient who has difficulty recalling recent
events but can vividly recall childhood memories, with intact long-term memory?
A. Procedural memory
B. Semantic memory
C. Recent memory
D. Remote memory
----------------------------------------
32. A psychiatrist is asking a patient about the similarity between a car and a plane. The patient replies
both are modes of transport. Which of the following statements is correct?
(or)
A psychiatrist is asking a patient about the similarity between a car and a plane. The patient replies
both are modes of transport. Which of the following statements is correct?
A. Insight is absent
B. Abstract thinking is impaired
C. Test Judgement is impaired
D. Abstract thinking is intact
----------------------------------------
33. A patient is presented with a hypothetical situation where he witnesses a child crying alone in front
of their house. The patient is asked what he will do in this situation. Which type of judgment is being
assessed?
(or)
Which type of judgment is being assessed when a patient is presented with a hypothetical situation and
asked what he will do?
A. Test judgment
B. Social judgment
C. Personal judgment
D. Secondary Judgement
----------------------------------------
34. Which of the following is an incorrect pair, matching the grade of insight with its corresponding
description?
(or)
Which of the following is an incorrect pair, matching the grade of insight with its corresponding
description?
Page 9
9
A. Grade 1: Complete denial of illness.
B. Grade 2: Slight awareness of being ill but denying it simultaneously.
C. Grade 3: Awareness of illness attributed to external/physical/medical/unknown factors.
D. Grade 4: Intellectual insight, refuses to apply knowledge to change behavior.
E. Grade 5: Patient refuses to acknowledge the illness.
----------------------------------------
35. Which of the following statements accurately differentiates neurosis from psychosis?
(or)
Which of the following statements accurately differentiates neurosis from psychosis?
A. Neurosis is characterized by preserved personality, while psychosis affects personality.
B. Neurosis is associated with impaired judgment, while psychosis maintains intact judgment.
C. Neurosis often involves the presence of delusions and hallucinations, whereas psychosis typically
lacks them.
D. Neurosis exhibits absent insight, while psychosis demonstrates present insight.
----------------------------------------
Correct Answers
Question Correct Answer
Question 1 1
Question 2 2
Question 3 4
Question 4 1
Question 5 2
Question 6 3
Question 7 3
Question 8 1
Question 9 3
Question 10 1
Question 11 1
Question 12 2
Question 13 2
Question 14 3
Question 15 1
Question 16 1
Question 17 3
Page 10
10
Question 18 2
Question 19 3
Question 20 2
Question 21 1
Question 22 3
Question 23 1
Question 24 1
Question 25 4
Question 26 4
Question 27 2
Question 28 1
Question 29 2
Question 30 2
Question 31 3
Question 32 4
Question 33 1
Question 34 5
Question 35 1
Page 11
11
• The individual's education history is examined to understand if they have encountered problems
in certain areas of learning, indicating potential learning difficulties or disabilities.
Incorrect Options:
Option A: Birth and early development
• While birth and early development can have implications for future health conditions, it does not
directly assess learning difficulties or disabilities.
Option C - Occupational history
• Occupational history focuses on the individual's work timeline and job changes, and it is not
directly related to assessing learning difficulties or disabilities.
Option D - Marital and sexual history
• Marital and sexual history involves information about marriage, relationships, and menstrual
history, and it is not specifically related to the assessment of learning difficulties or disabilities.
Page 12
12
• Negative symptoms in schizophrenia can include reduced speech output, flattened affect, and
lack of motivation. However, the absence of other characteristic symptoms of schizophrenia, such
as incoherent speech patterns, hallucinations, or delusions, makes this option less likely.
Option D - Generalized anxiety disorder
• Generalized anxiety disorder primarily involves excessive worry and anxiety. The symptoms
described in the scenario, such as low mood, reduced energy, and slow speech, are not consistent
with generalized anxiety disorder
Page 13
13
• Altruism refers to selfless concern for the well-being of others. It involves actions or behaviors that
benefit others, rather than an inability to express or understand emotions.
Option D - Anhedonia
• Anhedonia refers specifically to the inability to experience pleasure or a decreased ability to
experience joy. It is a symptom commonly associated with depression, schizophrenia, but it is not
synonymous with alexithymia.
Page 14
14
may exhibit illogical and rhyming associations. The described scenario involves the patient
returning to the main topic and answering the question, which does not align with a flight of ideas.
Option C - Circumstantiality
• Circumstantiality involves providing excessive and unnecessary details in communication,
occasionally getting sidetracked, but ultimately returning to the main topic and answering the
question. The thinking proceeds slowly here. Although the scenario shares some similarities with
circumstantiality, the term "prolixity" more specifically describes rapid speech and excessive
details.
Option D - Tangentiality
• Tangentiality refers to a communication pattern where the person goes off on tangents and never
returns to the main topic or fully answers the question. This is different from the described scenario,
where the patient eventually returns to the main topic and answers the question.
Page 15
15
• Delusions of persecution involve the irrational belief that one is being mistreated, harassed, or
targeted by others, even when there is no objective evidence to support such a belief.
Incorrect Options:
Option B - Delusion of Grandiosity
• Delusion of Grandiosity refers to the belief of having extraordinary powers, knowledge, or wealth,
which is not described in the given scenario.
Option C - Delusion of Reference
• Delusion of Reference involves the belief that others are talking about or spying on the individual,
which is not described in the given scenario.
Option D - Delusion of Nihilism
• Delusion of Nihilism refers to the belief that everything, including oneself and the world, does not
exist, which is not described in the given scenario.
Page 16
16
Incorrect Options:
Option A - Delusion of persecution
• This type of delusion involves the belief that someone is planning to harm or kill the individual. It
does not align with the scenario of insects crawling under the skin.
Option C - Delusion of dysmorphophobia
• This delusion involves the belief that one's body parts are misshapen or distorted. It does not
match the described scenario of insects crawling under the skin.
Option D - Delusion of foul body odor/halitosis
• This delusion entails the belief that one has a strong, unpleasant body odor even if it cannot be
detected by others. It is not applicable to the scenario of insects crawling under the skin.
Page 17
17
schizophrenia, where there is a disruption in the normal flow of thought and a breakdown in the
logical progression of ideas. The person's sentences may seem unrelated or jump from one topic to
another without a clear connection.
Incorrect Options:
Option B - Word salad
• Word salad refers to the incoherent speech where the words and sentences are random and
unintelligible, whereas in this scenario, there is a lack of logical connection between thoughts
without complete incoherence.
Option C - Neologism
• Neologism involves the use of completely new words or phrases that cannot be understood,
which is not the best description for the lack of logical connection seen in this scenario.
Option D - Tangentiality
• Tangentiality refers to a reply that is oblique or tangential, where the person touches the topic
briefly but fails to reach the intended goal. It does not specifically describe the lack of logical
connection between thoughts.
Page 18
18
• Tangentiality refers to a communication style characterized by going off on tangents, deviating
from the main topic, and not returning to it.
• The person may provide oblique or tangential replies to questions, failing to address the central
point. In the given scenario, the patient's response demonstrates tangentiality as they divert the
conversation towards answers from the same topic without directly answering the question about
hobbies.
Incorrect Options:
Option A - Loosening of association
• Loosening of association refers to a breakdown in logical connections between thoughts, where
sentences may not make sense. This is not exhibited in the scenario provided.
Option B - Derailment/Knight's move thinking
• Derailment/Knight's move thinking involves shifting to a different topic with no logical
association.In the derailment there is no pause; there is only shifting to a new topic.
Option D - Neologism
• Neologism refers to using completely new words or phrases whose derivation cannot be
understood. This is not demonstrated in the given scenario.
Page 19
19
Option A - The patient expresses his preference for blue shoes and talks about the weather
• It does not demonstrate incoherence or word salad as the patient's statements are related to his
preference for blue shoes and the weather.
Option B - The patient describes his daily routine in a clear and organized manner
• It represents normal and organized thinking, without any signs of incoherence or word salad.
Option D - The patient discusses his hobbies and interests in a logical and structured manner
• It describes normal and coherent thinking, with the patient discussing hobbies and interests in a
logical manner.
Page 20
20
Solution for Question 22:
Correct Option C: Illusion
• Illusion refers to a false perception or misinterpretation of a real object or stimulus. It occurs when
the brain misinterprets sensory information, leading to a distorted perception of reality. In the given
scenario, the patient's perception of seeing a spider on the ceiling, which is actually a
misinterpretation of the real object (such as a rope or curtain), is an example of an illusion.
Incorrect Options:
Option A - Delusion: Delusion refers to a
false belief held by an individual despite evidence to the contrary. It is a fixed, irrational belief that i
s not influenced by logical reasoning. In the context of the given scenario, the patient's perception
of seeing a spider on the ceiling is not a delusion because it is a
perceptual experience rather than a belief.
Option B - Hallucination: Hallucination is a false perception that occurs without any external stimulu
s. It involves experiencing sensory perceptions that are not based on reality. In the given scenario,
the patient's perception of seeing a spider on the ceiling is not a hallucination because there is a
real object present (the wall hanging), although the perception of the spider is false.
Option D - Disorientation: Disorientation refers to a state of confusion or loss of awareness regardi
ng one's surroundings, time, or identity. It is characterized by a difficulty in recognizing or compreh
ending the environment. Disorientation is not applicable to the given scenario as the patient's perc
eption of the spider on the ceiling does not indicate a state of confusion or loss of awareness.
Page 21
21
Solution for Question 24:
Correct Option A: Visual hallucinations are perceptions of objects that are not real but seem real to the
patient
• Visual hallucinations refers to the perceptions of objects that are not real but are experienced by
the patient as real.
Incorrect Options:
Option B - Visual hallucinations only occur in schizophrenia and not in organic mental disorders
• Visual hallucinations can occur in both schizophrenia and organic mental disorders.
Option C - Visual hallucinations are experienced by the patient as fleeting flashes of light only
• Visual hallucinations can involve various visual experiences, not limited to fleeting flashes of light.
Option D - Visual hallucinations are primarily observed in auditory processing disorders
• Visual hallucinations are specific to the visual modality and are not primarily associated with
auditory processing disorders.
Page 22
22
Incorrect Options:
Option A - Seeing someone else's reflection in the mirror
• The experience of perceiving another person's reflection in the mirror is not related to autoscopy.
Option B - Feeling disconnected from one's body
• Feeling disconnected from one's body describes depersonalization, a dissociative symptom
where individuals feel detached from their own body or experiences. It is different from autoscopy.
Option C - Experiencing the presence of another person in the room
• The perception of another person's presence in the room is not specific to autoscopy.
Page 23
23
Option B - Bipolar disorder
• Bipolar disorder is a mood disorder characterized by episodes of elevated mood (mania or
hypomania) and depressive episodes. While hallucinations can occur during manic or depressive
episodes, they are not a defining feature of bipolar disorder. Other symptoms, such as mood
swings, changes in energy levels, and altered sleep patterns, are more characteristic of bipolar
disorder.
Option C - Major depressive disorder
• Major depressive disorder, or depression, is a mood disorder characterized by persistent feelings
of sadness, loss of interest or pleasure, and a range of other emotional and physical symptoms.
Hallucinations are not typically associated with depression. However, in severe cases, depressive
psychosis may occur, leading to hallucinations or delusions.
Option D - Delusion
• Delusion refers to a fixed false belief that is not based on reality. Unlike hallucinations, which
involve false perceptions, delusions are false beliefs that persist despite evidence to the contrary.
In the given scenario, the patient is experiencing auditory hallucinations, which are not indicative of
a delusion.
Page 24
24
Incorrect Options:
Option A - 3 Subtractions: It is upto 5 subtractions.
Option C - 7 Subtractions: It is upto 5 subtractions.
Option D - 9 Subtractions:It is upto 5 subtractions.
Page 25
25
Solution for Question 33:
Correct Option A: Test judgment
• The correct option is a. Test judgment because in this scenario, the patient is presented with a
hypothetical situation, and their response to it is being assessed.
Incorrect Options:
Option B - Social judgment
• Social judgment is the social behavior of the patient in daily events.
Option C - Personal judgment
• Personal judgment is how the person is able to take self care.
Option D - Secondary Judgement: There is no such type of Judgement
Page 26
26
relatively intact. In contrast, psychosis can affect personality, causing changes or disturbances. The
other options do not accurately reflect the differences between neurosis and psychosis.
Page 27
27
Schizophrenia Spectrum and Other Psychotic
Disorders
1. Which neurotransmitter is increased in schizophrenic patients and is associated with the mesolimbic
pathway, leading to positive symptoms?
A. Serotonin (5HT)
B. GABA
C. Dopamine
D. Glutamate
----------------------------------------
2. A 26-year-old male presents to the psychiatric clinic with a history of persistent symptoms from the
last 2 months. He reports that he is hearing voices commenting on his actions. He also describes that a
group of people is conspiring against him. Upon examination, the patient displays disorganized speech,
jumping from one unrelated topic to another. He appears indifferent to his appearance and lacks
motivation in daily activities. There is no history of substance abuse or other medical conditions. Which
of the following is the most likely diagnosis for this patient?
(or)
What is the most likely diagnosis for a patient presenting with auditory hallucinations, paranoid
delusions, disorganized speech, indifference to appearance, and lack of motivation, without a history of
substance abuse or other medical conditions?
A. Major depressive disorder
B. Bipolar disorder
C. Schizophrenia
D. Generalized anxiety disorder
----------------------------------------
3. During a routine psychiatric evaluation, a 30-year-old male patient is observed to display unusual
motor symptoms. The examiner attempts to move his arms, there is a noticeable resistance initially and
then the examiner is able to move the arms freely and makes the patient come in a position as shown
in the image below and slowly the arms of the patients come down. Which of the following terms best
describes the patient’s behavior?
(or)
What term best describes the motor symptom observed in a 30-year-old male patient during a
psychiatric evaluation, where he maintains his arms extended in front of him for an extended period
without discomfort and exhibits resistance when the examiner attempts to move his arms?
28
A. Catalepsy
B. Posturing
C. Waxy flexibility
D. Negativism
----------------------------------------
4. Which subtype of schizophrenia is associated with predominantly negative symptoms and a poor
prognosis?
A. Paranoid schizophrenia
B. Catatonic schizophrenia
C. Simple schizophrenia
D. Undifferentiated schizophrenia
----------------------------------------
5. Which of the following statements accurately describes the difference between typical and atypical
antipsychotics in the treatment of schizophrenia?
A. Typical antipsychotics primarily act on positive symptoms, while atypical antipsychotics target both
positive and negative symptoms.
B. Atypical antipsychotics are associated with a higher risk of extrapyramidal symptoms compared to
typical antipsychotics.
C. Typical antipsychotics are more likely to cause metabolic side effects such as weight gain,
hypertension, diabetes, and hyperlipidemia than atypical antipsychotics.
D. Atypical antipsychotics primarily antagonize D2 receptors, while typical antipsychotics antagonize
both D2 and 5HT2A receptors.
----------------------------------------
6. A patient presents to the emergency department with sudden muscle contractions and upward rolling
of the eyes. Which of the following conditions is most likely responsible for these symptoms?
(or)
Which of the following is the most likely diagnosis of patient who presents with sudden muscle
contractions and upward rolling of the eyes?
A. Acute dystonia
B. Parkinson's disease
Page 2
29
C. Huntington's disease
D. Restless leg syndrome
----------------------------------------
7. What is the most likely cause of a patient with a history of schizophrenia, presenting coarse tremors
in both hands, limb rigidity, shuffling gait, and mask-like facial expression, approximately two weeks
after initiating first-generation antipsychotic medication?
(or)
A 45-year-old male patient with a history of schizophrenia has been on a first-generation antipsychotic
medication for the past few months. He presents to the clinic complaining of stiffness, tremors, and
difficulty with movement. The symptoms started approximately two weeks after initiating the
antipsychotic medication. On examination, the patient exhibits coarse tremors in both hands, rigidity in
the limbs, and a shuffling gait. His facial expression appears mask-like. There is no evidence of resting
tremors or pill-rolling movements. The patient denies any history of similar symptoms in the past or a
family history of Parkinson's disease. Which of the following is the most likely cause of the patient's
symptoms?
A. Drug-induced Parkinsonism
B. Acute Dystonia
C. Tardive Dyskinesia
D. Neuroleptic Malignant Syndrome
----------------------------------------
8. A 32-year-old female with a history of schizophrenia has been on a second-generation antipsychotic
for the past month. She reports an intense inner restlessness and an inability to sit still, impacting her
daily activities. On examination, she is observed constantly shifting in her chair, crossing and
uncrossing her legs, and frequently changing positions. She describes a strong urge to keep moving
with no relief. What is the most likely cause of her symptoms?
(or)
What is the most likely cause of a patient with a history of schizophrenia, taking a second-generation
antipsychotic medication for the past month, presenting with intense inner restlessness, an inability to
sit still, constantly shifting in chair, and a strong urge to keep moving?
A. Akathisia
B. Acute Dystonia
C. Tardive Dyskinesia
D. Neuroleptic Malignant Syndrome
----------------------------------------
9. A 50-year-old patient with a history of chronic schizophrenia has been on high-dose typical
antipsychotic medication for the past 10 years. Lately, the patient has developed abnormal involuntary
movements involving lip smacking, tongue protrusion, and choreiform hand movements. The
movements are more prominent during periods of stress. What is the most likely diagnosis for this
patient?
(or)
What is the most likely diagnosis for a patient with a history of chronic schizophrenia, on high-dose
typical antipsychotic medication for the past 10 years, and presenting with abnormal involuntary
Page 3
30
movements involving lip smacking, tongue protrusion, and choreiform hand movements that worsen
during periods of stress?
A. Akathisia
B. Acute Dystonia
C. Tardive dyskinesia
D. Parkinsonism
----------------------------------------
10. A 32-year-old male patient with a history of schizophrenia has been receiving high-dose typical
antipsychotic medication for the past month. He presents to the emergency department with severe
muscle rigidity, hyperthermia, altered mental status, and elevated creatinine phosphokinase (CPK)
levels. Physical examination reveals tachycardia and diaphoresis. Based on these findings, what is the
drug of choice for the treatment of this condition?
(or)
What is the drug of choice for treating a patient with a history of schizophrenia, who has been on
high-dose typical antipsychotic medication for the past month and presents with severe muscle rigidity,
hyperthermia, altered mental status, elevated creatinine phosphokinase (CPK) levels, tachycardia, and
diaphoresis?
A. Haloperidol
B. Benztropine
C. Dantrolene
D. Lorazepam
----------------------------------------
11. Which medication is associated with the potential side effect of retinal pigmentation?
A. Thioridazine
B. Fluoxetine
C. Diazepam
D. Ibuprofen
----------------------------------------
12. What are the contraindications for the use of Clozapine in treatment-resistant schizophrenia
patient?
(or)
A 35-year-old male patient with treatment-resistant schizophrenia is being considered for Clozapine
treatment. He has tried two different antipsychotic medications in the past without significant
improvement in symptoms. The psychiatrist is now evaluating the suitability of Clozapine for this
patient. What are the contraindications for the use of Clozapine in this patient?
A. White blood cell (WBC) count < 3500/mm3
B. Previous bone marrow disorder
C. Use with carbamazepinie
D. All of the above
Page 4
31
----------------------------------------
13. A nurse is administering an intramuscular injection to a patient. The nurse wants to use the Z-track
technique. Which of the following statements regarding the Z-track technique is correct?
(or)
Which of the following statements regarding the Z-track technique is correct?
A. It involves injecting the medication at a 45-degree angle into the muscle.
B. It is commonly used for subcutaneous injections.
C. It helps prevent medication leakage into subcutaneous tissues.
D. It is only used for pediatric patients.
----------------------------------------
14. Which psychological therapy is commonly used as an adjunctive treatment for individuals with
schizophrenia?
A. Exposure Sensitization Therapy
B. Cognitive behavioral therapy (CBT)
C. Art therapy
D. Music therapy
----------------------------------------
15. Patient Y, a 35-year-old, presents to the clinic with an abrupt onset of disorganized symptoms,
including incoherent speech and erratic behavior for four weeks. What is the most appropriate
diagnosis for this patient?
(or)
What is the most appropriate diagnosis for a 35-year-old patient with abrupt onset of disorganized
symptoms, including incoherent speech and erratic behavior for four weeks?
A. Schizophrenia
B. Acute psychotic disorder
C. Schizophreniform disorder
D. Delirium
----------------------------------------
16. A 28-year-old patient presents with a complex psychiatric history characterized by recurrent mood
disturbances, recurrent episodes of delusions and hallucinations, and significant periods of manic
symptoms. Despite comprehensive evaluations, the diagnosis remains uncertain. The patient's
symptoms do not fit neatly into either a primary mood disorder or schizophrenia. Which of the following
best describes this patient's condition?
(or)
What is the most likely diagnosis for a patient with a history of recurrent mood disturbances, recurrent
episodes of delusions and hallucinations, and significant periods of manic symptoms?
A. Schizophrenia
B. Bipolar disorder
C. Schizoaffective disorder
Page 5
32
D. Major Depressive Disorder with Psychotic Features
----------------------------------------
17. A 45-year-old woman, Sarah, presents to a psychiatric clinic accompanied by her close friend, Lisa.
Sarah has been experiencing persistent delusions of being monitored and followed by government
agents for the past six months. Lisa, who has been her best friend for many years, has noticed that
Sarah's beliefs have started to influence her own thoughts and she has developed similar delusions of
being watched and targeted by the same government agents. Lisa is concerned about Sarah's
deteriorating mental health and seeks help. What is likely to be their diagnosis?
(or)
What is the primary concern expressed by Lisa whose thoughts are influenced by her best friend
Sarah, who presents with persistent delusions of being monitored and followed by government agents
for the past six months?
A. Sarah is experiencing persecutory delusions and Lisa is experiencing shared psychotic disorder
B. Sarah is experiencing bipolar disorder and Lisa is experiencing major depressive disorder
C. Sarah is experiencing persecutory delusions and Lisa is experiencing major depressive disorder
D. Sarah is experiencing bipolar disorder and Lisa is experiencing induced psychotic disorder
----------------------------------------
18. Mark, a 35-year-old man, has been referred to a psychiatrist by his family due to his increasingly
unusual behavior and beliefs. His family reports that Mark has become convinced that his co-workers,
neighbors, and even random strangers he encounters on the street are all the same person in disguise.
He believes that this individual is stalking him and trying to harm him. Mark's family is concerned about
his safety and well-being and wants to understand his condition better.
(or)
What is the primary concern of Mark's family regarding his behavior and beliefs, which include the belief
that his co-workers, neighbors, and strangers are all the same person in disguise stalking and trying to
harm him?
A. Fregoli syndrome
B. Capgras syndrome
C. Paranoid personality disorder
D. Dissociative identity disorder
----------------------------------------
19. Which of the following factors is associated with a good prognosis in schizophrenia?
A. Late age of onset
B. Presence of prominent negative symptoms
C. Family history of schizophrenia
D. Insidious onset
----------------------------------------
20. A 45-year-old male patient presents with symptoms of persistent sadness, loss of interest,
decreased energy, and difficulty concentrating. He has been experiencing significant changes in
appetite and weight, with unintentional weight gain over the past few weeks. He was prescribed a drug
for his condition post which he started experiencing symptoms like agitation, confusion, tremors,
Page 6
33
restlessness and altered EKG findings like prolonged QT interval. . What is the treatment of choice for
this patient?
(or)
What is the treatment of choice for a patient presenting with symptoms of persistent sadness, loss of
interest, decreased energy, difficulty concentrating, and significant changes in appetite and weight, who
developed agitation, confusion, tremors, restlessness and altered EKG findings like prolonged QT
interval after starting a drug?
A. I.V Sodium Bicarbonate
B. Continue the medication and provide supportive care
C. Switch to a different medication from the same class
D. Adjust the dosage of the current medication
----------------------------------------
21. Which of the following statements is most accurate in the context of clozapine?
A. Antisuicidal because of dopamine antagonism
B. First-line treatment for all schizophrenia
C. Causes minimum weight gain
D. Reserved for refractory schizophrenia
----------------------------------------
Correct Answers
Question Correct Answer
Question 1 3
Question 2 3
Question 3 3
Question 4 3
Question 5 1
Question 6 1
Question 7 1
Question 8 1
Question 9 3
Question 10 3
Question 11 1
Question 12 4
Question 13 3
Question 14 2
Question 15 2
Page 7
34
Question 16 3
Question 17 1
Question 18 1
Question 19 1
Question 20 1
Question 21 4
Page 8
35
these must be (1), (2), or (3): Delusions. Hallucinations. Disorganized speech (e.g., frequent
derailment or incoherence). Grossly disorganized or catatonic behavior. Negative symptoms (i.e.,
diminished emotional expression or avolition).
• Delusions.
• Hallucinations.
• Disorganized speech (e.g., frequent derailment or incoherence).
• Grossly disorganized or catatonic behavior.
• Negative symptoms (i.e., diminished emotional expression or avolition).
• B. For a significant portion of the time since the onset of the disturbance, level of functioning in
one or more major areas, such as work, interpersonal relations, or self-care, is markedly below the
level achieved prior to the onset.
The diagnostic criteria for schizophrenia are outlined in the Diagnostic and Statistical Manual of Me
ntal Disorders, Fifth Edition (DSM-5). To be diagnosed with schizophrenia, a person must meet sp
ecific criteria, including the presence of characteristic symptoms and the duration of those sympto
ms. The criteria are as follows:
A. Two (or more) of the following, each present for a significant portion of time during a
one-month period (or less if successfully treated). At least one of these must be (1), (2), or (3):
• Delusions.
• Hallucinations.
• Disorganized speech (e.g., frequent derailment or incoherence).
• Grossly disorganized or catatonic behavior.
• Negative symptoms (i.e., diminished emotional expression or avolition).
B. For a significant portion of the time since the onset of the disturbance, level of functioning in one
or more major areas, such as work, interpersonal relations, or self-care, is markedly below the lev
el achieved prior to the onset.
Incorrect Options:
Option A - Major depressive disorder
• Major depressive disorder primarily presents with symptoms of persistent low mood, feelings of
worthlessness, changes in appetite and sleep patterns, and reduced energy. While individuals with
schizophrenia may experience depressive symptoms, the presence of positive symptoms
(hallucinations, delusions) and disorganized speech make major depressive disorder less likely in
this case.
Option B - Bipolar disorder
• Bipolar disorder involves episodes of mania or hypomania, alternating with periods of depression.
This patient's symptoms are more indicative of schizophrenia rather than bipolar disorder. The
presence of hallucinations, delusions, and disorganized speech is more characteristic of
schizophrenia.
Option D - Generalized anxiety disorder
• Generalized anxiety disorder is characterized by excessive and uncontrollable worry and anxiety
about various aspects of life. It does not typically involve hallucinations, delusions, and
disorganized speech suggests a different diagnosis, such as schizophrenia
Page 9
36
Solution for Question 3:
Correct Option: C Waxy flexibility
• Waxy flexibility is a motor symptom commonly observed in patients with catatonic features of
schizophrenia. It is characterized by the feeling of resistance encountered when attempting to
move the patient's limbs or body into different positions. The patient's body seems to exhibit a
pliable and moldable quality, similar to the bending of a wax candle. Waxy flexibility is a passive
phenomenon, meaning the resistance is felt when the examiner tries to move the patient's body.
Incorrect options:
Option A - Catalepsy
• Catalepsy refers to maintaining an odd posture for an extended period of time. It is a passive
phenomenon where the patient maintains the posture in response to external instructions, such as
the examiner asking the patient to hold a particular position. Catalepsy does not involve the feeling
of resistance encountered during movement, as seen in the described scenario.
Option B - Posturing
• Posturing is similar to catalepsy in that it involves maintaining an odd posture for an extended
duration. However, posturing is an active phenomenon where the patient spontaneously assumes
the posture without external instructions. In the described scenario, the patient complies with the
examiner's request, suggesting a response to external instruction rather than spontaneous
posturing.
Option D - Negativism
• Negativism refers to the patient's opposition or lack of response to instructions. It is characterized
by a resistance to comply with commands or a tendency to do the opposite of what is requested.
While negativism can be observed in patients with catatonic features, it does not specifically
describe the motor symptoms mentioned in the scenario.
Page 10
37
• Catatonic schizophrenia is incorrect because it is characterized by symptoms of catatonia,
including motor abnormalities and peculiar postures. Catatonic schizophrenia has a better
prognosis compared to simple schizophrenia.
Option D- Undifferentiated schizophrenia
• Undifferentiated schizophrenia is incorrect because it involves a mixture of symptoms and does
not specifically refer to predominantly negative symptoms. The prognosis of undifferentiated
schizophrenia can vary depending on the severity and combination of symptoms.
• Typical Antipsychotics (First Generation): These medications primarily act on positive symptoms
of schizophrenia, which include hallucinations, delusions, and thought disorders. They achieve this
effect by antagonizing dopamine D2 receptors in the brain, reducing the excessive dopamine
activity associated with positive symptoms.
• Atypical Antipsychotics (Second Generation): In addition to addressing positive symptoms,
atypical antipsychotics also target negative symptoms of schizophrenia. Negative symptoms
involve deficits in cognitive and emotional functioning, such as social withdrawal, reduced
motivation, and impaired attention. Atypical antipsychotics not only block dopamine receptors but
also affect serotonin (5HT2A) receptors. This dual action is believed to contribute to their efficacy in
addressing both positive and negative symptoms.
Incorrect options:
Option B: This statement is incorrect. Extrapyramidal symptoms (EPS), which include movement di
sorders like tremors and stiffness, are more commonly associated with typical antipsychotics. Atypi
cal antipsychotics are designed to have a lower risk of causing EPS, making them preferable in so
me cases due to their better tolerability in terms of motor side effects.
Option C: It is incorrect because the metabolic side effects are more with the atypical antipsychotic
s as compared to typical antipsychotics.
Option D: It is incorrect, Atypical antipsychotics not only block dopamine D2 receptors but also affe
ct serotonin 5HT2A receptors. This dual receptor antagonism is a key feature that contributes to th
eir broader efficacy and improved side effect profile compared to typical antipsychotics.
Page 11
38
• Parkinson's disease is a neurodegenerative disorder that primarily affects movement and is
characterized by symptoms such as tremors, rigidity, and bradykinesia. It does not typically cause
sudden muscle contractions and oculogyric crisis.
Option C- Huntington's disease
• Huntington's disease is a genetic disorder that leads to the progressive degeneration of brain
cells. It is characterized by motor abnormalities, cognitive decline, and psychiatric symptoms.
However, it does not typically present with acute episodes of sudden muscle contractions and
oculogyric crisis.
Option D- Restless leg syndrome
• Restless leg syndrome is a neurological disorder characterized by an irresistible urge to move the
legs, usually accompanied by uncomfortable sensations. It does not cause sudden muscle
contractions and oculogyric crisis.
Page 12
39
Solution for Question 8:
Correct Option A: Akathisia
• The clinical presentation of the patient, including the subjective feeling of restlessness, and
objective signs of motor restlessness such as rocking motions, pacing around, and rapid alteration
of sitting and standing, is consistent with akathisia. Akathisia is a common side effect of
antipsychotic medications and is characterized by intense inner restlessness and an inability to sit
still.
Incorrect Options
Option B - Acute Dystonia
• Acute Dystonia presents with sudden muscle contractions, such as oculogyric crisis, torticollis,
and trismus.
Option C -Tardive Dyskinesia
• Tardive Dyskinesia is characterized by abnormal involuntary movements, typically involving the
face, lips, tongue, and extremities, and it usually occurs after prolonged use of antipsychotic
medications.
Option D - Neuroleptic Malignant Syndrome
• Neuroleptic malignant syndrome presents with severe muscle rigidity, hyperthermia, autonomic
instability, and altered mental status.
Page 13
40
involving peri-oral tremors. Although Parkinsonism can be caused by antipsychotic medication, the
specific symptoms mentioned in the scenario, such as lip smacking and choreiform hand
movements, are not indicative of Parkinsonism.
Incorrect Choices:
• Option B: Fluoxetine: Fluoxetine is a selective serotonin reuptake inhibitor (SSRI) commonly used
to treat depression and other mood disorders. Retinal pigmentation is not a known side effect of
fluoxetine. Side Effects: Common side effects of fluoxetine include nausea, insomnia, headache,
and sexual dysfunction. In some cases, it may cause gastrointestinal disturbances or anxiety.
• Option C: Diazepam: Diazepam is a benzodiazepine used as a sedative, anxiolytic, and muscle
relaxant. Retinal pigmentation is not associated with diazepam use. Side Effects: Diazepam can
cause drowsiness, dizziness, and muscle weakness. Long-term use may lead to tolerance and
dependence. Other side effects include confusion, ataxia, and respiratory depression.
Page 14
41
• Option D: Ibuprofen: Ibuprofen is a nonsteroidal anti-inflammatory drug (NSAID) used for pain
relief and reducing inflammation. Retinal pigmentation is not a known side effect of ibuprofen. Side
Effects: Ibuprofen, as a nonsteroidal anti-inflammatory drug (NSAID), may cause gastrointestinal
irritation, peptic ulcers, and an increased risk of bleeding. Prolonged use can lead to renal
impairment and cardiovascular issues.
Option D: Ibuprofen: Ibuprofen is a nonsteroidal anti-inflammatory drug (NSAID) used for pain relief
and reducing inflammation. Retinal pigmentation is not a
known side effect of ibuprofen. Side Effects: Ibuprofen, as a nonsteroidal anti-inflammatory drug (N
SAID), may cause gastrointestinal irritation, peptic ulcers, and an increased risk of bleeding. Prolon
ged use can lead to renal impairment and cardiovascular issues.
Page 15
42
Correct Option B: Cognitive behavioral therapy (CBT)
• Cognitive Behavioral Therapy (CBT) is commonly used as an adjunctive treatment for individuals
with schizophrenia. CBT is an evidence-based psychotherapy approach that helps individuals
understand and manage their symptoms, thoughts, and behaviors. In the context of schizophrenia,
CBT is often used to address issues such as delusions, hallucinations, and disorganized thinking.
Incorrect Choices:
Option A: Exposure Sensitization Therapy: Exposure Sensitization Therapy is not a
standard or commonly used therapy for individuals with schizophrenia. It is not a
recognized treatment approach for this condition.
Option C: Art therapy: Art therapy can be beneficial and may help with self-expression and emotion
al processing, it is not a primary treatment for schizophrenia.
Option D: Music therapy: Music therapy is often used to address emotional and social well-being ra
ther than core symptoms of schizophrenia like delusions and hallucinations.
Page 16
43
• In this scenario, the patient's presentation of experiencing symptoms of schizophrenia, such as
delusions and hallucinations, for 8 months, along with significant periods of manic symptoms,
suggests a diagnosis of schizoaffective disorder (bipolar type). Schizoaffective disorder is
characterized by the presence of both schizophrenia symptoms and mood disturbances, with the
bipolar type specifically indicating the presence of manic symptoms.
Incorrect Options:
Option A- Schizophrenia
• Schizophrenia is an incorrect option because it does not account for the patient's significant
periods of manic symptoms. Schizophrenia alone does not involve mood disturbances or manic
episodes.
Option B- Bipolar disorder
• Bipolar disorder is an incorrect option because it does not fully capture the presence of
schizophrenia symptoms experienced by the patient for 8 months. Bipolar disorder involves mood
episodes (manic and depressive), but not necessarily the presence of schizophrenia symptoms.
Option D- Major Depressive Disorder with Psychotic Features
• Major Depressive Disorder with Psychotic Features involves severe depression with psychotic
symptoms such as delusions and hallucinations. It does not encompass the manic symptoms seen
in the patient's presentation.
Page 17
44
Solution for Question 18:
Correction Option A: Fregoli syndrome
• Mark's belief that multiple different individuals are actually the same person in disguise and
stalking him aligns with Fregoli syndrome. In Fregoli syndrome, a person holds the delusional belief
that different people are actually a single person who changes appearance to deceive and
persecute them.
Incorrect Options:
Option B- Capgras syndrome
• Capgras syndrome is characterized by the delusional belief that a person, usually someone close
to the individual, has been replaced by an imposter. Mark's beliefs are more consistent with Fregoli
syndrome, where he believes multiple individuals are the same person in disguise.
Option C- Paranoid personality disorder
• Paranoid personality disorder involves a pervasive pattern of distrust and suspicion toward others,
but it does not involve the specific delusion of multiple individuals being the same person in
disguise. Mark's symptoms suggest a delusional disorder rather than a personality disorder.
Option D- Dissociative identity disorder
• Dissociative identity disorder, formerly known as multiple personality disorder, involves the
presence of two or more distinct personality states. Mark's symptoms do not align with the
characteristic features of dissociative identity disorder. His belief of individuals being the same
person in disguise is more indicative of a delusional disorder.
Page 18
45
• The patient is suffering from depression and he was probably prescribed a TCA (tricyclic
antidepressant) following which the patient exhibited the symptoms of TCA toxicity.
• For any drug toxicity, the first step should always be to stop the drug and then administer
antidote/treatment.
Incorrect Options:
Option B - Continue the medication and provide supportive care: We cannot continue the drug post
toxicity symptoms. The drug needs to be discontinued.
Option C - Switch to a different medication from the same class: If the toxicity symptoms are specifi
c to the particular medication, switching to another medication within the same class or a different c
lass can be considered. This option aims to find an alternative medication that is better tolerated by
the patient but this is not the treatment of choice.
Option D - Adjust the dosage of the current medication: sometimes toxicity symptoms can occur du
e to an excessively high dosage of the medication. Adjusting the dosage to a lower, safer level ma
y help alleviate the toxicity symptoms while still providing therapeutic benefits but this is not the tre
atment of choice
Incorrect Options:
Option A - Antisuicidal because of dopamine antagonism: The mechanism of action of clozapine's i
s attributed to D4, D2 and 5HT2a receptor antagonism. The exact mechanism of clozapine's anti-s
uicidal effect is much more complex.
Option B - First-line treatment for all schizophrenia: Clozapine is not the first-line treatment for all s
chizophrenia patients. It is typically reserved for patients with treatment-resistant schizophrenia, wh
ich means they have not responded to at least two other antipsychotic medications.
Option C - Causes minimum weight gain: Clozapine is associated with a maximum weight gain am
ong all anti-psychotics and higher risk of metabolic side effects compared to some other antipsych
otic medications.
Page 19
46
Previous Year Questions
1. What is the probable diagnosis for a middle-aged man who visits the psychiatry OPD claiming that
he has been hearing voices from aliens instructing him to harm his family members for the last seven
months?
A. Mania
B. Delusional disorder
C. Schizophrenia
D. Brief psychotic disorder
----------------------------------------
2. Which of the following does not suggest a medical cause of mental illness?
A. Elderly age
B. Loss of consciousness
C. Auditory hallucination
D. Acute onset
----------------------------------------
3. What medication was prescribed to a schizophrenic patient who did not show improvement with
haloperidol and thioridazine, and is now experiencing symptoms of excessive salivation, elevated blood
glucose levels, and hyperlipidemia?
A. Ziprasidone
B. Risperidone
C. Clozapine
D. Aripiprazole
----------------------------------------
4. A patient is diagnosed with schizophrenia and started on risperidone. After taking the medication last
night, the patient rushed to OPD today, complaining of upward rolling of his eyes. What is the
management of the condition?
A. Lorazepam
B. Promethazine
C. Olanzapine
D. Stop antipsychotic.
----------------------------------------
5. What is the probable diagnosis for a 16-year-old girl who exhibits symptoms such as excessive
familiarity, rapid flow of thoughts, heightened mood, heightened sexual desire, and experiencing
pseudohallucinations?
A. Mania
B. Hypomania
C. Cyclothymia
47
D. Schizomania
----------------------------------------
6. What is the term used to describe a person experiencing a reduced sense of enjoyment in activities
that they previously found pleasurable?
A. Tangentiality
B. Anhedonia
C. Alexithymia
D. Asociality
----------------------------------------
7. What is the probable diagnosis for a 43-year-old individual who arrives at the emergency department
accompanied by the police, alleging that his daughter intends to harm him, but subsequent
investigation by the police reveals it to be a false accusation? The patient's physical examination does
not reveal any notable findings.
A. Delusional disorder
B. Illusions
C. Cotard syndrome
D. Hallucinations
----------------------------------------
8. Which one of the options does not contribute as a risk factor for delusional disorder?
A. Recent immigration
B. Social isolation
C. Family history
D. Young age
----------------------------------------
9. What is the recommended treatment for a 35-year-old female patient presenting to the psychiatric
outpatient department with symptoms of reduced sleep, excessive elation, and increased talkativeness
lasting for one month? The patient is exhibiting disruptive behavior by shouting at the nurses and is
difficult to control. She also reports irritability and difficulty in staying focused. There is no known
medical history for this patient.
A. Lithium
B. Lorazepam
C. Carbamazepine
D. Haloperidol
----------------------------------------
10. A 21-year-old girl suspects that the college principal is making a conspiracy against her and is
planning to fail her in the upcoming exam. She feels that other teachers have also joined the conspiracy
and students are helping them too. What is the diagnosis?
A. Thought insertion
B. Delusion of control
Page 2
48
C. Delusion of persecution
D. Delusion of reference
----------------------------------------
11. What is the probable diagnosis for a 37-year-old man who experienced symptoms after a sudden
financial loss? He reported feeling fearful of harm, having sleep and appetite disturbances, and hearing
abusive voices. These symptoms lasted for about a week. The patient has no relevant medical history,
drug abuse history, or past medical history.
A. Schizophrenia
B. Schizophreniform disorder
C. Brief psychotic episode
D. Schizoaffective disorder
----------------------------------------
12. Which lobes may be impacted in a 40-year-old woman who has a history of urinating in public
without feeling any remorse, along with difficulties in motivation and mild memory deficits?
A. Parietal
B. Occipital
C. Frontal
D. Temporal
----------------------------------------
13. Which of the following factors does not indicate a favorable prognosis in schizophrenia?
A. Positive symptoms
B. Late age of onset
C. Insidious onset of symptoms
D. Associated with depression
----------------------------------------
14. What is the recommended medical approach for a psychotic patient who experiences voluntary
aimless motions and has been observed standing motionless for extended periods with waxy flexibility
and negativism?
A. Haloperidol
B. Lorazepam
C. Clonidine
D. Propanolol
----------------------------------------
15. Which of the following is not considered a core symptom of schizophrenia?
A. Autism
B. Automatism
C. Association defect
Page 3
49
D. Ambivalence
----------------------------------------
16. What is the probable diagnosis for a young male who displays social withdrawal, shyness,
emotional aloofness, and lives in solitude, without any reported instances of hallucinations or
delusions?
A. Paranoid personality disorder
B. Schizoid personality disorder
C. Antisocial personality disorder
D. Emotionally unstable personality disorder
----------------------------------------
17. Which of the following is not considered a negative symptom of schizophrenia?
A. Over activity
B. Anhedonia
C. Alogia
D. Apathy
----------------------------------------
18. What is the most frequently encountered acute organic mental disorder?
A. Delirium
B. Dementia
C. Amnesia
D. Anxiety disorder
----------------------------------------
Correct Answers
Question Correct Answer
Question 1 3
Question 2 3
Question 3 3
Question 4 2
Question 5 1
Question 6 2
Question 7 1
Question 8 4
Question 9 4
Question 10 3
Question 11 3
Page 4
50
Question 12 3
Question 13 3
Question 14 2
Question 15 2
Question 16 2
Question 17 1
Question 18 1
Page 5
51
Option c. An abrupt or sudden onset of mental illness symptoms may indicate a
medical emergency, such as a stroke, brain injury, or infection.
Page 6
52
• Mania is a mood disorder characterized by a distinct period of abnormally elevated, expansive, or
irritable mood and increased energy or activity levels. The symptoms mentioned, such as
overfamiliarity, flight of ideas, elevated mood, increased sexual desire, and pseudohallucination,
are consistent with the manic episode seen in bipolar disorder.
Incorrect Options:
Option B: Hypomania is a less severe form of mania.
Option C: Cyclothymia is a chronic mood disorder characterized by cycling between periods of hyp
omanic and depressive symptoms.
Option D: Schizomania is a
mood disorder with manic and mixed episodes in the bipolar type of schizoaffective disorder.
Page 7
53
Solution for Question 8:
Correct Option D. Young age.
• Delusional disorder is a psychiatric condition characterized by the presence of persistent
delusions without significant impairment in other cognitive functions. Delusions are fixed, false
beliefs that are maintained despite evidence to the contrary. While various factors can contribute to
the development of delusional disorder, young age is not considered a significant risk factor.
Incorrect options:
Option A. Recent immigration: Recent immigration can be a
risk factor for delusional disorder. The stressors and challenges associated with adapting to a new
environment, cultural differences, and social isolation can potentially contribute to the development
of delusions.
Option B. Social isolation: Social isolation or lack of social support is considered a risk factor for del
usional disorder. It can lead to feelings of loneliness, exacerbate existing delusions, and contribute
to the persistence of abnormal beliefs.
Option C. Family history: Family history of psychiatric disorders, including delusional disorder, is co
nsidered a risk factor. There may be a genetic predisposition or shared environmental factors withi
n families that contribute to the development of delusions.
Page 8
54
parent. It is frequently vital to provide prompt symptom alleviation in the acute condition described i
n order to protect the patient's safety and well-being.
Page 9
55
Option B: Here, the duration of symptoms must be more than one month but less than 6
months, according to DSM.
Option D: If a patient has schizophrenia symptoms for 8
months and most of the time also has mood symptoms - schizoaffective disorder.
Page 10
56
Option B. Late age of onset: Late age of onset refers to the development of schizophrenia sympto
ms later in life, typically after the age of 40. While late-onset schizophrenia is associated with better
premorbid functioning and less severe cognitive impairment compared to early-onset schizophreni
a, it is generally considered a better prognostic factor.
Option D. Associated with depression: Comorbid depression in schizophrenia is common and can
significantly impact a person's overall functioning and quality of life. However, the presence of depr
ession alone is not considered a good prognostic factor in schizophrenia.
Page 11
57
atterns. Option D. Ambivalence: Ambivalence, or the coexistence of conflicting feelings or attitudes
, can be present in schizophrenia but is not considered a fundamental symptom. Ambivalence can
manifest as mixed emotions, contradictory thoughts, or indecisiveness. It is more commonly associ
ated with other psychiatric conditions, such as borderline personality disorder, rather than being sp
ecific to schizophrenia.
Page 12
58
Solution for Question 18:
Correct option. A
• Delirium is the most common acute organic mental disorder. It is an acute confusional state
characterized by a disturbance in attention and awareness. Delirium often has a rapid onset and
fluctuating course, and it is commonly caused by underlying medical conditions, such as infections,
metabolic imbalances, drug intoxication, or withdrawal. Delirium is a medical emergency that
requires prompt evaluation and treatment to address the underlying cause and manage the
symptoms.
Incorrect option:
Option B. Dementia: Dementia is a
chronic and progressive neurocognitive disorder characterized by a decline in memory, thinking, be
havior, and the ability to perform daily activities. Unlike delirium, dementia develops slowly and gra
dually over time, and it is not considered an acute disorder. While dementia can cause cognitive im
pairment, it is not the most common acute organic mental disorder.
Option C. Amnesia: Amnesia refers to the loss of memory, either partial or complete, due to variou
s causes such as brain injury, trauma, or psychological factors. While amnesia can occur acutely, it
is not as common as delirium as an acute organic mental disorder. Delirium involves not only mem
ory impairment but also disturbances in attention, awareness, and other cognitive functions.
Option D. Anxiety disorder: Anxiety disorder is a broad term that encompasses various mental heal
th conditions characterized by excessive worry, fear, and anxiety. While anxiety disorders can occu
r acutely, they are not classified as acute organic mental disorders. Acute organic mental disorders
typically refer to conditions with an acute onset and organic (physiological) etiology, such as deliriu
m caused by medical factors.
Page 13
59
Mood Disorders
1. Which of the following neurotransmitters are reduced in a patient with depression?
A. Serotonin, norepinephrine, and dopamine
B. Acetylcholine and cholinergic agonists
C. GABA
D. GABA and dopamine
----------------------------------------
2. Which endocrine disturbance is associated with depression and can be demonstrated by the
dexamethasone suppression test?
A. Elevated HPA axis activity
B. Hypothyroidism
C. Hyperglycemia
D. Decreased cortisol levels
----------------------------------------
3. What is the most likely diagnosis for a patient presenting with a 3-week history of feeling sad,
decreased interest in activities, guilt, worthlessness, easy fatigability, difficulty concentrating, changes
in appetite and weight, psychomotor retardation, and suicidal thoughts, along with difficulty falling
asleep and early morning awakenings?
(or)
A 32-year-old patient presents to the clinic with a 3-week history of feeling sad and having a decreased
interest in previously enjoyed activities. The patient also reports feeling guilty and worthless,
experiencing easy fatigability, and having difficulty concentrating. There have been significant changes
in appetite and weight, with a 7% weight loss over the past month. The patient has been experiencing
psychomotor retardation and has been having suicidal thoughts. On further questioning, the patient
mentioned having difficulty falling asleep and waking up at least 2 hours before the usual time. Based
on these symptoms, what is the most likely diagnosis?
A. Major depressive disorder
B. Bipolar disorder
C. Generalized anxiety disorder
D. Post-traumatic stress disorder
----------------------------------------
4. A 28-year-old woman presents to her primary care physician with a chief complaint of persistent
sadness and loss of interest in activities she used to enjoy. She reports feeling overwhelmed by
feelings of guilt and worthlessness. The patient mentions that she has been experiencing extreme
fatigue, finding it difficult to concentrate on tasks, and has noticed a significant decrease in her appetite,
leading to unintentional weight loss. She states that she has been having trouble falling asleep and
wakes up early in the morning feeling restless. Based on these symptoms , what is the diagnostic
criteria for the above explained clinical scenario?
(or)
Which of the following is the correct statement regarding the Diagnostic criteria for depression?
60
A. Presence of at least 3 symptoms, including depressed mood, for a duration of 2 weeks
B. Presence of at least 4 symptoms, including anhedonia, for a duration of 1 month
C. Presence of at least 5 symptoms, including either depressed mood or anhedonia, for a duration of 2
weeks
D. Presence of at least 5 symptoms, including either depressed mood or anhedonia, for a duration of 1
month
----------------------------------------
5. What is the minimum duration required between episodes to diagnose Recurrent Depressive
Disorder?
A. Minimum duration of 2 weeks between episodes.
B. Minimum duration of 1 month between episodes.
C. Minimum duration of 2 months between episodes.
D. Minimum duration of 6 months between episodes.
----------------------------------------
6. A 35-year-old patient presents with persistent sadness, loss of interest in activities, and decreased
energy for the past few months. The patient also reports changes in appetite, weight loss, and difficulty
sleeping. Based on the symptoms, the patient is diagnosed with major depressive disorder. Which is
the most appropriate treatment option for this patient?
(or)
Which is the most appropriate treatment for the major depressive disorder patient?
A. Cognitive-behavioural therapy (CBT)
B. Selective serotonin reuptake inhibitor (SSRI)
C. Electroconvulsive therapy (ECT)
D. Mindfulness-based stress reduction (MBSR)
----------------------------------------
7. A 35-year-old patient presents with symptoms of depression, including persistent sadness,
decreased energy, feelings of guilt, and suicidal thoughts. The patient also reports experiencing
hallucinations of falling from a building. Which of the following statements accurately describes the
presentation of this patient?
(or)
Which of the following statements accurately describes the hallucinations that are consistent with the
theme of depression?
A. Mood congruent hallucinations indicative of depression with psychotic symptoms
B. Mood incongruent hallucinations indicative of depression with psychotic symptoms
C. Hallucinations are not related to the depressive symptoms, so evaluated separately.
D. Hallucinations are a common side effect of the antidepressant medication.
----------------------------------------
8. Which of the following combinations of medications poses the highest risk for serotonin syndrome?
A. Selective serotonin reuptake inhibitors (SSRIs) + Monoamine oxidase inhibitors (MAOIs)
Page 2
61
B. Tricyclic antidepressants (TCAs) + Benzodiazepines
C. Atypical antipsychotics + Nonsteroidal anti-inflammatory drugs (NSAIDs)
D. Selective serotonin reuptake inhibitors (SSRIs) + Beta-blockers
----------------------------------------
9. A 38-year-old patient presents with symptoms of depression, including a distinct quality of depressed
mood, early morning awakening, profound despondency, weight loss, excessive guilt, and psychomotor
disturbance from the last 3 weeks. The patient reports a loss of pleasure in all activities and anorexia.
Based on the symptoms described, what is the most appropriate diagnosis for this patient?
(or)
What is the most appropriate diagnosis for a patient presenting with symptoms of depression, including
a distinct quality of depressed mood, early morning awakening, profound despondency, weight loss,
excessive guilt, psychomotor disturbance, loss of pleasure in activities, and anorexia from the last 3
weeks?
A. Bipolar disorder with psychotic features
B. Major depressive disorder with melancholic features
C. Generalized anxiety disorder
D. Dysthymic disorder
----------------------------------------
10. Which of the following symptoms are commonly associated with discontinuation syndrome?
A. Dry mouth, blurred vision, and urinary retention
B. Diarrhea, restlessness, and hyperreflexia
C. Agitation, seizures, and hyperthermia
D. Flu-like symptoms, insomnia, and nausea
----------------------------------------
11. What is the most appropriate diagnosis for a patient presenting with symptoms of depression,
including mood reactivity, weight gain, interpersonal rejection sensitivity, leaden paralysis, and
hypersomnia?
(or)
A 30-year-old patient presents with symptoms of depression, including mood reactivity, weight gain,
interpersonal rejection sensitivity, leaden paralysis, and hypersomnia. Based on the symptoms
described, what is the most appropriate diagnosis for this patient?
A. Bipolar disorder with psychotic features
B. Major depressive disorder with atypical features
C. Social anxiety disorder
D. Persistent depressive disorder (dysthymia)
----------------------------------------
12. What is the most appropriate diagnosis for a patient presenting with symptoms of feeling extremely
sad, hopeless, lacking interest in activities, along with prolonged periods of immobility, unusual body
postures, and occasional purposeless movements?
Page 3
62
(or)
A 42-year-old male reports to his psychiatrist and presents with the symptoms of feeling extremely sad,
hopeless, and lacking interest in activities he used to enjoy. He also mentions experiencing significant
changes in his motor behaviour. During the interview, the psychiatrist observes that the patient exhibits
prolonged periods of immobility, adopts unusual body postures, and occasionally engages in repetitive,
purposeless movements. Which of the following is the most appropriate diagnosis for this patient?
A. Major depressive disorder with melancholic features
B. Major depressive disorder with psychotic features
C. Major depressive disorder with atypical features
D. Major depressive disorder with catatonic features
----------------------------------------
13. What is the most common psychiatric disorder associated with suicide?
A. Bipolar disorder
B. Anxiety disorder
C. Schizophrenia
D. Depression
----------------------------------------
14. Which of the following can trigger a hypertensive crisis in individuals taking monoamine oxidase
inhibitors (MAOIs)?
A. Cheese, beer, fish, and red wine
B. Citrus fruits and juices
C. Dairy products
D. Caffeinated beverages
----------------------------------------
15. Which of the following medications is most commonly associated with priapism?
A. Sertraline
B. Lorazepam
C. Trazodone
D. Omeprazole
----------------------------------------
16. Which of the following is true regarding the sexual side effects of Mirtazapine?
A. Erectile dysfunction.
B. Reduces sexual desire
C. Priapism.
D. Low risk of sexual side effects.
----------------------------------------
Page 4
63
17. Which of the following types of bipolar disorder is characterized by at least one episode of
hypomania and one episode of depression?
A. Bipolar I disorder
B. Bipolar II disorder
C. Mixed episode bipolar disorder
D. Rapid cycling bipolar disorder
----------------------------------------
18. What is the most likely diagnosis for a patient presenting with a one-week history of significantly
elevated mood, increased energy, decreased need for sleep, excessive talkativeness, feeling
invincible, and engaging in impulsive and risky behaviors?
(or)
A 32-year-old male presents with a one-week history of significantly elevated mood, increased energy,
decreased need for sleep, and excessive talkativeness. He reports feeling invincible and engages in
impulsive and risky behaviours, such as excessive spending and engaging in multiple sexual
encounters. Which of the following is the most likely diagnosis?
A. Major depressive disorder
B. Mania
C. Generalized anxiety disorder
D. Borderline personality disorder
----------------------------------------
19. A patient presents with symptoms of elevated mood, increased energy, and decreased need for
sleep. The symptoms have been present for the past 4 days and are not severe enough to cause
marked impairment in social or occupational functioning. Which of the following is the most likely
diagnosis?
(or)
What is the most likely diagnosis for a patient presenting with symptoms of elevated mood, increased
energy, and decreased need for sleep that have been present for the past 4 days without causing
marked impairment in social or occupational functioning?
A. Major depressive disorder
B. Hypomania
C. Generalized anxiety disorder
D. Borderline personality disorder
----------------------------------------
20. A 26-year-old patient with a known diagnosis of bipolar disorder presents with symptoms of
depression, including low mood, loss of interest, and decreased energy. Which of the following
medications is commonly used as a first-line treatment for bipolar depression?
(or)
Which of the following medications is commonly used as a first-line treatment for bipolar depression?
A. Fluoxetine
B. Carbamzepine
Page 5
64
C. Quetiapine
D. Alprazolam
----------------------------------------
21. Which of the following statements regarding the treatment of acute depression in bipolar disorder is
correct?
A. Antidepressants should be used as the first-line treatment.
B. Combination therapy with an antidepressant and mood stabilizer is recommended.
C. Antidepressant use should be avoided completely.
D. Medications such as Lithium, Lamotrigine are ineffective.
----------------------------------------
22. Ms. A is a 32-year-old female who presents to the mental health clinic with a chief complaint of long
standing low mood and a feeling of emptiness. She reports that she has been experiencing these
symptoms for as long as she can remember. Ms. A describes her mood as consistently "down" and
states that she rarely experiences moments of joy or happiness. She acknowledges that she has
become used to feeling this way over the years. Which of the following best describes her condition?
(or)
What best describes the condition of a 32-year-old patient experiencing a long standing low mood,
feeling of emptiness, consistently "down" and states that she rarely experiences moments of joy or
happiness from the last 5 years?
A. Major depressive disorder
B. Bipolar disorder
C. Persistent Depressive Disorder
D. Generalized anxiety disorder
----------------------------------------
23. A 35-year-old individual presents with a history of alternating periods of elevated mood and mild
depressive symptoms. The symptoms are not severe enough to meet the criteria for a full manic or
depressive episode. The patient reports functional impairment but it is not severe. The symptoms have
been present for the past 2 years. What is the most likely diagnosis for this patient based on the given
information?
(or)
What is the most likely diagnosis for a 35-year-old individual with a history of alternating periods of
elevated mood and mild depressive symptoms, not meeting the criteria for a full manic or depressive
episode, and experiencing functional impairment that is not severe for the past 2 years?
A. Bipolar I disorder
B. Bipolar II disorder
C. Dysthymia
D. Cyclothymia
----------------------------------------
24. What is the most appropriate diagnosis for a 30-year-old woman experiencing irritability, depressed
mood, breast tenderness, wanting to stay home and changes in eating patterns that consistently occur
Page 6
65
1 week before her menstrual cycle and improve after the onset of menses?
(or)
A 30-year-old woman presents with symptoms of irritability, depressed mood, breast tenderness, and
changes in eating patterns. These symptoms consistently occur 1 week before her menstrual cycle and
improve after the onset of menses. During these days, she finds herself availing leave from her work
and wanting to stay home. Which of the following is the most appropriate diagnosis for her condition?
A. Premenstrual syndrome (PMS)
B. Pre-Menstrual Dysphoric Disorder (PMDD)
C. Major Depressive Disorder (MDD)
D. Generalized Anxiety Disorder (GAD)
----------------------------------------
25. What is the appropriate treatment for post partum depression?
(or)
Sarah, a 32-year-old woman, gave birth to her first child three weeks ago. She has been experiencing
feelings of sadness, guilt, and tearfulness since then. She also struggles with insomnia and has noticed
changes in her appetite and weight. She is worried about her ability to care for her baby and
occasionally has thoughts of harming herself. What is the most appropriate treatment for Sarah?
A. Lifestyle changes and self-care strategies
B. Pain relievers and relaxation techniques
C. Pharmacotherapy and psychotherapy
D. Support groups and counseling
----------------------------------------
26. Rebecca, a 28-year-old woman, recently gave birth to her second child. Two weeks after delivery,
she started experiencing severe insomnia, excessive tearfulness, and extreme mood swings. She also
began to have delusions and hallucinations, believing that her baby was possessed. There is a history
of mood disorder in her family. What is the most appropriate initial treatment for Rebecca?
(or)
What is the appropriate treatment for the female patient who experiencing severe insomnia and
excessive tearfulness and extreme mood swings after 2 weeks of delivery
A. Antipsychotic medication only
B. Lithium medication only
C. Combination of antipsychotic and lithium medication
D. Antidepressant medication only
----------------------------------------
27. What is the most likely condition experienced 29-year-old woman who gave birth one week ago,
with symptoms of mood swings, tearfulness, and irritability, but still able to care for herself and her
baby?
(or)
Jennifer, a 29-year-old woman, gave birth to her first child one week ago. She has been experiencing
mood swings, tearfulness, and irritability since the birth. She feels overwhelmed at times but is able to
Page 7
66
take care of herself and her baby. What is the most likely condition Jennifer is experiencing?
A. Postpartum depression
B. Postpartum psychosis
C. Postpartum blues
D. Postpartum anxiety disorder
----------------------------------------
28. Which of the following is not a known risk factor for suicide?
A. Previous suicide attempts
B. Male gender, age over 45
C. Family history of suicide
D. Being married
----------------------------------------
29. Which of the following endocrine effects is associated with the use of lithium?
A. Hypothyroidism
B. Hyperglycemia
C. Hypernatremia
D. Hypercalcemia
----------------------------------------
30. During a routine psychiatric evaluation, a 40-year-old patient presents with symptoms of persistent
sadness, loss of interest, and feelings of worthlessness. The patient's facial expression appears flat
and lacking in emotional responsiveness. Additionally, the physician notices a physical finding as seen
in the picture below. Based on these observations, what clinical signs are associated with this patient's
depression?
(or)
What clinical signs are associated with the depression of a 40-year-old patient who presents with
persistent sadness, loss of interest, feelings of worthlessness, and a flat facial expression? Additionally,
a physical finding, as seen in the picture below, is noted. What clinical sign is related to the depression
exhibited by this patient?
Page 8
67
C. Psychomotor agitation and Leaden paralysis
D. Suicidal ideation and Pseudobulbar affect
----------------------------------------
31. All of the following drugs increase renal clearance of lithium except:
A. Loop diuretics
B. K+ sparing diuretics
C. Xanthine
D. Methazolamide
----------------------------------------
32. A 26-year-old patient with a history of bipolar disorder and ongoing treatment presents to the OPD
with new-onset convulsions, muscle weakness, decreased urine output, and impaired renal function.
The doctor advises the patient to discontinue the current treatment. What is the next best step in
managing this patient?
A. IV Sodium bicarbonate
B. Activated charcoal
C. Hemodialysis
D. 1g Pralidoxime
----------------------------------------
Correct Answers
Question Correct Answer
Question 1 1
Question 2 1
Question 3 1
Question 4 3
Question 5 3
Question 6 2
Question 7 1
Question 8 1
Question 9 2
Question 10 4
Question 11 2
Question 12 4
Question 13 4
Question 14 1
Question 15 3
Page 9
68
Question 16 4
Question 17 2
Question 18 2
Question 19 2
Question 20 3
Question 21 2
Question 22 3
Question 23 4
Question 24 2
Question 25 3
Question 26 3
Question 27 3
Question 28 4
Question 29 1
Question 30 2
Question 31 2
Question 32 3
Page 10
69
Correct Option is A: Elevated HPA axis activity
• Endocrinal disturbance associated with depression includes elevated HPA
(Hypothalamic–pituitary–adrenal) axis activity, which can be demonstrated by the dexamethasone
suppression test. In depression, there is hypercortisolemia, and the corticoid surge does not get
suppressed as it normally would in response to dexamethasone, indicating an increase in the HPA
axis activity.
Incorrect Options:
Option B - Hypothyroidism
• While hypothyroidism is associated with depression, it is not specifically diagnosed using a
dexamethasone suppression test or related to elevated HPA axis activity.
Option C - Hyperglycemia
• Hyperglycemia refers to high blood sugar levels and is not directly associated with endocrinal
disturbances in depression.
Option D - Decreased cortisol levels
• Decreased cortisol levels are not typically observed in depression but rather elevated cortisol
levels due to the dysregulation of the HPA axis.
Page 11
70
traumatic event or symptoms associated with PTSD.
Page 12
71
Option A - Cognitive Behavioural Therapy (CBT): While CBT is a
valuable and evidence-based psychotherapy for depression, the question specifically asks for a
treatment option. CBT alone may be a suitable treatment option for some patients, but in this scen
ario, pharmacotherapy is more appropriate considering the severity of the symptoms and the need
for intervention.
Option C - Electroconvulsive therapy (ECT): ECT is generally reserved for cases of severe or treat
ment-resistant depression when other treatment modalities have been ineffective. It involves the ad
ministration of controlled electric currents to induce a brief seizure, which is believed to have a ther
apeutic effect on depression. ECT is typically considered when other treatment options have failed
or in cases of urgent clinical need.
Option D - Mindfulness-based stress reduction (MBSR): MBSR is a form of psychotherapy that inc
orporates mindfulness meditation and stress reduction techniques. While it can be beneficial for m
anaging stress and improving overall well-being, it may not be the primary treatment choice for maj
or depressive disorder. In most cases, MBSR would be used as an adjunct to other treatment mod
alities, such as medication or psychotherapy.
Page 13
72
• The combination of selective serotonin reuptake inhibitors (SSRIs) and monoamine oxidase
inhibitors (MAOIs) poses the highest risk for serotonin syndrome. SSRIs increase serotonin levels
by blocking its reuptake, while MAOIs inhibit the breakdown of serotonin, leading to an excess of
serotonin in the brain. When these medications are taken together, the accumulation of serotonin
can result in serotonin syndrome, a potentially life-threatening condition.
• Serotonin syndrome is a potentially life-threatening condition characterized by an excess of
serotonin, a neurotransmitter, in the central nervous system. It typically results from the use of
serotonergic medications that increase serotonin levels.
Page 14
73
• Duration: The duration of symptoms can vary, but they often resolve within a few weeks to a
month.
• Symptoms: The specific symptoms depend on the class of medication but may include flu-like
symptoms (lethargy, weakness, headache), gastrointestinal disturbances, mood changes,
dizziness, and sensory disturbances.
Incorrect Options:
Option A - Dry mouth, blurred vision, and urinary retention: These symptoms are commonly associ
ated with anticholinergic side effects of certain medications, but they are not characteristic of disco
ntinuation syndrome.
Option B - Diarrhoea, restlessness, and hyperreflexia: These symptoms are more commonly seen i
n serotonin syndrome, which is a potentially life-threatening condition caused by excessive seroton
in activity. They are not typically observed in discontinuation syndrome.
Option C - Agitation, seizures, and hyperthermia: These symptoms are also indicative of serotonin
syndrome, which is a
severe and potentially life-threatening condition. They are not typical of discontinuation syndrome.
Page 15
74
• The patient's symptoms of depression, including sadness, hopelessness, and loss of interest,
along with the presence of catatonic behaviour, indicate a diagnosis of major depressive disorder
with catatonic features. Catatonic features in depression involve motor abnormalities such as
immobility, adopting unusual body postures, and engaging in repetitive movements.
Incorrect Options:
Option A - Major depressive disorder with melancholic features: Major depressive disorder with mel
ancholic features is characterized by a distinct quality of depressed mood, anhedonia (loss of pleas
ure), psychomotor disturbance (often agitation or retardation), early morning awakening, significant
weight loss, excessive guilt, and worse mood in the morning. While this patient displays motor abn
ormalities, the scenario does not provide sufficient information to support the diagnosis of melanch
olic features.
Option B - Major depressive disorder with psychotic features: Major depressive disorder with psych
otic features involves the presence of delusions or hallucinations during a depressive episode. The
se delusions or hallucinations are typically mood congruent, meaning they are consistent with the t
heme of depression. The scenario does not provide any information suggesting the presence of ps
ychotic symptoms.
Option C - Major depressive disorder with atypical features: Major depressive disorder with atypical
features is characterized by a specific set of symptoms, including mood reactivity (mood brightens
in response to positive events), significant weight gain or increased appetite, hypersomnia, a feelin
g of heaviness in the limbs (leaden paralysis), and interpersonal rejection sensitivity. While this pati
ent exhibits some motor abnormalities, the presence of atypical features is not supported by the sc
enario.
Page 16
75
Solution for Question 15:
Correct Option is C: Trazodone
• Trazodone, an antidepressant with sedating properties, has been associated with priapism.
Priapism is an unwanted, prolonged, and painful erection that is unrelated to sexual stimulation.
Incorrect Options:
Option A - Sertraline: Sertraline is a selective serotonin reuptake inhibitor (SSRI) commonly used f
or the treatment of depression and anxiety disorders. While SSRIs may have sexual side effects, p
riapism is not typically associated with sertraline.
Option B - Lorazepam: Lorazepam is a
benzodiazepine used to treat anxiety and insomnia. It is not known to cause priapism.
Option D - Omeprazole: Omeprazole is a
proton pump inhibitor used to reduce stomach acid production. It is not known to cause priapism.
Page 17
76
• Rapid cycling bipolar disorder refers to experiencing four or more mood episodes within a year.
Incorrect choices:
Option A: Bipolar I disorder:
• Bipolar I disorder is characterized by at least one episode of mania, which is a more severe and
disruptive mood state than hypomania. A diagnosis of bipolar I does not require a history of major
depression, but episodes of depression can occur alongside manic episodes.
Option C: Mixed episode bipolar disorder:
• Mixed episode bipolar disorder involves the simultaneous occurrence of symptoms of both mania
(or hypomania) and depression. This means that an individual experiences symptoms of elevated
mood (mania or hypomania) and symptoms of depression at the same time, which can be
distressing and challenging to manage.
Option D: Rapid cycling bipolar disorder:
• Rapid cycling is not a distinct type of bipolar disorder but rather a specifier that can be applied to
bipolar I or bipolar II. It is defined as having at least four or more mood episodes (mania,
hypomania, depression, or mixed) within a year. Rapid cycling does not determine the presence of
hypomania and depression; it relates to the frequency and pattern of mood episodes.
Page 18
77
• In certain cases, benzodiazepines like lorazepam or clonazepam may be used for short-term
management of acute agitation and insomnia.
• Psychotherapy: Cognitive-Behavioral Therapy (CBT): CBT can be beneficial in addressing
distorted thought patterns and behaviors associated with manic episodes. Family-Focused
Therapy: Involving family members in the treatment process can be helpful in providing support and
education.
• Cognitive-Behavioral Therapy (CBT): CBT can be beneficial in addressing distorted thought
patterns and behaviors associated with manic episodes.
• Family-Focused Therapy: Involving family members in the treatment process can be helpful in
providing support and education.
• Hospitalization: Severe manic episodes may require hospitalization, especially if there is a risk of
harm to oneself or others. Inpatient care allows for close monitoring, medication adjustments, and
stabilization.
• Severe manic episodes may require hospitalization, especially if there is a risk of harm to oneself
or others.
• Inpatient care allows for close monitoring, medication adjustments, and stabilization.
• Electroconvulsive Therapy (ECT): ECT may be considered for individuals who do not respond to
or cannot tolerate medications. It is often used in severe cases or when a rapid response is
necessary.
• ECT may be considered for individuals who do not respond to or cannot tolerate medications. It is
often used in severe cases or when a rapid response is necessary.
Here are common approaches to treating mania:
Mood Stabilizers:
• Lithium: Lithium is a classic mood stabilizer that has proven effective in managing manic
episodes. Regular blood tests are required to monitor lithium levels and kidney function.
• Anticonvulsant Medications: Drugs such as valproate (divalproex) and carbamazepine may be
used as mood stabilizers.
Atypical Antipsychotics:
• Medications like olanzapine, quetiapine, risperidone, and aripiprazole are often prescribed to
control acute manic symptoms.
• Some atypical antipsychotics may be used as monotherapy or in combination with mood
stabilizers.
Benzodiazepines:
• In certain cases, benzodiazepines like lorazepam or clonazepam may be used for short-term
management of acute agitation and insomnia.
Psychotherapy:
• Cognitive-Behavioral Therapy (CBT): CBT can be beneficial in addressing distorted thought
patterns and behaviors associated with manic episodes.
• Family-Focused Therapy: Involving family members in the treatment process can be helpful in
providing support and education.
Hospitalization:
Page 19
78
• Severe manic episodes may require hospitalization, especially if there is a risk of harm to oneself
or others.
• Inpatient care allows for close monitoring, medication adjustments, and stabilization.
Electroconvulsive Therapy (ECT):
• ECT may be considered for individuals who do not respond to or cannot tolerate medications. It is
often used in severe cases or when a rapid response is necessary.
Incorrect Options:
Option A - Major depressive disorder: The patient's symptoms are consistent with mania rather tha
n depression. Major depressive disorder is characterized by persistent feelings of sadness, hopele
ssness, and loss of interest or pleasure in activities.
Option C - Generalized anxiety disorder: The patient's symptoms are not indicative of generalized
anxiety disorder. Generalized anxiety disorder is characterized by excessive worry and anxiety abo
ut various events or activities.
Option D - Borderline personality disorder: The patient's symptoms are more indicative of a
mood disorder (mania) rather than a personality disorder. Borderline personality disorder is charact
erized by unstable moods, impulsivity, and difficulties with interpersonal relationships.
Page 20
79
Typically less severe and less disruptive to daily life
Symptoms
Exaggerated elevation in mood
Elevated mood, increased activity
Energy
Highly energetic, excessive
Increased but manageable
Risky Behavior
Pronounced and risky behavior
Risk-taking behavior, but less extreme
Psychotic Features
Often present (hallucinations, delusions)
Absent
Incorrect Options:
Option A - Major depressive disorder: The patient does not exhibit symptoms of a major depressive
episode. Major depressive disorder is characterized by persistent feelings of sadness, hopelessne
ss, and loss of interest or pleasure in activities. The scenario describes an elevated mood, which is
inconsistent with a major depressive episode.
Option C - Generalized anxiety disorder: The patient's symptoms are not indicative of generalized
anxiety disorder. Generalized anxiety disorder is characterized by excessive worry and anxiety abo
ut various events or activities, whereas the scenario describes an elevated mood and increased en
ergy.
Option D - Borderline personality disorder: The patient's symptoms are more indicative of a
mood disorder (hypomania) rather than a personality disorder. Borderline personality disorder is ch
aracterized by unstable moods, impulsivity, and difficulties with interpersonal relationships.
Page 21
80
Solution for Question 21:
Correct Option B: Combination therapy with an antidepressant and mood stabilizer is recommended.
• The treatment of acute depression in bipolar disorder requires caution due to the risk of inducing
manic or hypomanic episodes with antidepressant use alone. Therefore, combination therapy with
an antidepressant and mood stabilizer is often recommended to balance the mood and minimize
the risk of switching to mania.
• Medications such as Lithium, Lamotrigine, Quetiapine, Lurasidone, and Olanzapine + Fluoxetine
have shown efficacy in treating acute depression in bipolar disorder.
Incorrect options:
Option A:Antidepressants should be used as the first-line treatment - Antidepressants alone are ge
nerally not recommended as the first-line treatment for acute depression in bipolar disorder. Their u
se can carry a risk of inducing manic or hypomanic episodes, which can worsen the condition. Ther
efore, using antidepressants without a mood stabilizer is often discouraged.
Option C:Antidepressant use should be avoided completely - Antidepressants
be used in combination with a mood stabilizer to minimize the risk of inducing mania or hypomania.
Option D:Medications such as Lithium, Lamotrigine are ineffective
- Medications like Lithium, Lamotrigine, Quetiapine, Lurasidone, and Olanzapine + Fluoxetine are o
ften used as treatment options for acute bipolar depression. They are considered effective and ma
y be used as mood stabilizers or adjuncts to manage depressive symptoms in bipolar disorder.
Page 22
81
• Based on the clinical presentation of alternating periods of elevated mood and mild depressive
symptoms, along with functional impairment that is not severe, the most likely diagnosis for this
patient is cyclothymia.
Incorrect Options:
Option A - Bipolar I disorder: Bipolar I
disorder is characterized by the presence of at least one manic episode, which is a distinct period o
f abnormally elevated mood and energy and one episode of depression. This patient's symptoms d
o not meet the criteria for a manic episode, making bipolar I disorder an incorrect option.
Option B - Bipolar II disorder: Bipolar II disorder is characterized by the presence of at least one hy
pomanic episode and one major depressive episode. Hypomanic episodes are less severe than full
manic episodes. However, this patient's symptoms do not meet the criteria for a
hypomanic episode, ruling out bipolar II disorder as the diagnosis.
Option C - Dysthymia: Dysthymia, also known as persistent depressive disorder, involves chronic d
epressive symptoms that last for at least 2 years in adults and 1 year in children. While this patient
does experience mild depressive symptoms, the presence of alternating periods of elevated mood i
ndicates a different diagnosis than dysthymia.
Page 23
82
Solution for Question 25:
Correct Option C: Pharmacotherapy and psychotherapy
• Postpartum depression is a serious condition that requires professional intervention. Treatment
typically involves a combination of pharmacotherapy, such as antidepressant medications, and
psychotherapy to address the underlying causes and provide support.
Incorrect Options:
Option A - Lifestyle changes and self-care strategies: Lifestyle changes and self-care strategies ma
y be helpful adjuncts to treatment, but alone they are unlikely to be sufficient for managing the sym
ptoms of postpartum depression.
Option B - Pain relievers and relaxation techniques: Pain relievers and relaxation techniques are n
ot the primary treatments for postpartum depression.
Option D
- Support groups and counseling: Support groups and counselling can be beneficial as part of a co
mprehensive treatment plan, but they alone may not provide adequate relief for the severity of sym
ptoms described in the scenario.
Page 24
83
Option D - Postpartum anxiety disorder: Postpartum anxiety disorder involves excessive worry, rest
lessness, and fear, which are not the primary symptoms described in the scenario.
Page 25
84
to excessive use of the corrugator muscle, which can be observed in some patients with
depression. The other options mentioned in the incorrect options are either unrelated or do not
match the clinical signs described in the scenario. Psychomotor agitation is excessive motor activity
often seen in certain mental disorders, and suicidal ideation is the presence of thoughts related to
self-harm or suicide.
Incorrect Options:
Option A - Loop diuretics: Loop diuretics (e.g., furosemide) are known to increase the renal clearan
ce of lithium. They inhibit the sodium-potassium-chloride cotransporter in the thick ascending limb
of the loop of Henle, leading to increased sodium and water excretion and subsequently increased
lithium clearance.
Option C - Xanthine: Xanthines, like caffeine, can lead to increased lithium excretion. They work by
inhibiting sodium reabsorption in the proximal tubules of the nephron, which indirectly affects lithiu
m reabsorption as well.
Option D - Methazolamide: Methazolamide is a carbonic anhydrase inhibitor that can increase the r
enal clearance of lithium. It reduces bicarbonate reabsorption in the proximal tubules, causing an in
crease in urinary excretion of lithium.
Incorrect Options:
Option A - IV Sodium Bicarbonate: While sodium bicarbonate can be used in some cases of lithium
toxicity, it is not the first-line treatment for severe lithium toxicity with impaired renal function. Hem
odialysis is a more effective and rapid method of removing excess lithium.
Option B - Activated Charcoal: Activated charcoal is not effective in treating lithium toxicity. It is typi
cally used for acute poisoning with substances that can be adsorbed by activated charcoal, which i
s not the case with lithium.
Page 26
85
Option D - 1g Pralidoxime: Pralidoxime is not indicated for lithium toxicity. It is used for a completel
y different category of poisoning involving organophosphates and is unrelated to lithium toxicity ma
nagement.
Page 27
86
Previous Year Questions
1. What is the probable diagnosis for a woman who is 4 days postpartum and experiencing tearfulness,
mood swings, and occasional insomnia?
A. Postpartum depression
B. Postpartum blues
C. Postpartum psychosis
D. Postpartum anxiety
----------------------------------------
2. Which of the following teratogenic effects can occur as a result of lithium usage during pregnancy?
A. Cardiac defects
B. Facial defects
C. CNS defects
D. Urogenital defects
----------------------------------------
3. In your OPD, a young male patient with a paternal family history of bipolar disorder has presented
with three episodes of mania. Considering the most effective treatment, which of the following mood
stabilizers can be prescribed to him?
A. Valproate
B. Lamotrigine
C. Olanzapine
D. Lithium
----------------------------------------
4. Which antidepressant would you recommend for a 38-year-old professor experiencing depression,
who is concerned about potential sexual dysfunction as a side effect?
A. Bupropion
B. Escitalopram
C. Venlafaxine
D. Fluoxetine
----------------------------------------
5. What is the most probable diagnosis for a woman who is experiencing tearfulness, mood swings,
and occasional insomnia, and is 4 days postpartum?
A. Postpartum depression
B. Postpartum blues
C. Postpartum psychosis
D. Postpartum anxiety
----------------------------------------
87
6. Which statement is incorrect regarding bipolar disorder?
A. It most commonly affects ages 35-45 years
B. Type 1 is common in both sexes
C. Prevalence in the general population is 1-2%
D. Attempt to commit suicide seen in around 15% of people
----------------------------------------
7. A pregnant mother was on SSRIs. The risk of which of the following is increased in the newborn?
Low APGAR score ADHD Persistent pulmonary hypertension Delayed developmental milestones
A. A, C
B. A, B, C, D
C. B, C, D
D. A, B, C
----------------------------------------
8. The patient expresses to the psychiatrist: "I feel like my brain is gone. What is the purpose of eating?
I am already deceased." What type of delusion does the patient exhibit?
A. Delusion of misidentification
B. Nihilistic delusion
C. Bizarre Delusion
D. Hypochondriacal Delusion
----------------------------------------
9. What could be the probable diagnosis for a woman who has been brought to the psychiatrist 5 weeks
post-delivery, displaying symptoms of fearfulness, irritability, and suicidal ideation?
A. Post-partum blues
B. Post-partum psychosis
C. Post-partum depression
D. Panic disorder
----------------------------------------
10. What is the diagnosis for a 21-year-old female who avoids using elevators due to fear and always
opts for stairs instead?
A. Social phobia
B. Agoraphobia
C. Specific phobia
D. Panic disorder
----------------------------------------
11. A Clinician asks a patient “how is your mood”. Patient replies by saying “ It is going up & down”.
What is the phenomenon seen here?
A. Derailment
Page 2
88
B. Tangentiality
C. Clang association
D. Neologism
----------------------------------------
12. What is the most probable diagnosis for a 60-year-old man who is grieving the loss of his wife and
expressing guilt, believing his intestines have deteriorated and that he deserves to be imprisoned? He
also reports persistent sadness and a lack of interest in daily activities since his wife's passing.
A. Normal grief reaction
B. Psychotic depression
C. Delusional disorder
D. Schizophrenia
----------------------------------------
13. What is the preferred treatment option for postpartum blues?
A. Fluoxetine
B. Cognitive behavioral therapy
C. Lithium carbonate
D. Support and Education
----------------------------------------
14. What diagnosis according to DSM-V would be appropriate for a middle-aged female patient who
has been experiencing symptoms such as loss of interest in daily activities, weight loss, insomnia, and
persistent sadness for a period of six months? Additionally, she reports a single incident of hearing a
voice instructing her to commit suicide.
A. Major depressive disorder with psychotic features
B. Schizophrenia
C. Schizoaffective disorder
D. Delusional disorder
----------------------------------------
15. What is the recommended treatment for a recently diagnosed female patient with mild depression?
A. Cognitive behavioral therapy
B. Cognitive behavioral therapy and antidepressants
C. Mild antidepressants
D. Vagus nerve stimulation
----------------------------------------
16. Which of the following disorders may present with symptoms similar to Addison's disease?
A. Depression
B. Generalised anxiety disorder
C. Bipolar disorder
Page 3
89
D. Panic disorder
----------------------------------------
17. What is the most common emotional state?
A. Love
B. Hate
C. Anger
D. Anxiety
----------------------------------------
18. What should be the next course of action for a 31-year-old woman who recently went through a
divorce and is currently experiencing symptoms such as palpitations, sweating, chest heaviness, and
pain? She has had several similar episodes over the past few weeks, but her blood test results show no
abnormalities.
A. Admit for detailed investigations
B. Start counseling sessions
C. Use beta-blockers
D. Use benzodiazepines for symptom relief
----------------------------------------
19. Abrupt stoppage of which of the following drugs causes irritability,rebound anxiety, and insomnia?
A. Valproate
B. Fluoxetine
C. Olanzepine
D. Venlafaxine
----------------------------------------
20. Which one of the following characteristics is not typically associated with atypical depression?
A. Increased sleep
B. Reactive mood response to positive stimulus
C. The heaviness of the body
D. Responds better to TCAs than MAO-I and SSRI
----------------------------------------
21. What was the intended function of thiopentone and succinylcholine given to a patient receiving ECT
treatment for severe depression?
A. To bring calming effect
B. To produce mood elevation
C. To induce anesthesia and relax muscles
D. To prevent memory loss after the ECT
----------------------------------------
22. Which neurotransmitters are implicated in depression?
Page 4
90
A. GABA and dopamine
B. Serotonin and norepinephrine
C. Dopamine and serotonin
D. GABA and norepinephrine
----------------------------------------
23. Which of the following side effects would most likely be caused by a drug that specifically
decreases the reabsorption of serotonin?
A. Constipation
B. Sexual dysfunction
C. Siallorhea
D. Visual disturbance
----------------------------------------
24. Which of the following adverse effects is the least probable in a patient prescribed with
escitalopram?
A. Nausea
B. Vivid dreams
C. Anorgasmia
D. Sialorrhea
----------------------------------------
25. Double depression in:-
A. Major depressive disorder from 2 years
B. Major depression with OCD
C. Depression with dysthymia
D. Depression with anxiety attack
----------------------------------------
26. What is the diagnosis of a 40-year-old patient who presents with a significant history of mood
disorder, as depicted in the provided image?
A. Cyclothymia
B. Dysthymia
C. Rapid cycling disorder
D. Bipolar disorder
----------------------------------------
27. Which of the options below does not characterize melanocholia?
A. Severe anhedonia
B. Weight loss
C. Sleep-onset insomnia
Page 5
91
D. Profound feelings of guilt over trivial events
----------------------------------------
Correct Answers
Question Correct Answer
Question 1 2
Question 2 1
Question 3 4
Question 4 1
Question 5 2
Question 6 1
Question 7 2
Question 8 2
Question 9 3
Question 10 3
Question 11 2
Question 12 2
Question 13 4
Question 14 1
Question 15 1
Question 16 1
Question 17 1
Question 18 4
Question 19 4
Question 20 4
Question 21 3
Question 22 2
Question 23 2
Question 24 4
Question 25 3
Question 26 3
Question 27 3
Page 6
92
• The symptoms of tearfulness, occasional insomnia, and mood swings in a woman who is 4 days
postpartum indicate the diagnosis of postpartum blues. Postpartum blues, also known as "baby
blues," is a common condition that affects up to 80% of women after giving birth. It is a mild and
self-limiting condition characterized by symptoms such as mood swings, tearfulness, anxiety,
irritability, and sleep disturbances. The symptoms usually appear within the first few days after
delivery and resolve within a week or two without requiring any treatment.
Incorrect Options:
Option A: Postpartum depression: After giving birth, postpartum depression can also happen, alth
ough it often manifests with more severe and enduring symptoms, including depressed mood, loss
of interest, feelings of guilt or worthlessness, and suicidal thoughts. Postpartum depression typicall
y begins within the first few weeks following delivery and, if ignored, can linger for months to years.
Option C: Postpartum psychosis: Postpartum psychosis is a rare but severe condition that can occ
ur within the first few weeks after childbirth. It is characterized by symptoms such as delusions, hall
ucinations, disorientation, and severe mood swings. Women with postpartum psychosis require urg
ent medical attention and may need to be hospitalized.
Optoin D: Postpartum anxiety: While anxiety can be a symptom of postpartum blues, it is not typica
lly the primary presenting symptom. Postpartum anxiety can also occur after childbirth and is chara
cterized by excessive worry, restlessness, and physical symptoms such as palpitations and shortn
ess of breath.
Page 7
93
• Mood stabilizers are the primary class of medications used to treat bipolar disorder. They work to
help stabilize mood and prevent episodes of mania or depression.
• In this case, the patient has a family history of bipolar disorder and has already experienced three
episodes of mania. Lithium is considered the most effective mood stabilizer for treating bipolar
disorder, particularly for preventing future episodes of mania.
Incorrect Choices
Option a: Valproate: Valproate is a mood stabilizer that can be used to treat bipolar disorder. Howe
ver, it is typically more effective at preventing future episodes of depression than mania.
Option b: Lamotrigine: Lamotrigine is a mood stabilizer that can be used to treat bipolar disorder. H
owever, it is less effective at preventing manic episodes than lithium.
Option c: Olanzapine: Olanzapine is an atypical antipsychotic medication that can be used to treat
bipolar disorder. It is effective in treating manic episodes but is not typically used as a
first-line treatment for bipolar disorder.
Page 8
94
Solution for Question 5:
Correct Option B:
• Postpartum blues, also known as "baby blues," is a common and normal reaction following
childbirth and is characterized by transient mood lability, tearfulness, anxiety, occasional insomnia,
and irritability. It usually occurs within the first week postpartum and spontaneously resolves within
two weeks.
Incorrect Options:
Option A. Postpartum depression: Postpartum depression typically involves more severe symptom
s than postpartum blues, including persistent depressed mood, feelings of worthlessness, and dimi
nished interest or pleasure in almost all activities. It typically emerges over the first 2-3 months post
partum but can occur at any point after delivery.
Option C. Postpartum psychosis: Postpartum psychosis is a rare and severe condition that often pr
esents abruptly in the first 1-2 weeks postpartum. Symptoms include delusions, hallucinations, and
rapid mood swings.
Option D. Postpartum anxiety: Postpartum anxiety can involve excessive worry, restlessness, and i
rritability. However, it typically presents with more intense symptoms than those described in the q
uestion.
Page 9
95
Solution for Question 7:
Correct Option B - A, B, C, D:
• Babies born to mothers who took selective serotonin reuptake inhibitors during pregnancy are at a
high risk of developing pulmonary hypertension, low APGAR score, ADHD, and delayed
developmental milestones. Similarly, paroxetine should be avoided in pregnancy as it is known to
cause serious side effects (heart defects) in the baby (ASD, VSD).
Page 10
96
Solution for Question 10:
Correct Option:
Option C: Specific phobia
• Specific phobias involve strong, persistent, and irrational fear of an object or situation. The phobia
of elevators or lifts given in the question is a situational phobia.
Incorrect Options:
Option A: Social Phobia involves intense fear and anxiety in social situations. It is also known as a
social anxiety disorder. Individuals with this phobia usually fear embarrassment or humiliation in fro
nt of others.
Option B: Individuals with Agoraphobia has the fear of places which are difficult to escape. For exa
mple, fear of open spaces, travelling alone, crowded places, or public transport.
Option D: Panic disorder involves recurring panic attacks, but the fear here is specifically related to
elevators rather than panic attacks.
Page 11
97
his intestines being rotten and feeling deserving of imprisonment, suggesting the presence of
psychotic symptoms. These delusions are not based on reality and are consistent with the features
of psychotic depression.
Incorrect options:
Option A: Normal grief reaction: While it is normal to experience grief and sadness after the loss of
a loved one, the presence of delusions and severe depressive symptoms suggests a
more severe condition than a typical grief reaction.
Option C: Delusional disorder: Delusional disorder is characterized by persistent delusions without
prominent mood symptoms such as depression. The presence of depressive symptoms in addition
to delusions points more towards psychotic depression.
Option D: Schizophrenia: Schizophrenia is a chronic psychiatric disorder characterized by a combi
nation of symptoms, including hallucinations, delusions, disorganized thinking, and negative sympt
oms. While the patient in the scenario has delusions, there is no mention of other typical symptoms
of schizophrenia.
Page 12
98
Correct Option A:
• Based on the information provided, the patient is experiencing symptoms of both depression and
psychosis. The presence of symptoms such as loss of interest, weight loss, insomnia, and sadness
for six months indicates a depressive episode. Additionally, the patient's report of hearing a voice
that ordered her to kill herself suggests the presence of psychotic symptoms.
• According to the Diagnostic and Statistical Manual of Mental Disorders, Fifth Edition (DSM-5), the
diagnosis that best fits this presentation is Major depressive disorder with psychotic features. This
diagnosis is assigned when an individual meets the criteria for a major depressive episode and also
experiences hallucinations, delusions, or other psychotic symptoms.
Incorrect options:
Option B. Schizophrenia: Schizophrenia is a separate psychiatric disorder characterized by persist
ent symptoms such as hallucinations, delusions, disorganized thinking, and negative symptoms. T
he presence of a depressive episode and the lack of sufficient symptoms to meet the criteria for sc
hizophrenia make this diagnosis less likely.
Option C. Schizoaffective disorder: Schizoaffective disorder is characterized by a
combination of symptoms of both schizophrenia and a
mood disorder (such as depression or mania). However, the information provided suggests a
primary diagnosis of major depressive disorder rather than a
combination of schizophrenia and mood disorder symptoms.
Option D. Delusional disorder: Delusional disorder is characterized by the presence of one or more
delusions for at least one month. However, the patient's symptoms of loss of interest, weight loss,
insomnia, and sadness are more consistent with a
depressive episode, making major depressive disorder a more appropriate diagnosis.
Page 13
99
• Corrects: cognitive distortions Maladaptive behaviors
• cognitive distortions
• Maladaptive behaviors
• cognitive distortions
• Maladaptive behaviors
Incorrect choices:
Option B: Cognitive behavioral therapy(CBT) and antidepressants: For mild depression, CBT alone
will be sufficient to treat. For moderate/severe depression there is a
combination of CBT and drugs.
Option C: Mild antidepressants :
It is usually indicated in chronic moderate to severe depression and mot mild depression
Option D: Vagus nerve stimulation: Not indicated for mild depression. Vagus nerve stimulation is w
herein the Left vagus nerve is stimulated by electrodes via a
pulse generator implanted in the chest. It is indicated in resistant depression.
Page 14
100
Solution for Question 17:
Correct option: A
• Love is the most common state because it is a basic human need. Everyone needs to feel loved
and connected to others. Love can be expressed in many different ways, including through physical
touch, words of affirmation, acts of service, gifts, and quality time. Love is a universal emotion that
can be felt by family, friends, romantic partners, and even pets.
• Love affects the brain in a variety of ways, activating different parts of the brain and releasing
particular neurotransmitters and hormones. The reward system of the brain, especially the release
of dopamine, is crucial to the sense of love. Dopamine is linked to pleasure, motivation, and
reinforcement.
• Love is also viewed as a fundamental human feeling from a cultural and sociological standpoint
because of how common it is and how important it is to many different facets of life. It creates and
deepens connections, empathy, and understanding between people, promoting harmony and
cooperation in communities. It has the power to strengthen interpersonal relationships, deepen
emotional intimacy, and provide a sense of security and support.
Incorrect choices:
Option B. Hate: While hate is an intense and negative emotion, it is not considered the most comm
on emotional state. Hate is typically driven by strong feelings of anger, resentment, or animosity to
wards someone or something. While hate exists in society, it is not as prevalent as other emotions
like love or anxiety.
Option C. Anger: Anger is a common emotional state, but it is not considered the most common ov
erall. Anger arises in response to perceived threats, frustrations, or injustices. While anger can be
a natural and healthy response in certain situations, it is not as universally experienced as love.
Option D. Anxiety: Anxiety is a prevalent emotional state, particularly in today's fast-paced and stre
ssful world. However, when compared to love, anxiety is generally considered to be less common.
Anxiety involves feelings of worry, apprehension, and unease, often associated with anticipated fut
ure events or situations.
Page 15
101
Solution for Question 19:
Correct Option D.
• Discontinuation syndrome occurs with abrupt cessation of certain short-acting drugs like
venlafaxine, fluvoxamine, and paroxetine.
• It is not seen with long-acting drugs such as fluoxetine.
• Symptoms include flu-like symptoms (lethargy, weakness, headache), insomnia, nausea,
dizziness, sensory disturbances like paraesthesia, and hyperarousal (rebound anxiety, irritability).
Incorrect Options:
Option A,B and C are examples of long actinge drugs
Page 16
102
Incorrect options:
Option A. To bring calming effect: ECT is not administered to bring a
calming effect but rather to induce a controlled seizure for therapeutic purposes.
Option B. To produce mood elevation: ECT is not directly administered to produce mood elevation
but rather to alleviate severe depressive symptoms.
Option D. To prevent memory loss after ECT: The medications given in ECT are not intended to pr
event memory loss associated with the procedure, but rather to induce anesthesia and muscle rela
xation.
Page 17
103
Option A: Constipation: While constipation can be a side effect of certain medications, it is not a
commonly reported side effect of SSRIs.
Option C: Sialorrhea: Sialorrhea refers to excessive salivation or drooling, and it is not typically ass
ociated with the use of SSRIs.
Option D: Visual disturbance: Visual disturbances are not commonly associated with the use of SS
RIs as a side effect.
Page 18
104
Option A. Major depressive disorder from 2 years: While it is possible to have a chronic course of
major depressive disorder, the specific term "double depression" is not used to describe this conditi
on. Major depressive disorder itself can be a recurrent and chronic condition, but double depressio
n specifically refers to the co-occurrence of MDD and dysthymia.
Option B. Major depression with OCD: Obsessive-Compulsive Disorder (OCD) is a separate psychi
atric disorder characterized by intrusive thoughts (obsessions) and repetitive behaviors or mental a
cts (compulsions). While individuals with OCD may also experience depressive symptoms, the ter
m "double depression" is not used to describe this combination. Double depression specifically inv
olves the coexistence of major depressive disorder and dysthymia.
Option D. Depression with anxiety attack: While anxiety symptoms can often co-occur with depress
ive symptoms, the term "double depression" is not used to describe this combination. Double depr
ession specifically refers to the combination of major depressive disorder and dysthymia.
Page 19
105
• Less ability to concentrate, think, and/or make decisions
• Less energy
• Fatigue
• Feeling hopeless
• Weight and/or appetite changes due to over- or under-eating
• Changes in sleep patterns, such as fitful sleep, inability to sleep, early morning awakening, or
sleeping too much
• Low self-esteem
To diagnose this condition, an adult must have a depressed mood for at least 2
years (or one year in children and adolescents), along with at least 2
of the above symptoms. The symptoms of this illness may look like other mental health conditions.
Option D: Bipolar disorder: Bipolar disorder, formerly known as manic depression, is a mental healt
h condition that can cause extreme mood swings like mania or hypomania as well as depression.
When one develops depression, they may experience feelings of sadness or hopelessness and ma
y lose interest in or enjoyment in most activities. In the case of milder forms of manic depression, t
he mood may change to one that is euphoric, full of energy, or unusually irritable. Rest, energy, mo
vement, judgment, conduct, and the ability to think plainly can be in every way impacted by these e
motional episodes. Emotional episodes may occur periodically or repeatedly throughout the year. B
etween episodes, most individuals will encounter a
few profound side effects. However, some may not. If the person follows a treatment plan, they hav
e some control over their emotional episodes and different side effects, despite the fact that bipolar
issues endure forever. Psychotherapy and medicine are the most well-known medicines for bipola
r turmoil.
Page 20
106
Neurotic, Trauma and Stress Related Disorders
1. What is the most likely diagnosis and drug of choice for a 30-year-old female with sudden-onset
palpitations, sweating, trembling, and fear of losing control, who experiences recurring panic attacks
and anticipates future attacks?
(or)
A 30-year-old female presents to the emergency department with sudden-onset palpitations, sweating,
trembling, and a sense of impending doom. She feels like she is losing control and fears that she might
be going crazy. The episode lasts for about 30 minutes, and she has experienced similar episodes on
multiple occasions over the past month. However, she is asymptomatic in between these episodes.
She reports anticipating future attacks and has been avoiding certain situations due to fear of having a
panic attack. She denies any recent stressful events or medical conditions. Which of the following is the
most likely diagnosis for this patient and the drug of choice?
A. Panic disorder / SSRI
B. Myocardial infarction /Beta blockers
C. Mitral valve prolapse /Beta blockers
D. Pulmonary embolism / Anti-coagulants
E. Migraine /NSAIDS
----------------------------------------
2. A 38-year-old woman has been experiencing intense fear and anxiety in crowded places, open
spaces, and while using public transportation. These episodes are so severe that she often avoids
going out and has become mostly homebound. She has been experiencing these symptoms for the
past 10 months. What is the most likely diagnosis for this patient?
(or)
What is the most likely diagnosis for a patient who experiences intense fear and anxiety in crowded
places, open spaces, and while using public transportation, leading to avoidance of these situations
and becoming mostly homebound for the past 10 months?
A. Generalized anxiety disorder (GAD)
B. Panic disorder
C. Agoraphobia
D. Social anxiety disorder (SAD)
E. Specific phobia
----------------------------------------
3. Jane is a 30-year-old woman who experiences an intense, irrational fear of flying in airplanes. This
fear has persisted for the past 8 months and causes significant distress whenever she has to travel by
air. She avoids flying whenever possible and experiences severe anxiety at the thought of boarding an
airplane. Which of the following treatment options is most appropriate for Jane's condition?
(or)
What is the most appropriate treatment option for a woman, who experiences an intense fear of flying in
airplanes, leading to avoidance and significant distress?
A. Psychotherapy using systematic desensitization
107
B. Pharmacotherapy with selective serotonin reuptake inhibitors (SSRIs)
C. Hypnosis therapy for fear reduction
D. Supportive therapy for emotional reassurance
----------------------------------------
4. What is the most likely diagnosis for a female with complaints of persistent excessive worry and
anxiety for six months, along with symptoms of restlessness, fatigue, difficulty concentrating, and
muscle tension?
(or)
A 35-year-old female presents to the clinic with complaints of excessive worry and anxiety that has
been persistent for the past six months. She reports feeling restless, fatigued, having difficulty
concentrating, and experiencing muscle tension. The anxiety is not restricted to any particular situation
and interferes with her daily functioning. Based on these symptoms, which of the following is the most
likely diagnosis?
A. Generalized Anxiety Disorder (GAD)
B. Panic Disorder
C. Specific Phobia
D. Social Anxiety Disorder (Social Phobia)
----------------------------------------
5. A 28-year-old woman presents with intrusive and distressing thoughts about harming her newborn
baby. She constantly worries that she might act on these thoughts, despite having no desire to harm
her child. She performs repeated rituals, such as checking on the baby's safety, to alleviate her anxiety.
Which disorder is most likely present in this patient?
(or)
What disorder is most likely present in a woman who experiences distressing intrusive thoughts about
harming her newborn baby and performs repeated rituals to alleviate her anxiety?
A. Obsessive-Compulsive Disorder (OCD)
B. Generalized Anxiety Disorder (GAD)
C. Postpartum Depression
D. Panic Disorder
----------------------------------------
6. Which psychological therapy is considered the first-line treatment for Obsessive-Compulsive
Disorder (OCD)?
A. Cognitive Behavioural Therapy (CBT)
B. Psychodynamic Psychotherapy
C. Family Therapy
D. Thought-stopping
----------------------------------------
7. Which neuroanatomical circuit dysfunction is primarily associated with Obsessive-Compulsive
Disorder (OCD)?
Page 2
108
A. Cortico-Striatal-Thalamic-Cortical (CSTC) Circuit
B. Mesolimbic Pathway
C. HPA Axis
D. Basal Ganglia-Thalamocortical Circuit
----------------------------------------
8. A 25-year-old woman is excessively preoccupied with a perceived flaw in her nose. She spends
several hours each day looking in the mirror, trying different angles to assess her nose's appearance.
She avoids social situations and feels extremely distressed about her nose's appearance, despite
reassurances from others. What is the most likely diagnosis?
(or)
What is the most likely diagnosis for a 25-year-old woman excessively preoccupied with a perceived
flaw in her nose, spending hours looking in the mirror, and feeling distressed about her appearance
despite reassurances?
A. Body Dysmorphic Disorder (BDD)
B. Social Anxiety Disorder (SAD)
C. Generalized Anxiety Disorder (GAD)
D. Major Depressive Disorder (MDD)
----------------------------------------
9. A 62-year-old woman has been accumulating stacks of old clothes in her house for several years.
Her living space is cluttered, making it difficult for her to move around. She experiences significant
distress and is unable to discard the clothes due to an intense fear of losing something important.
Which disorder is most likely present in this patient?
(or)
What disorder is most likely present in a woman who accumulates and hoards old newspapers,
magazines, and clothes in her house, experiencing distress and an intense fear of losing something
important?
A. Hoarding Disorder
B. Obsessive-Compulsive Disorder (OCD)
C. Generalized Anxiety Disorder (GAD)
D. Major Depressive Disorder
----------------------------------------
10. A 25-year-old woman presents with a complaint of recurrent hair pulling. She admits to an
irresistible urge to pull out her hair, primarily from her scalp. This behavior has resulted in noticeable
hair loss and has caused significant distress and impairment in her daily functioning. She has made
multiple unsuccessful attempts to stop the behavior. What is the most likely diagnosis?
(or)
What is the most likely diagnosis for a patient who experiences recurrent hair pulling, noticeable hair
loss, distress, and impairment in daily functioning, despite unsuccessful attempts to stop the behavior?
A. Trichotillomania (Hair-Pulling Disorder)
B. Body Dysmorphic Disorder
Page 3
109
C. Obsessive-Compulsive Disorder (OCD)
D. Generalized Anxiety Disorder (GAD)
----------------------------------------
11. What is the most likely diagnosis for a women who exhibits recurrent skin picking, noticeable skin
lesions, and tension before engaging in the behavior, with unsuccessful attempts to stop it?
(or)
A 30-year-old woman presents with a history of recurrent picking of her own skin, particularly on her
face, resulting in noticeable skin lesions. She has been unsuccessful in her attempts to stop this
behavior. Prior to engaging in skin picking, she experiences an increasing sense of tension. Which of
the following is the most likely diagnosis?
A. Trichotillomania
B. Excoriation Disorder
C. Factitious Dermatitis
D. Obsessive-Compulsive Disorder (OCD)
----------------------------------------
12. A 32-year-old female named Sarah recently survived a severe car accident. She was trapped in the
wreckage for several hours before being rescued. Since the accident, she has been experiencing
recurrent distressing dreams about the event, avoiding driving or even being near cars, and constantly
feeling on edge. Which of the following best describes Sarah's symptoms?
(or)
What best describes the symptoms of female, who experienced a severe car accident, and now has
recurrent distressing dreams, avoids cars, and feels constantly on edge?
A. Anterograde amnesia
B. Social withdrawal and isolation.
C. Hypervigilance and avoidance behavior.
D. Excessive sleepiness and fatigue.
----------------------------------------
13. Which of the following statements best describes the role of Eye Movement Desensitization and
Reprocessing (EMDR) in the treatment of Post-Traumatic Stress Disorder (PTSD)?
A. EMDR focuses on providing medication to alleviate PTSD symptoms.
B. EMDR aims to modify maladaptive thought patterns associated with PTSD.
C. EMDR utilizes eye movements to facilitate the processing of traumatic memories.
D. EMDR focuses on improving social support networks for individuals with PTSD.
----------------------------------------
14. A patient experiences significant distress and anxiety following a car accident. He recounts having
flashes of the accident and has been trying to avoid going in cars and prefers buses. However, the
symptoms have only been for a month. Which diagnosis is most likely applicable to this patient?
(or)
Page 4
110
What diagnosis is most likely applicable to a patient who experiences distress, anxiety, re-experiencing
trauma, avoidance of reminders, and negative mood alterations following a traumatic event, with
symptoms expected to resolve within a month?
A. Post-Traumatic Stress Disorder (PTSD)
B. Acute Stress Disorder (ASD)
C. Generalized Anxiety Disorder (GAD)
D. Major Depressive Disorder (MDD)
----------------------------------------
15. A 42-year-old man recently went through a divorce after a long-term marriage. He has been
experiencing irritability, decreased sleep, and a persistent low mood for the past 2 months. Which
diagnosis is most likely applicable to this patient?
(or)
What diagnosis is most likely applicable to a 42-year-old man who experiences irritability, decreased
sleep, and low mood following a recent divorce, with symptoms expected to resolve within six months?
A. Adjustment Disorder
B. Major Depressive Disorder (MDD)
C. Generalized Anxiety Disorder (GAD)
D. Post-Traumatic Stress Disorder (PTSD)
----------------------------------------
Correct Answers
Question Correct Answer
Question 1 1
Question 2 3
Question 3 1
Question 4 1
Question 5 1
Question 6 1
Question 7 1
Question 8 1
Question 9 1
Question 10 1
Question 11 2
Question 12 3
Question 13 3
Question 14 2
Question 15 1
Page 5
111
Solution for Question 1:
Correct Option A: Panic disorder / SSRI
• The patient exhibits typical symptoms of panic attacks, including palpitations, sweating, trembling,
a sense of impending doom, fear of losing control or going crazy.
• The presence of recurrent panic attacks over the past month, along with anticipation of future
attacks and avoidance behaviour, suggests panic disorder.
• The patient also mentions being asymptomatic in between panic attacks, which is consistent with
panic disorder.
• The first-line treatment medication for panic disorder is Selective serotonin reuptake inhibitors
(SSRIs).
• SSRIs, such as sertraline, fluoxetine, or escitalopram, are considered the preferred
pharmacological option for panic disorder due to their effectiveness in reducing the frequency and
severity of panic attacks.
• SSRIs work by increasing serotonin levels in the brain, which helps regulate mood and alleviate
anxiety symptoms.
• They are typically used as long-term maintenance treatment for panic disorder.
Incorrect Options:
Option B - Myocardial infarction / Beta blockers: Myocardial infarction can present with symptoms s
uch as chest pain, shortness of breath, sweating, and fear. However, in this scenario, the patient's
symptoms are more consistent with panic attacks rather than cardiac symptoms. The absence of ri
sk factors and the recurrent nature of the episodes make myocardial infarction less likely.
Option C - Mitral valve prolapse/Beta blockers: Mitral valve prolapse is a condition in which the valv
e between the left atrium and left ventricle doesn't close properly. While it can cause palpitations a
nd anxiety-like symptoms, it is less likely to be the cause in this case since the patient's symptoms
are more characteristic of panic attacks. Additionally, mitral valve prolapse does not typically cause
recurrent episodes of anxiety.
Option D - Pulmonary embolism / Anticoagulants: Pulmonary embolism occurs when a blood clot bl
ocks the pulmonary arteries. It can present with symptoms such as shortness of breath, chest pain,
and anxiety. While it is essential to consider pulmonary embolism in patients with acute-onset sym
ptoms, the absence of respiratory symptoms and the recurrent nature of the attacks make it less lik
ely in this scenario.
Option E - Migraine/NSAIDS: Migraine headaches can sometimes be accompanied by symptoms s
imilar to panic attacks, such as palpitations, sweating, and a
sense of impending doom. However, the absence of a headache and the recurrent nature of the ep
isodes without typical triggers for migraines make it less likely in this case.
Page 6
112
Option A - Generalized anxiety disorder (GAD): GAD is characterized by excessive and persistent
worry and anxiety about various aspects of life, such as work, health, and relationships.
• While individuals with GAD may experience some anxiety in specific situations, it does not
typically involve the specific fear and avoidance of certain places or situations seen in agoraphobia.
Option B - Panic disorder: Panic disorder is characterized by recurrent panic attacks, which are su
dden episodes of intense fear and physical symptoms such as heart palpitations, shortness of brea
th, and trembling.
• Agoraphobia often coexists with panic disorder, but the key feature of panic disorder is the
presence of panic attacks, rather than the specific fear and avoidance of certain places or
situations.
Option D - Social anxiety disorder (SAD): SAD, also known as social phobia, is characterized by int
ense fear and anxiety in social situations, such as speaking in public or interacting with others.
• While individuals with SAD may experience anxiety in certain situations, it does not typically
involve the fear and avoidance of crowded places, open spaces, or public transportation seen in
agoraphobia.
Option E - Specific phobia: Specific phobia involves an intense and irrational fear of a
specific object or situation, such as spiders, heights, or flying.
• While specific phobias can cause significant distress and avoidance behaviours, agoraphobia is
characterized by the fear and avoidance of multiple situations, rather than a specific phobia.
Page 7
113
elp individuals overcome their phobia. It is not the recommended treatment approach for specific p
hobias.
Page 8
114
ptoms such as palpitations, sweating, trembling, and a fear of dying or losing control. The scenario
does not mention symptoms of panic attacks as the primary presentation, making panic disorder le
ss likely in this case.
Page 9
115
Option D - Basal Ganglia-Thalamocortical Circuit is closely connected to the CSTC circuit and is in
volved in motor control and movement. While basal ganglia dysfunction has been implicated in OC
D, it is more accurately described as part of the broader CSTC circuit dysfunction that underlies the
disorder.
Page 10
116
specifically.
Option C - Generalized Anxiety Disorder (GAD): Generalized Anxiety Disorder is characterized by
excessive and uncontrollable worry and anxiety about various aspects of life. While hoarding beha
vior can be associated with anxiety, GAD typically involves excessive worry about everyday life situ
ations, rather than specific compulsive behaviours related to hoarding.
Option D - Major Depressive Disorder: Major Depressive Disorder is a mood disorder characterize
d by persistent feelings of sadness, loss of interest or pleasure, changes in appetite and sleep patt
erns, decreased energy, and difficulty concentrating. While individuals with depression may exhibit
symptoms such as a lack of motivation or difficulty organizing their living space, hoarding behavior
alone is not indicative of Major Depressive Disorder.
Page 11
117
Option D - Obsessive-Compulsive Disorder (OCD): Incorrect. While OCD may involve compulsive
behaviours, the specific focus on recurrent skin picking is more indicative of Excoriation (Skin Picki
ng) Disorder.
Page 12
118
Solution for Question 14:
Correct Option B : Acute Stress Disorder (ASD)
• In the given scenario, the patient is experiencing distress and anxiety following a traumatic event,
with symptoms that are similar to PTSD. However, the key distinguishing factor is the expected
duration of symptoms, which is within a month. This aligns with the diagnosis of Acute Stress
Disorder (ASD), as per the information provided. ASD is characterized by symptoms similar to
PTSD but with a shorter duration, typically resolving within a month.
Incorrect Options:
Option A - Post-Traumatic Stress Disorder (PTSD): The duration of symptoms described in the sce
nario does not meet the criteria for PTSD, which requires symptoms to persist for more than a
month.
Option C - Generalized Anxiety Disorder (GAD): The symptoms described in the scenario are speci
fically related to a traumatic event and not generalized anxiety. GAD is characterized by excessive
and persistent worry across various domains of life.
Option D - Major Depressive Disorder (MDD): The symptoms described in the scenario primarily fo
cus on distress and anxiety related to the traumatic event, rather than meeting the criteria for MDD,
which includes depressive symptoms such as persistent sadness, loss of interest, and changes in
appetite and sleep.
Page 13
119
Page 14
120
Dissociative Disorders, Conversion Disorder,
Somatic Symptoms, & Related Disorders
1. Which of the following statements is true regarding the primary gains observed in dissociative
disorders?
(or)
Which of the following statements is true regarding the primary gains observed in dissociative
disorders?
A. Primary gains involve external benefits and advantages.
B. Primary gains are achieved by keeping internal conflicts outside of awareness.
C. Primary gains are gains derived from personal achievements and accomplishments.
D. Primary gains reduce conscious conflicts by producing external symptoms.
----------------------------------------
2. A 28-year-old woman, Emily, presents to a psychiatrist with an inability to recall important personal
information regarding a traumatic event. Which of the following conditions is most likely causing her
symptoms?
(or)
What is the most likely condition causing a 28-year-old woman, to have an inability to recall important
personal information regarding a traumatic event?
A. Alzheimer's disease, a form of dementia.
B. Dissociative amnesia.
C. Traumatic brain injury.
D. Ordinary forgetting due to aging.
----------------------------------------
3. A 42-year-old man suddenly leaves his home and travels to a different city. He is unable to recall his
past and assumes a new identity. However, he is able to maintain his basic self-care. Which of the
following best describes the condition this man is experiencing?
(or)
What condition is the 42-year-old man experiencing when he suddenly leaves his home, travels to a
different city, and assumes a new identity while being unable to recall his past?
A. Dissociative Fugue
B. Bipolar disorder
C. Generalized anxiety disorder
D. Antisocial personality disorder
----------------------------------------
4. A 35-year-old woman experiences episodes where she enters a state of bewilderment. During these
episodes, she displays marked changes in her state of consciousness and feels a loss of personal
identity. She also finds it difficult to recall certain activities she engaged in during these episodes.
Which of the following best describes her condition?
121
(or)
What condition does the 35-year-old woman with a state of bewilderment, marked changes in
consciousness, and difficulty recalling activities during those episodes likely have?
A. Trance Disorder
B. Dissociative Identity Disorder
C. Generalized Anxiety Disorder
D. Schizophrenia
----------------------------------------
5. Sarah, a 25-year-old woman, has been experiencing recurrent episodes where she feels detached
from herself and the world around her. During these episodes, she often feels like an outside observer
of her own life, as if watching a movie. She also describes a sense of unreality in her surroundings, as if
the world is not real. However, she is fully aware that these experiences are not based in reality. Based
on the information provided, which disorder is most likely affecting Sarah?
(or)
What is the most likely disorder affecting Sarah, a 25-year-old woman who experiences recurrent
episodes of feeling detached from herself and the world around her, as if watching a movie and
experiencing a sense of unreality, while remaining aware that these experiences are not real?
A. Dissociative amnesia
B. Panic disorder
C. Depersonalization-derealization disorder
D. Generalized anxiety disorder
----------------------------------------
6. A patient, Mr. A, seeks help from a mental health professional. He describes experiencing periods of
memory loss and feeling like different personalities take control of his behavior at times. The mental
health professional suspects Dissociative Identity Disorder. Which of the following statements is true
regarding Dissociative Identity Disorder (DID)?
(or)
Which of the following statements is true regarding Dissociative Identity Disorder (DID)?
A. The different personalities in DID are aware of each other's existence.
B. Memory loss is not a common symptom in individuals with DID.
C. Stressful or traumatic events can trigger the switching of personalities in DID.
D. DID is characterized by a single personality with consistent thoughts and behaviours.
----------------------------------------
7. A 25-year-old woman visits a neurology clinic with complaints of sudden loss of sensation in her right
arm. She states that she woke up one morning and found her arm completely numb and unable to
move. She denies any history of trauma or physical injury to the arm. On examination, there are no
signs of muscle weakness or any physical abnormalities. All neurological tests and imaging studies
come back normal. What is the most likely diagnosis for this patient based on the clinical presentation?
(or)
Page 2
122
What is the most likely diagnosis for a 25-year-old woman who presents with sudden loss of sensation
and immobility in her right arm, with normal neurological tests and imaging, and no history of trauma or
physical injury?
A. Conversion Disorder
B. Peripheral Nerve Injury
C. Stroke
D. Multiple Sclerosis
----------------------------------------
8. A 35-year-old patient is brought to the emergency department. The patient appears confused and
disoriented. During the assessment, the following exchange takes place: Doctor: "What is the capital of
France?" Patient: "London." Based on the above scenario, which condition is most likely to be present
in this patient?
(or)
What condition is most likely present in a 35-year-old patient who appears confused and answers
"London" when asked about the capital of France during a medical assessment?
A. Ganser Syndrome
B. Dissociative Identity Disorder
C. Conversion Disorder
D. Brainwashing
----------------------------------------
9. A 25-year-old patient presents with symptoms of memory loss, identity confusion, and sudden
unexplained travels. Upon evaluation, you observe that the patient seems unconcerned and indifferent
towards these symptoms. The patient denies any distress or impairment. Which treatment plan would
be most appropriate for this patient?
(or)
What is the most appropriate treatment plan for a 25-year-old patient with memory loss, identity
confusion, and sudden unexplained travels, who appears unconcerned and indifferent towards these
symptoms?
A. Behavior Therapy
B. Abreaction
C. Psychodynamic Psychotherapy
D. Benzodiazepines
----------------------------------------
10. A 35-year-old patient visits the clinic with complaints of chronic fatigue, multiple somatic symptoms,
and excessive preoccupation with their health. Medical evaluations reveal no underlying medical cause
for the symptoms. The duration of these symptoms has been more than six months. What is the most
likely diagnosis for this patient?
(or)
What is the most likely diagnosis for a 35-year-old patient with chronic fatigue, multiple somatic
symptoms, and excessive health preoccupation lasting more than six months, and no underlying
medical cause found?
Page 3
123
A. Major Depressive Disorder
B. Somatic Symptom Disorder
C. Generalized Anxiety Disorder
D. Panic Disorder
----------------------------------------
11. A 35-year-old individual is constantly preoccupied with having a serious illness. Despite multiple
medical investigations and reassurances from doctors that there is no evidence of any illness, the
preoccupation persists. Which of the following disorders best describes this presentation?
(or)
What disorder best describes a individual who is constantly preoccupied with having a serious illness
despite multiple medical investigations and reassurances from doctors?
A. Somatic Symptom Disorder
B. Conversion Disorder
C. Hypochondriasis
D. Dissociative Identity Disorder
----------------------------------------
12. A patient presents with multiple physical symptoms and provides a history that is inconsistent with
objective findings. The patient seems knowledgeable about medical procedures and frequently seeks
medical attention. What is the most likely diagnosis?
(or)
What is the most likely diagnosis for a patient who presents with multiple physical symptoms, an
inconsistent medical history, and frequently seeks medical attention?
A. Factitious Disorder
B. Conversion Disorder
C. Illness Anxiety Disorder
D. Malingering
----------------------------------------
13. A patient is involved in a legal case and claims to have severe physical impairments that are
inconsistent with objective medical findings. The patient is uncooperative during evaluations and shows
signs of an anti-social personality disorder. What is the most likely explanation for the patient's
behavior?
(or)
What is the most likely explanation for a patient involved in a legal case, showing severe physical
impairments inconsistent with objective findings, uncooperative during evaluations, and displaying
signs of an anti-social personality disorder?
A. Factitious disorder
B. Malingering
C. Conversion disorder
D. Somatic symptom disorder
Page 4
124
----------------------------------------
14. A 35-year-old woman presents to the clinic with a firm belief that she is pregnant. She reports
missed periods, abdominal enlargement, and breast tenderness. On examination, her abdomen
appears distended, and she shows signs of breast engorgement. Laboratory tests confirm negative
pregnancy results. What is the most appropriate treatment approach for this patient?
(or)
What is the most appropriate treatment approach for a woman who firmly believes she is pregnant
despite negative pregnancy test results and displays physical symptoms like missed periods,
abdominal enlargement, and breast tenderness?
A. Prescribe hormonal therapy to induce menstruation and alleviate symptoms.
B. Perform an ultrasound to confirm the absence of pregnancy and reassure the patient.
C. Engage the patient in cognitive behavioural therapy (CBT) to address underlying psychological
factors.
D. Administer anti-anxiety medication to alleviate stress and symptoms.
----------------------------------------
Correct Answers
Question Correct Answer
Question 1 2
Question 2 2
Question 3 1
Question 4 1
Question 5 3
Question 6 3
Question 7 1
Question 8 1
Question 9 1
Question 10 2
Question 11 3
Question 12 1
Question 13 2
Question 14 3
Page 5
125
• Example: Consider a person who experienced severe childhood trauma but has no conscious
memory of the traumatic events. Instead, they might exhibit dissociative amnesia, where memories
related to the trauma are blocked from conscious recall. In this scenario: Primary Gain: By
dissociating from the traumatic memories, the individual avoids conscious distress and anxiety
associated with the traumatic experiences. Function: The primary gain of dissociative amnesia is
the protection from emotional pain or anxiety related to the trauma, allowing the person to function
without being overwhelmed by distressing memories.
• Consider a person who experienced severe childhood trauma but has no conscious memory of
the traumatic events. Instead, they might exhibit dissociative amnesia, where memories related to
the trauma are blocked from conscious recall. In this scenario:
• Primary Gain: By dissociating from the traumatic memories, the individual avoids conscious
distress and anxiety associated with the traumatic experiences.
• Function: The primary gain of dissociative amnesia is the protection from emotional pain or
anxiety related to the trauma, allowing the person to function without being overwhelmed by
distressing memories.
• Consider a person who experienced severe childhood trauma but has no conscious memory of
the traumatic events. Instead, they might exhibit dissociative amnesia, where memories related to
the trauma are blocked from conscious recall. In this scenario:
• Primary Gain: By dissociating from the traumatic memories, the individual avoids conscious
distress and anxiety associated with the traumatic experiences.
• Function: The primary gain of dissociative amnesia is the protection from emotional pain or
anxiety related to the trauma, allowing the person to function without being overwhelmed by
distressing memories.
Incorrect Options:
Option A - Primary gains involve external benefits and advantages: Primary gains involve internal c
onflict reduction rather than external benefits.
Option C - Primary gains are gains derived from personal achievements and accomplishments: Pri
mary gains are not derived from personal achievements.
Option D - Primary gains reduce conscious conflicts by producing external symptoms: Primary gain
s involve reducing internal conflicts rather than producing external symptoms.
Page 6
126
Option C - Traumatic brain injury: While it can cause memory problems, dissociative amnesia is sp
ecifically associated with psychological trauma rather than physical brain injury.
Option D - Ordinary forgetting due to aging Dissociative amnesia is different from normal memory d
ecline associated with aging. Dissociative amnesia involves a
specific and selective loss of memory related to a traumatic event.
Page 7
127
Solution for Question 5:
Correct Option C : Depersonalization-derealization disorder
• This disorder is characterized by recurrent experiences of depersonalization (feeling detached
from oneself) and/or derealization (feeling detached from the world). Individuals with this disorder
maintain intact reality testing, meaning they are aware that their experiences are not based in
reality.
• Depersonalization/ Derealisation is also a symptom of PTSD and substance abuse, psychotic
disorders.
Incorrect Options:
Option A - Dissociative amnesia: Dissociative amnesia involves the inability to recall important pers
onal information, typically related to a traumatic event. It is characterized by memory loss rather th
an feelings of detachment from self or the world.
Option B - Panic disorder: Panic disorder is characterized by recurrent panic attacks, which are su
dden and intense episodes of fear or discomfort. While panic attacks can sometimes involve feelin
gs of unreality or detachment, the key feature of panic disorder is the presence of panic attacks.
Option D - Generalized anxiety disorder: Generalized anxiety disorder involves excessive and persi
stent worry and anxiety about various aspects of life. While individuals with this disorder may exper
ience symptoms such as restlessness or irritability, it does not typically involve the detachment fro
m self or the world seen in depersonalization-derealization disorder.
Page 8
128
• Conversion Disorder is a condition characterized by the presence of neurological symptoms that
cannot be explained by a physical cause. The symptoms often mimic those of a neurological
disorder but lack evidence of structural or physiological abnormalities. It is believed that
psychological factors, such as stress or trauma, contribute to the development of these symptoms.
Incorrect Options:
Option B - Peripheral Nerve Injury: Peripheral nerve injury refers to damage or trauma to the nerve
s located outside the brain and spinal cord. It can result in various symptoms, including sensory los
s, weakness, or paralysis in the affected area. In this scenario, the absence of physical trauma or i
njury suggests that peripheral nerve injury is less likely as the cause of the patient's symptoms.
Option C - Stroke: A stroke occurs when the blood supply to a part of the brain is disrupted, leading
to neurological symptoms. Common symptoms include sudden weakness or paralysis on one side
of the body, speech difficulties, and loss of sensation. However, in this case, the absence of any o
ther neurological signs and the normal findings on examination make stroke an unlikely diagnosis.
Option D - Multiple Sclerosis: Multiple Sclerosis (MS) is a chronic autoimmune disease affecting th
e central nervous system. It involves inflammation and damage to the protective covering of nerve f
ibres, leading to various neurological symptoms. However, the sudden onset of symptoms limited t
o one limb without any other characteristic features of MS makes it an unlikely diagnosis in this sce
nario.
Incorrect Options:
Option B - Dissociative Identity Disorder: Dissociative Identity Disorder is characterized by the pres
ence of two or more distinct personalities within one individual, with only one evident at a
time. This disorder does not explain the patient's approximate answers in this scenario.
Option C - Conversion Disorder: Conversion Disorder involves the presence of neurological sympt
oms that are suggestive of a deficit in motor or sensory functions, without evidence of a physical di
sorder. The scenario does not provide information about neurological symptoms or deficits.
Option D - Brainwashing: Brainwashing refers to the process of intense coercive persuasion that m
ay result in identity disturbance. It is typically associated with situations such as torture, terrorist ho
stages, and long-term political imprisonment. The scenario does not provide evidence of coercive p
ersuasion or prolonged changes in personality, beliefs, and behaviours.
Page 9
129
• The patient in the given scenario is likely experiencing symptoms of memory loss, identity
confusion, and unexplained travels, suggesting a possible diagnosis of Dissociative Fugue or
Dissociative Amnesia with identity confusion. The presence of "la belle indifference," or an
unconcerned attitude towards the symptoms, is also observed.
• This option focuses on addressing current symptoms and minimizing secondary and tertiary gains
associated with the illness. It helps the patient move away from the symptom phase and
encourages the assumption of a healthier role.
Incorrect Options:
Option B - Abreaction: This option involves bringing unconscious memories and emotions to the pa
tient's awareness. While it can be beneficial in certain cases, the patient's indifference and lack of
distress make it less suitable in this scenario.
Option C - Psychodynamic Psychotherapy: This option explores the patient's unconscious conflicts
and underlying psychological processes. However, since the patient appears unconcerned about t
heir symptoms, other treatment approaches may be more appropriate.
Option D - Benzodiazepines: While benzodiazepines can be used for short-term anxiety relief, they
are not the primary treatment for dissociative disorders. In this case, the patient's lack of distress a
nd impairment suggests that pharmacotherapy may not be the first-line treatment.
Page 10
130
• This disorder fits the given scenario as the individual is constantly preoccupied with having a
serious illness, despite medical reassurances. The absence of significant somatic symptoms and
the persistence of the preoccupation for more than six months align with the features of Illness
Anxiety Disorder.
Incorrect Options:
Options A - Somatic Symptom Disorder: Somatic Symptom Disorder involves preoccupation with s
omatic symptoms, but in this case, the somatic symptoms are either absent or mild, and the main f
ocus is on the fear of having a serious illness, which is more indicative of Illness Anxiety Disorder.
Option B - Conversion Disorder: Conversion Disorder is characterized by neurological symptoms th
at are inconsistent with any known medical condition. It does not involve preoccupation with having
a serious illness.
Option D - Dissociative Identity Disorder: Dissociative Identity Disorder involves the presence of m
ultiple distinct identities or personalities within an individual, which is not evident in the given scena
rio.
Page 11
131
Incorrect Options:
Option A - Factitious disorder: Factitious disorder involves the intentional production of physical or
psychological symptoms to assume the sick role and receive medical attention. However, in this ca
se, the patient's motivation is related to a
legal case rather than seeking medical attention, making factitious disorder less likely.
Option C - Conversion disorder: Conversion disorder is characterized by the presence of neurologi
cal symptoms that are not explained by any underlying medical condition. While conversion disord
er may involve the manifestation of physical symptoms, it is typically not associated with deliberate
deception or the presence of an antisocial personality disorder.
Option D - Somatic symptom disorder: Somatic symptom disorder involves the excessive preoccup
ation and distress with one's physical symptoms, leading to impairment in daily functioning. Althou
gh this disorder may involve the exaggeration or focus on physical symptoms, malingering is a mor
e appropriate explanation in this case due to the legal context and presence of an antisocial perso
nality disorder.
Page 12
132
Previous Year Questions
1. What is the term used to describe the phenomenon where a patient in the ward responds to the
question "what is your name?" by repeating the same sentence?
A. Coprolalia
B. Echopraxia
C. Copropraxia
D. Echolalia
----------------------------------------
2. What is the probable diagnosis for a 25-year-old woman who presents with symptoms such as
anxiety, palpitations, sweating, breathlessness, chest pain, and a sense of impending doom? The
patient mentions having encountered approximately 5-6 comparable episodes in the last half-year, with
each episode persisting for around 20-30 minutes.
A. Depression
B. Panic disorder
C. Generalized anxiety disorder
D. Phobia
----------------------------------------
3. What is the correct statement regarding the condition in a 14-year-old girl who has experienced
sudden-onset blindness for the past 4 hours, but does not express concern about it, yet expresses
regret over not spending enough time with her recently deceased mother?
A. In children, equally among males and females
B. In adults, equally among males and females
C. In adults, it occurs more in males than in females
D. In children, it occurs more in females than in males
----------------------------------------
4. SCOFF criteria is used as a screening tool for which of the following disorder?
A. Eating disorder
B. Sexual disorder
C. Alcohol use disorder
D. Schizophrenia
----------------------------------------
5. What is the probable medical condition for a 25-year-old male who experienced a road traffic
accident and is now experiencing anxiety about driving and going to the site of the incident for a period
of 6 weeks? He is also having recurring memories of the accident and struggles with attending work.
A. Acute stress disorder
B. Post-traumatic stress disorder
C. Panic disorder
133
D. Major depressive disorder
----------------------------------------
6. What is the most probable diagnosis for a first-year MBBS student who presents at the OPD with
intermittent episodes of intense sweating and a sensation of imminent danger, commonly experienced
before her examinations?
A. Panic disorder
B. Conversion disorder
C. Social anxiety disorder
D. Generalized anxiety disorder
----------------------------------------
7. What is the probable diagnosis for a 20-year-old female who exhibits abrupt palpitations, sweating,
breathlessness, and a sensation of being choked? She had contracted COVID-19 a year ago and is
worried about her life due to her prior positive status. All physical examinations, ECG, and laboratory
tests have yielded no significant findings. This episode, lasting approximately 20 minutes, is the first
occurrence of its kind.
A. Post-traumatic stress disorder
B. Bipolar disorder
C. Depression
D. Panic attack
----------------------------------------
8. Match the following: A. Depression B. Suicide C. Burnout D. Overworking The feeling goes away as
soon as you leave the hospital State of emotional and physical exhaustion caused by prolonged stress,
relieved after relaxation Difficult to diagnose due to the shell of physician’s pride Women physicians,
substance abuse, relationship troubles
u leave the hospitalState of emotional and physical exhaustion caused by prolonged stress, relieved after relaxationDifficult to diagnose due
Page 2
134
A. DBT
B. Exposure and response prevention
C. Systematic desensitization
D. CBT
----------------------------------------
11. According to ICD 11, which of the options below is not categorized as Obsessive-Compulsive
Disorder (OCD)?
A. Trichotillomania
B. Body dysmorphic disorder
C. Hypochondriac disorder
D. PTSD
----------------------------------------
12. What is the fear of heights known as?
A. Acarophobia
B. Algophobia
C. Agoraphobia
D. Acrophobia
----------------------------------------
13. Which of the following symptoms is least likely to be associated with a panic attack?
A. Palpitations
B. Fear of dying
C. Chest discomfort
D. Suicide attempt
----------------------------------------
14. What deficiency is responsible for cognitive impairment in the elderly?
A. Vitamin C
B. Vitamin B12
C. Vitamin B6
D. Vitamin B1
----------------------------------------
15. Which of the following conditions does the 40-year-old woman, who was involved in a car accident
2 months ago, experience when she wakes up screaming at night due to recurring nightmares of the
same incident?
A. Acute stress reaction
B. Adjustment disorder
C. Mania
Page 3
135
D. Post- traumatic stress disorder
----------------------------------------
16. How can we describe the emotional release and discharge that occurs when a person consciously
relieves a repressed painful experience?
A. Catharsis
B. Abreaction
C. Venting out
D. Guided relaxation
----------------------------------------
17. Which of the subsequent options is not classified as an obsessive-compulsive disorder (OCD)
related ailment?
A. Hair pulling disorder
B. Temper tantrums
C. Hoarding disorder
D. Skin picking
----------------------------------------
18. What is the probable diagnosis for a 24-year-old woman who experienced sudden chest pain and
palpitations lasting approximately 20 minutes, with a history of three similar episodes in the past,
despite normal investigation results?
A. Acute psychosis
B. Panic attack
C. Post-traumatic stress disorder
D. Mania
----------------------------------------
19. All are stages of grief or loss, except:
A. Denial
B. Anger
C. Bargaining
D. Agitation
----------------------------------------
20. In the psychiatry OPD, a male patient aged 40 presents with a concern regarding persistent
thoughts. Specifically, he experiences a constant belief that his hands are dirty, despite them being
clean. Consequently, he feels uneasy and is compelled to repeatedly wash his hands. The condition
can be classified as a disorder involving thought processes known as ______.
A. Flow
B. Form
C. Possession
D. Content
Page 4
136
----------------------------------------
21. What could be the likely diagnosis for a male patient who recently lost his job and is experiencing
persistent irritability, sadness, and worry about the future? He also displays irritability towards his family
members. Although he is able to temporarily enjoy watching movies with friends, upon returning home,
he experiences the same symptoms.
A. Generalised anxiety disorder
B. Adjustment disorder
C. Mixed anxiety with depression
D. Moderate depression
----------------------------------------
22. What is the likely diagnosis for a 10-year-old child experiencing selective mutism?
A. Childhood depression
B. Childhood psychosis
C. Childhood anxiety disorder
D. Hyperkinetic disorder
----------------------------------------
23. Among the following, what is the most common symptom of adult obsessive-compulsive disorder?
A. Aggressive symptoms
B. Need for symmetry
C. Pathological doubt
D. Sexual symptoms
----------------------------------------
24. What is the psychiatric disorder most frequently observed in patients with medical or surgical
conditions?
A. Depression
B. Schizophrenia
C. Delirium
D. Dementia
----------------------------------------
25. A concerned grandmother brings an 18-year-old boy to you, reporting that he had multiple episodes
of body jerking the day before his final exam. There were no signs of confusion or tongue biting after
the episodes, and his medical tests came back normal. Which of the following conditions should be
taken into consideration?
A. Malingering
B. Factitious disorder
C. Somatisation syndrome
D. Hypochondriasis
----------------------------------------
Page 5
137
26. What is the diagnosis of a 40-year-old man who has been experiencing excessive and
uncontrollable worry about daily life activities, finances, and his health for the past two years? He also
presents with restlessness, fatigue, disturbed sleep, and waking up once due to a fear that his family
was involved in an accident.
A. Depression
B. Illness anxiety disorder
C. Somatoform disorder
D. Generalized anxiety disorder
----------------------------------------
Correct Answers
Question Correct Answer
Question 1 4
Question 2 2
Question 3 4
Question 4 1
Question 5 2
Question 6 1
Question 7 4
Question 8 3
Question 9 3
Question 10 2
Question 11 4
Question 12 4
Question 13 4
Question 14 2
Question 15 4
Question 16 2
Question 17 2
Question 18 2
Question 19 4
Question 20 3
Question 21 2
Question 22 3
Question 23 3
Question 24 3
Page 6
138
Question 25 1
Question 26 4
Page 7
139
specific object or situation, but that's not the case here.
Page 8
140
• Would you describe food as dominating your life?
Incorrect Options:
Option B: Sexual disorder: Sexual disorders encompass a range of conditions that affect a person'
s sexual thoughts, desires, or behaviors. The SCOFF criteria are not designed to assess or evaluat
e these types of disorders.
Option C: Alcohol use disorder: Alcohol use disorder refers to a problematic pattern of alcohol cons
umption leading to significant impairment or distress. While screening tools do exist for alcohol use
disorder, including the AUDIT (Alcohol Use Disorders Identification Test) and the CAGE questionn
aire but the SCOFF criteria are not among them.
Option D: Schizophrenia: Schizophrenia is a complex mental disorder characterized by symptoms
such as hallucinations, delusions, disorganized thinking, and impaired social functioning. The SCO
FF criteria are specific to screening for eating disorders and do not assess symptoms related to sc
hizophrenia.
Page 9
141
• In the given scenario, the student experiences intermittent episodes of a feeling of impending
doom accompanied by intense perspiration, particularly prior to exams. These symptoms are
consistent with panic attacks, which commonly occur in situations that provoke anxiety or fear, such
as exams or stressful events.
Incorrect options:
Option B. Conversion disorder: Conversion disorder involves the presence of neurological sympto
ms that cannot be explained by any underlying medical condition. The symptoms are thought to be
related to psychological stress or conflict. While anxiety and stress can contribute to various physi
cal symptoms, the specific symptoms described in the scenario are more indicative of panic attack
s, suggesting panic disorder rather than conversion disorder.
Option C. Social anxiety disorder: Social anxiety disorder is characterized by an intense fear of soc
ial situations, leading to avoidance or distress in social settings. The symptoms typically revolve ar
ound social interactions, while the described symptoms in the scenario are more consistent with pa
nic attacks occurring in the context of exams.
Option D. Generalized anxiety disorder: Generalized anxiety disorder involves excessive worry and
anxiety about various aspects of life, persisting for at least six months. While exam-related stress
and worry may be present in generalized anxiety disorder, the intense physical symptoms of panic
attacks described in the scenario are not specific to generalized anxiety disorder.
Page 10
142
d the nature of symptoms.
Option B. Suicide is more among women physicians, substance abuse and relationship troubles. O
ther risk factors include previous suicide attempts, unemployed people, chronic illness, and sexual
abuse.
Option D. Overworking is a feeling that goes away as soon as you leave the hospital. It can cause
physical and mental health problems, bad relationships, reduced productivity, and poor work-life ba
lance.
So , the correct option is C.
Page 11
143
• b. Systematic desensitization: Desensitization, thought-stopping, flooding, and aversive
conditioning are also used to treat OCD. However, exposure and response prevention is the best
way of managing this condition.
• d. CBT: Exposure and response prevention is a kind of CBT that helps reduce anxiety and
distress associated with repeated exposure to thoughts or objects triggering the condition.
Page 12
144
• But few studies report that patients experience fear of dying but it could be an independent risk
factor for subsequent suicide attempts, especially for those patients with depressive disorders.
• Panic attacks are unexpected i.e. not restricted to any particular situation usually lasting for 20-30
minutes and rarely last > 1 hour
Incorrect choices:
Option A,B,C:
• Palpitations, fear of dying, and chest discomfort are experienced by patients with a panic attack
along with sweating, tremors, and shortness of breath.
• Patients experiencing panic attacks will be having the fear of dying, they might be overwhelmed,
and frustrated with the inability to control the symptoms of panic attacks. This might or less likely
lead to suicide attempts.
Page 13
145
se symptoms usually resolve within a short period, typically within a few days or weeks. In the give
n scenario, the symptoms have persisted for two months, suggesting a
more chronic condition like PTSD.
Option B: Adjustment disorder- Adjustment disorder is a psychological reaction to a
stressful life event, such as a major life change or trauma. It is characterized by emotional and beh
avioral symptoms that occur within three months of the stressful event. While the lady's symptoms
may initially resemble adjustment disorder, the persistent and recurring nature of the nightmares a
nd distress point more towards PTSD.
Option C: Mania- Mania is a symptom associated with bipolar disorder and is characterized by an e
levated or irritable mood, increased energy, racing thoughts, and impulsive behavior. The symptom
s described in the scenario, such as nightmares and distress related to the car accident, are not co
nsistent with mania.
Page 14
146
Incorrect Options:
Option A: Hair pulling disorder (trichotillomania)- Trichotillomania, also known as hair pulling disord
er, is a disorder characterized by recurrent pulling out of one's hair, resulting in noticeable hair loss
. It is considered an OCD-related disorder because it shares similarities with obsessive-compulsive
disorder (OCD) in terms of repetitive behaviors and urges. Individuals with trichotillomania may ex
perience a strong urge to pull out their hair and may feel a
sense of relief or satisfaction after doing so.
Option C: Hoarding disorder- Hoarding disorder is a condition characterized by persistent difficulty i
n parting with or discarding possessions, regardless of their value. It involves excessive accumulati
on of items, leading to clutter and significant distress or impairment in daily functioning. Hoarding di
sorder is considered an OCD-related disorder because it shares similarities with OCD in terms of c
ompulsive behaviors and difficulties with decision-making and organization.
Option D: Skin picking (excoriation disorder)- Excoriation disorder, also known as skin picking disor
der, is a disorder characterized by recurrent picking at one's own skin, resulting in skin lesions. It is
considered an OCD-related disorder because it involves repetitive behaviors similar to those seen
in OCD. Individuals with excoriation disorder may engage in skin picking to relieve tension, experie
nce a sense of gratification, or attempt to improve the appearance of their skin.
Page 15
147
Incorrect options
Option A. Denial -
The initial stage where individuals may refuse to believe or accept the reality of the situation.
Option B. Anger - The stage where individuals may experience feelings of anger and frustration.
Option C. Bargaining -
The stage where individuals may attempt to negotiate or make deals to change the outcome.
Page 16
148
onsistent with adjustment disorder, which is a distinct diagnostic category related to a
specific stressor.
Page 17
149
ymptoms of adult OCD typically involve doubt, intrusive thoughts, and repetitive behaviors rather th
an primarily focusing on sexual content.
Page 18
150
Option B: Factitious disorder: A condition known as a factitious disorder is one in which a person d
eliberately causes or feigns symptoms in order to assume the sick role. Dissimilar to malingering, i
ndividuals with factitious turmoil have no outer inspiration for their way of behaving. They might do i
t for the consideration, the compassion, or the sensation of being really focused.
Option C: Somatization syndrome: People with somatization syndrome experience a
wide range of physical symptoms that are unrelated to any medical condition. A person with somati
c symptom disorder (SSD) experiences extreme, exaggerated anxiety regarding physical symptom
s. The person experiences symptoms-related thoughts, feelings, and behaviors that are so intense
that they believe they are unable to perform some of their day-to-day activities.
Option D: Hypochondriasis: An illness anxiety disorder, also known as hypochondriasis or health a
nxiety, is excessive worry about getting or staying ill. There may be no physical symptoms. Or on t
he other hand, one might accept that typical body sensations or minor side effects are indications o
f extreme disease, although an intensive clinical test doesn't uncover a
serious ailment. One might worry a
lot that body sensations like muscle twitching or fatigue are related to a particularly serious illness.
Rather than the physical symptom itself, excessive anxiety causes severe distress that can disrupt
their life.
Page 19
151
Substance Use and Addictive Disorders
1. Which of the following neurotransmitters is not involved in addiction?
A. Opioid (like endorphins)
B. Catecholamine (Mainly Dopamine)
C. Gamma-aminobutyric acid (GABA)
D. Serotonin
----------------------------------------
2. John consumes a significant amount of alcohol during a social gathering. As time passes, his blood
alcohol level starts to fluctuate. During which phase is John likely to experience more pronounced
intoxicating effects?
A. During the initial phase when his blood alcohol level is increasing.
B. During the later phase when his blood alcohol level is decreasing.
C. John will experience the same level of intoxication throughout the entire process.
D. The Mellanby Effect does not apply to alcohol intoxication.
----------------------------------------
3. What is reverse tolerance?
A. Increased sensitivity to the effects of a drug with repeated use.
B. Decreased sensitivity to the effects of a drug with repeated use.
C. Unchanged sensitivity to the effects of a drug with repeated use.
D. A phenomenon observed only with cocaine consumption.
----------------------------------------
4. What is the principle behind a Breathalyzer?
A. Alcohol in the breath is directly proportional to the alcohol content in the bottle.
B. Alcohol in the breath is inversely proportional to the alcohol content in the blood.
C. Alcohol in the breath is unrelated to the alcohol content in the blood.
D. Alcohol in the breath is in equilibrium with the alcohol content in the blood passing through the lungs.
----------------------------------------
5. At what BAC level is an individual likely to experience incoordination, judgment errors, and mood
lability?
A. 20-30 mg/dl
B. 30-80 mg/dl
C. 80-200 mg/dl
D. 200-300 mg/dl
----------------------------------------
6. What is the most likely diagnosis for a chronic heavy drinker who presents to the emergency room
with confusion, disorientation, visual hallucinations, increased heart rate, and elevated blood pressure
152
48 hours after stopping alcohol consumption?
(or)
Mr. Johnson, a chronic heavy drinker, presents to the emergency room with confusion, disorientation,
and visual hallucinations 48 hours after stopping alcohol consumption. Vital signs reveal increased
heart rate and elevated blood pressure. What is the most likely diagnosis?
A. Alcohol intoxication
B. Hepatic Encephalopathy
C. Delirium Tremens
D. Alcohol-related psychosis
----------------------------------------
7. Mrs. Thompson, a 62-year-old woman with a history of chronic alcoholism, is brought to the
emergency department by her family. They report that she has been experiencing confusion, abnormal
eye movements, and difficulty walking for the past few days. On examination, you note global
confusion, horizontal nystagmus, and ataxia. Laboratory tests reveal thiamine deficiency. What is the
most likely diagnosis?
(or)
What is the most likely diagnosis for Mrs. Thompson, a 62-year-old chronic alcoholic presenting with
confusion, abnormal eye movements, and difficulty walking, and is found to have thiamine deficiency
on laboratory tests?
A. Alcohol intoxication
B. Wernicke's Encephalopathy
C. Alzheimer's disease
D. Multiple sclerosis
----------------------------------------
8. A 55-year-old male patient presents with a history of memory impairment and difficulty recalling
recent events. He often fabricates false stories to fill memory gaps. On examination, he displays signs
of anterograde and retrograde amnesia. Which of the following is the most likely diagnosis?
(or)
What is the most likely diagnosis for a 55-year-old male patient with memory impairment, difficulty
recalling recent events, and fabrication of false stories to fill memory gaps, and displaying signs of
anterograde and retrograde amnesia?
A. Alzheimer's Disease
B. Korsakoff Syndrome
C. Parkinson's Disease
D. Vascular Dementia
----------------------------------------
9. A 45-year-old male presents with a history of seizures, unsteady gait, visual hallucinations, and
slurred speech. On examination, he displays signs of ataxia and intellectual deterioration. MRI reveals
demyelination of the corpus callosum, optic tracts, and cerebellar peduncles. Which of the following is
the most likely diagnosis?
Page 2
153
(or)
What is the most likely diagnosis for a 45-year-old male patient with seizures, ataxia, visual
hallucinations, slurred speech, and MRI findings of demyelination in the corpus callosum, optic tracts,
and cerebellar peduncles?
A. Multiple Sclerosis
B. Marchiafava-Bignami Disease
C. Wilson's Disease
D. Huntington's Disease
----------------------------------------
10. Joginder Singh, a 40-year-old man, has been experiencing ease in falling asleep and waking up
frequently during the night for the past few weeks. He visits his doctor to discuss his sleep problems.
During the evaluation, Joginder Singh reveals that he usually has a glass of wine before bedtime,
thinking it helps him relax and fall asleep faster. However, he has noticed that despite this routine, his
sleep has been disrupted and he wakes up feeling tired in the morning. Which of the following best
describes the effects of alcohol on Joginder Singh's sleep?
(or)
Which of the following best describes the effects of alcohol on sleep?
A. Increases REM sleep and deep sleep
B. Reduces sleep latency
C. Decreases sleep fragmentation
D. Improves overall sleep quality
----------------------------------------
11. The CAGE questionnaire is a screening tool for alcohol disorders. Which of the following options
incorrectly matches the full form of the CAGE acronym?
(or)
Which of the following options incorrectly matches the full form of the CAGE acronym?
A. "C" stands for "Cut down on Drinking."
B. "A" stands for "Annoyance when asked about your drinking."
C. "G" stands for "Guilty about your drinking."
D. "E" stands for "Excessive alcohol consumption."
----------------------------------------
12. Which laboratory parameter is highly specific for detecting alcohol use?
A. Carbohydrate Deficient Transferrin (CDT)
B. Gamma-Glutamine Transferase (GGT)
C. Mean Corpuscular Volume (MCV)
D. Uric Acid
----------------------------------------
Page 3
154
13. What is the treatment of choice for a 45-year-old patient with alcohol withdrawal symptoms,
including tremors, anxiety, and restlessness, and a history of heavy alcohol use?
(or)
A 45-year-old patient presents with alcohol withdrawal symptoms, including tremors, anxiety, and
restlessness. The patient has a history of heavy alcohol use. Which of the following is the treatment of
choice for this patient?
A. Selective serotonin reuptake inhibitors (SSRIs)
B. Benzodiazepines (BZDs)
C. Antipsychotics (such as Olanzapine)
D. Carbamazepine
----------------------------------------
14. A 42-year-old male, Mr. Smith, seeks treatment for alcohol dependence. After assessing his
motivation to stay abstinent, the healthcare provider decides to prescribe Disulfiram as a deterrent
agent. Before initiating Disulfiram, the provider educates Mr. Smith about its mechanism of action and
potential side effects. Which of the following statements regarding Disulfiram is correct?
(or)
Which of the following statements regarding Disulfiram is correct?
A. Disulfiram inhibits the conversion of acetic acid to acetaldehyde.
B. Disulfiram is used to treat alcohol withdrawal symptoms.
C. Disulfiram should be initiated immediately after alcohol intake.
D. Disulfiram is contraindicated in patients without any motivation to quit alcohol.
----------------------------------------
15. A 28-year-old male is brought to the emergency department by his friends. They report finding him
unconscious at home. On examination, the patient's breathing is shallow, and his pupils are pinpoint.
He is unresponsive and exhibits signs of hypothermia and hypotension. What is the most likely
diagnosis, and what is the appropriate treatment for this condition?
(or)
What is the most likely diagnosis and appropriate treatment for a 28-year-old male brought to the
emergency department with unconsciousness, shallow breathing, pinpoint pupils, hypothermia, and
hypotension?
A. Alcohol intoxication; administer IV lorazepam
B. Opioid intoxication; administer IV naloxone
C. Benzodiazepine overdose; administer IV flumazenil
D. Stimulant overdose; administer IV diazepam
----------------------------------------
16. A 32-year-old male presents with complaints of severe muscle cramps, bone aches, abdominal
cramps, diarrhoea, lacrimation, rhinorrhoea, sweating, piloerection (gooseflesh), excessive yawning,
dilated pupils, insomnia, hypertension, and restlessness. He reports a history of prolonged opioid use
and recently decided to stop using opioids. On examination, his vital signs are stable. Which of the
following is the most appropriate treatment option?
Page 4
155
(or)
What is the most appropriate treatment for a 32-year-old male with symptoms of severe muscle and
bone cramps, abdominal cramps, lacrimation, rhinorrhoea, sweating, piloerection, excessive yawning,
dilated pupils, insomnia, hypertension, and restlessness after stopping prolonged opioid use?
A. IV Naloxone
B. Methadone
C. Benzodiazepine
D. Bupropion
----------------------------------------
17. Which of the following statements about amphetamine intoxication is correct?
A. It primarily affects the serotonin receptors in the brain.
B. Symptoms may include pupillary constriction and bradycardia.
C. It commonly causes hallucinations similar to LSD.
D. Hallmark symptoms include euphoria, dilated pupils, and tachycardia.
----------------------------------------
18. What is the drug of choice for the treatment of nicotine addiction, and what is its mechanism of
action?
(or)
A 45-year-old male presents with symptoms of tobacco intoxication, including increased heart rate,
sweating, and euphoria. He has a history of smoking for the past 20 years. You suspect nicotine
overdose. What is the drug of choice for the treatment of nicotine addiction in this patient, and what is
its mechanism of action?
A. Varenicline - partial agonist at α4β2 nicotinic acetylcholine receptors
B. Bupropion - Norepinephrine Dopamine Reuptake Inhibitor (NDRI)
C. Nicotine Nasal Spray - acts as a nicotine replacement therapy
D. Nicotine Patch - slowly releases nicotine into the bloodstream
----------------------------------------
19. Match the following types of cannabis with the part from which they are derived: 1. Bhang A. Dried
leaves 2. Ganja B. Flowering stems 3. Hashish C. Dried resin 4. Hash Oil D. Oil-based extract
1. Bhang A. Dried leaves
2. Ganja B. Flowering stems
3. Hashish C. Dried resin
4. Hash Oil D. Oil-based extract
Page 5
156
20. A 21-year-old individual presents to the emergency department after attending a party. On
examination, the person appears euphoric, with redness of the conjunctiva, increased appetite, and
impaired motor coordination. The person also experiences a sense of time distortion and complains of
dry mouth. The most likely diagnosis for this individual is:
(or)
What is the most likely diagnosis for a 21-year-old individual with symptoms of euphoria, redness of the
conjunctiva, increased appetite, impaired motor coordination, time distortion, and dry mouth after
attending a party?
A. Cannabis Intoxication
B. Alcohol Intoxication
C. Cocaine Intoxication
D. Amphetamine Intoxication
----------------------------------------
21. Which of the following conditions is characterized by a person's unwillingness to persist in tasks,
apathy, lethargy, and possible weight gain after consuming marijuana?
A. Cannabis Intoxication
B. Cannabis Withdrawal
C. Running Amok
D. Amotivational Syndrome
----------------------------------------
22. Which of the following statements about PCP and Ketamine intoxication is correct?
(or)
Phencyclidine (PCP) and Ketamine belong to the class of drugs known as dissociative anaesthetics.
Which of the following statements about PCP and Ketamine intoxication is correct?
A. They have stimulant-like effects and increase energy levels.
B. They primarily affect the serotonin receptors in the brain.
C. They induce feelings of relaxation and sedation.
D. They mimic symptoms similar to Schizophrenia, including hallucinations and disorganized thoughts.
----------------------------------------
23. A 27-year-old individual is brought to the emergency department with various neurological and
perceptual symptoms. The patient reports recent substance use at a party. Further evaluation reveals
dilated pupils, tactile hallucinations, and a distorted sense of time. The patient also experiences
heightened emotions and an altered perception of sensory stimuli. The symptoms persist for several
hours and gradually subside. Which of the following substances is most likely responsible for the
described clinical presentation?
(or)
What substance is most likely responsible for a 27-year-old individual's neurological and perceptual
symptoms, including dilated pupils, tactile hallucinations, altered time perception, heightened emotions,
and altered sensory stimuli perception, after recent use at a party?
A. Cocaine
Page 6
157
B. Alcohol
C. Phencyclidine (PCP)
D. LSD (Lysergic Acid Diethylamide)
----------------------------------------
24. Which substance is most likely responsible for a patient's symptoms of euphoria, dilated pupils,
tachycardia, hypertension, sweating, auditory hallucinations, tactile hallucinations of insects under the
skin, and scratch marks on the body?
(or)
A patient presents with symptoms of euphoria, dilated pupils, tachycardia, hypertension, sweating, and
auditory hallucinations. The patient also reports tactile hallucinations of insects crawling under the skin.
Physical examination reveals scratch marks on the body. Which of the following substances is most
likely responsible for these symptoms?
A. Heroin
B. Marijuana
C. Cocaine
D. LSD
----------------------------------------
25. According to the Trans-theoretical Model of Change, which stage is characterized by a person's
denial of any problem with their behaviour and a lack of intention to change?
A. Pre contemplation
B. Contemplation
C. Preparation
D. Action
----------------------------------------
26. A 21-year-old male presents to a mental health clinic with concerns about his excessive gaming
habits. He reports spending most of his waking hours playing video games and neglecting his daily
activities and responsibilities. He acknowledges that his gaming behaviour has led to conflicts with
family members and has caused financial difficulties. Based on the given information, what is the most
appropriate diagnosis for this individual?
(or)
What is the most appropriate diagnosis for a 21-year-old male presenting with excessive gaming
habits, neglecting daily activities, and experiencing conflicts with family members and financial
difficulties?
A. Major Depressive Disorder
B. Internet Addiction Disorder
C. Gaming Disorder
D. Obsessive-Compulsive Disorder
----------------------------------------
27. Alcohol can interact with various medications, leading to unexpected effects. Which of the following
drugs, when taken concurrently with alcohol, may not result in a disulfiram-like reaction?
Page 7
158
A. Tolbutamide
B. Furazolidine
C. Chloramphenicol
D. Diazepam
----------------------------------------
28. The "beer potomania" phenomenon can occur in individuals who consume excessive amounts of
alcohol. What electrolyte imbalance is commonly associated with this condition due to dilutional
effects?
A. Hypernatremia
B. Hyponatremia
C. Hyperkalemia
D. Hypokalemia
----------------------------------------
29. A 25-year-old individual who has been a daily cannabis user for several years decides to quit.
Which of the following withdrawal symptoms is most commonly associated with cannabis cessation?
A. Profuse sweating, fever, hypertension
B. Hallucinations and delirium
C. Severe agitation and violence
D. Irritability and disturbed sleep patterns
----------------------------------------
30. A 45- year-old patient with seizure disorder is seeking help to quit smoking. Which non-nicotine
medication is contraindicated for this patient due to potential risk of inducing seizures at higher doses?
A. Bupropion
B. Nicotine nasal spray
C. Varenicline
D. Nicotine gum
----------------------------------------
Correct Answers
Question Correct Answer
Question 1 4
Question 2 1
Question 3 1
Question 4 4
Question 5 3
Question 6 3
Page 8
159
Question 7 2
Question 8 2
Question 9 2
Question 10 2
Question 11 4
Question 12 1
Question 13 2
Question 14 4
Question 15 2
Question 16 2
Question 17 4
Question 18 1
Question 19 2
Question 20 1
Question 21 4
Question 22 4
Question 23 4
Question 24 3
Question 25 1
Question 26 3
Question 27 4
Question 28 2
Question 29 4
Question 30 1
Page 9
160
Option C - Gamma-aminobutyric acid (GABA): GABA is an inhibitory neurotransmitter and can mo
dulate the activity of the reward pathway. It plays a
role in addiction by regulating the balance between inhibition and excitation in the brain.
Page 10
161
• The principle of a Breathalyzer is based on the fact that alcohol in the alveolar air (breath) is in
equilibrium with the alcohol content in the blood as it passes through the lungs. This equilibrium
allows the measurement of alcohol concentration in the breath to provide an estimation of the
alcohol content in the blood.
Incorrect Options:
Option A - Alcohol intoxication: This choice is incorrect because Mr. Johnson's symptoms occur 48
hours after stopping alcohol, which is beyond the timeframe of acute alcohol intoxication.
Option B - Hepatic Encephalopathy: Hepatic Encephalopathy is a neuropsychiatric syndrome asso
ciated with liver dysfunction. It can occur in chronic alcoholics with liver disease and is characterize
d by confusion, disorientation, and personality changes.
Option D - Alcohol-related psychosis: While alcohol-related psychosis can occur with heavy alcohol
use, the presence of confusion, disorientation, and vital sign abnormalities in this scenario is more
indicative of Delirium Tremens.
Page 11
162
Option C - Alzheimer's disease: Alzheimer's disease is a
progressive neurodegenerative disorder that primarily affects the brain, leading to a decline in cogn
itive function and memory and behavioural changes. It is not associated with the acute onset of co
nfusion, eye movement abnormalities, and ataxia seen in this case.
Option D - Multiple sclerosis: Multiple sclerosis (MS) is a chronic autoimmune disease that affects t
he central nervous system, which includes the brain and spinal cord.While multiple sclerosis can pr
esent with neurological symptoms such as impaired coordination and abnormal eye movements, th
e history of chronic alcoholism and the presence of thiamine deficiency point towards Wernicke's E
ncephalopathy as the most likely diagnosis.
Page 12
163
Option D - Huntington's Disease: It is a genetic neurodegenerative disorder characterized by motor
abnormalities, cognitive decline, and psychiatric symptoms. While cognitive deterioration and mov
ement abnormalities may overlap with Marchiafava-Bignami Disease, the distinctive demyelination
pattern observed in the corpus callosum, optic tracts, and cerebellar peduncles is not seen in Hunti
ngton's Disease.
Page 13
164
Solution for Question 12:
Correct Option A: Carbohydrate Deficient Transferrin (CDT)
• It is a laboratory parameter that shows high specificity for detecting alcohol use. Elevated levels of
CDT are observed after heavy drinking and it has good sensitivity and specificity.
Incorrect Options:
Option B - Gamma-Glutamine Transferase (GGT): It is a liver enzyme that increases in heavy drink
ing, but it is not highly specific for alcohol use as its levels can also be elevated due to other factors
.
Option C - Mean Corpuscular Volume (MCV): It is a parameter that is elevated in heavy drinking, b
ut it is not highly specific to alcohol use as it can also be affected by other conditions.
Option D - Uric Acid: Levels may be high-normal in regular heavy drinkers, but it is not a
highly specific marker for alcohol use.
Page 14
165
Option B - Disulfiram is used to treat alcohol withdrawal symptoms: Disulfiram is not used to treat a
lcohol withdrawal symptoms, but rather as a deterrent agent to discourage alcohol consumption by
inducing an unpleasant reaction upon alcohol ingestion.
Option C - Disulfiram should not be initiated immediately after alcohol intake: Instead, a sufficient p
eriod of time (at least 12 hours) should elapse between the last alcohol intake and starting Disulfira
m to avoid the occurrence of a Disulfiram Alcohol Reaction.
Page 15
166
Solution for Question 17:
Correct Option D: Hallmark symptoms include euphoria, dilated pupils, and tachycardia.
• The given information states that amphetamine intoxication is characterized by symptoms such as
euphoria, pupillary dilation, tachycardia, hypertension, and cardiac arrhythmias.
Incorrect Options:
Option A - It primarily affects the serotonin receptors in the brain: It is incorrect as amphetamine pri
marily affects the release of catecholamines such as dopamine or NE, not serotonin receptors.
Option B - Symptoms may include pupillary constriction and bradycardia: It is incorrect as ampheta
mine causes pupillary dilation, not constriction, and it typically increases heart rate (tachycardia), n
ot decreases it (bradycardia).
Option C - It commonly causes hallucinations similar to LSD: It is incorrect as amphetamine-induce
d hallucinations are not similar to those caused by LSD.
Page 16
167
Solution for Question 19:
Correct Option B: 1-A, 2-B, 3-C, 4-D
• Bhang is derived from dried leaves.
• Ganja is derived from flowering stems.
• Hashish is derived from dried resin.
• Hash Oil is an oil-based extract.
Page 17
168
Option C - Running Amok: Running amok is a rare syndrome characterized by sudden outbursts of
aggressive and violent behaviour. It is not directly related to cannabis use. People experiencing ru
nning amok may engage in acts of physical violence, including harming or killing others. This syndr
ome is not specific to cannabis use.
Page 18
169
Solution for Question 24:
Correct Option C: Cocaine
• The given symptoms are consistent with cocaine intoxication. Cocaine use can lead to euphoria,
dilated pupils, increased heart rate (tachycardia), high blood pressure (hypertension), excessive
sweating, and auditory hallucinations. Tactile hallucinations, known as "Cocaine Bugs," are a
specific manifestation associated with cocaine use. The presence of scratch marks on the body
further supports the suspicion of cocaine use.
Incorrect Options:
Option A - Heroin: It is an opioid drug and its intoxication presents with different symptoms compar
ed to the scenario described. Heroin intoxication typically includes symptoms such as euphoria, dr
owsiness, constricted pupils, slowed breathing, and nodding off.
Option B - Marijuana: It use is not typically associated with the specific symptoms mentioned in the
scenario. While marijuana can cause euphoria and altered perception, it is less likely to cause hall
ucinations or tactile sensations of insects crawling under the skin.
Option D - LSD: It is a hallucinogenic drug, but its intoxication does not typically manifest with the s
pecific symptoms described in the scenario. LSD intoxication commonly involves visual hallucinatio
ns, perceptual distortions, and changes in mood and cognition. It is not typically associated with tac
tile hallucinations or scratch marks on the body.
Incorrect Options:
Option B - Contemplation: This stage occurs when the person starts realizing that they have a prob
lem and considers the risks and benefits of stopping their behaviour. However, they are not yet co
mmitted to taking action.
Option C
- Preparation: During this stage, the person intends to stop their behaviour and begins making a
plan to do so. They are actively preparing for change.
Option D - Action: In the action stage, the person takes action and makes changes to their behavio
ur. They actively stop the behaviour they want to change.
Page 19
170
• The description in the scenario aligns with the criteria for Gaming Disorder. The individual exhibits
impaired control over gaming, gives priority to gaming over daily activities, continues gaming
despite negative consequences (such as conflicts and financial difficulties), and experiences
impairment in functioning. Therefore, Gaming Disorder is the most appropriate diagnosis.
Incorrect Options:
Option A: Major Depressive Disorder.: While excessive gaming can be associated with symptoms
of depression, the primary focus of the scenario is on problematic gaming behaviour and its conse
quences. The impairment in daily activities and the continuation of gaming despite negative conseq
uences are more indicative of Gaming Disorder.
Option B - Internet Addiction Disorder: Internet Addiction Disorder is not the most appropriate diag
nosis in this scenario, as the individual's excessive gaming behaviour is specified and emphasized.
Internet Addiction Disorder encompasses a
broader range of internet-related activities, and although gaming can be a
part of it, the scenario suggests that gaming is the primary concern.
Option D - Obsessive-Compulsive Disorder: While excessive gaming can share some similarities w
ith obsessive-compulsive behaviours, the scenario does not provide sufficient information to suppo
rt an OCD diagnosis. The emphasis is on gaming behaviour and its consequences rather than spe
cific obsessions or compulsions associated with OCD.
Incorrect Options:
Option A - Tolbutamide: Tolbutamide is a sulfonylurea medication used to treat type 2
diabetes. When taken concurrently with alcohol, it can lead to a
disulfiram-like reaction, causing symptoms such as nausea and facial flushing.
Option B - Furazolidine: Furazolidine is an antibiotic used to treat bacterial infections. When combi
ned with alcohol, it can also result in a
disulfiram-like reaction, with symptoms including nausea and flushing.
Option C - Chloramphenicol: Chloramphenicol is an antibiotic, and its concurrent use with alcohol c
an lead to a disulfiram-like reaction, causing symptoms similar to those produced by disulfiram, incl
uding nausea, vomiting, and headache.
Page 20
171
symptoms such as confusion, nausea, headache, and, in severe cases, seizures and coma.
Incorrect Options:
Option A - Hypernatremia: Hypernatremia indicates elevated levels of sodium in the blood. It is not
associated with "beer potomania" because excessive beer consumption typically results in dilution
al hyponatremia.
Option C - Hyperkalemia: Hyperkalemia is an elevated level of potassium in the blood and is not ty
pically associated with "beer potomania." Excessive beer consumption is more likely to lead to hyp
onatremia due to dilutional effects.
Option D - Hypokalemia: Hypokalemia is a low level of potassium in the blood and is not directly as
sociated with "beer potomania." While alcohol can affect potassium levels, it is not the primary elec
trolyte imbalance observed in this context.
Incorrect Options:
Option A - Profuse sweating, fever and hypertension: Profuse sweating, fever and hypertension ar
e not typical symptoms of cannabis withdrawal. These symptoms are more commonly associated
with withdrawal from substances like opioids or certain stimulants.
Option B - Hallucinations and delirium: Hallucinations and delirium are not typical cannabis withdra
wal symptoms. These severe symptoms are more characteristic of alcohol withdrawal, benzodiaze
pine withdrawal, or withdrawal from certain hallucinogenic substances.
Option C - Severe agitation and violence: While irritability and mood swings are common in cannab
is withdrawal, the manifestation of severe agitation and violence is not a typical symptom. These b
ehaviors are more commonly associated with substances that can induce extreme agitation, such
as amphetamines or some hallucinogens.
Incorrect Options:
Page 21
172
Option B - Nicotine nasal spray: Nicotine nasal spray is a form of nicotine replacement therapy use
d for smoking cessation. It is generally considered safe for patients with seizure disorder.
Option C - Varenicline: Varenicline is a
medication used for smoking cessation. It is not contraindicated for patients with seizure disorder.
Option D
- Nicotine gum: Nicotine gum is another form of nicotine replacement therapy that provides a contr
olled release of nicotine to help with smoking cessation. It is generally considered safe for patients
with seizure disorder.
Page 22
173
Previous Year Questions
1. Which drug among the options provided would assist a chronic smoker in their aim to quit smoking?
A. Bupropion
B. Mirtazapine
C. Varenicline
D. Both a and c
----------------------------------------
2. What is the predominant manifestation experienced with cannabis consumption?
A. Anxiety
B. Depressed mood
C. Dream-like state
D. Decreased appetite
----------------------------------------
3. What is the probable diagnosis for a chronic alcoholic who arrives at the emergency department
presenting confusion, ataxia, painful eye movements, and involvement of the 6th cranial nerve?
A. Wernicke's encephalopathy
B. Korsakoff psychosis
C. Delirium tremens
D. De Clerambault syndrome
----------------------------------------
4. How would you treat a patient who is a long-term alcoholic and is currently experiencing seizures,
altered sensorium, auditory hallucinations, and secondary delusions, with his last drink being 2 days
ago? The patient has consumed an average of 24 units per day for the past 10 years. Diazepam
Thiamine Naltrexone Flupenthixol depot injection
A. 1, 4
B. 1, 2, 4
C. 1, 3, 4
D. 1, 2
----------------------------------------
5. Which of the following statements accurately describes lithium?
A. It can cause fine postural tremors at therapeutic dosage
B. It is not absorbed from the gut
C. It is not teratogenic
D. It is also used for treatment of absence seizures
----------------------------------------
174
6. A patient stopped alcohol consumption for 3 days and presented with irritability, disorientation,
paranoid delusions, agitation, visual hallucinations, and altered sensorium. What is the likely diagnosis
in this case?
A. Delirium tremens
B. Wernicke’s encephalopathy
C. Korsakoff psychosis
D. Alcohol-induced psychosis
----------------------------------------
7. Which of the following drug abuses can lead to the symptoms described by a 25-year-old male
patient, who complains of a sensation resembling insects crawling beneath his skin, and has presented
to the clinic?
A. Cannabis
B. Cocaine
C. Amphetamine
D. Alcohol
----------------------------------------
8. In the scenario provided, what is the probable diagnosis for a patient who ceased drinking alcohol for
three days and subsequently exhibited symptoms such as irritability, disorientation, paranoid delusions,
agitation, visual hallucinations, and changes in consciousness?
A. Delirium tremens
B. Wernicke’s encephalopathy
C. Korsakoff psychosis
D. Alcohol-induced psychosis
----------------------------------------
9. What is a true statement about nicotine substitution therapy?
A. Preferably given by gastrointestinal route.
B. Varenicline comes with a black box warning of cardiovascular death
C. There should be a 15-minute gap between nicotine gum and coffee/soda/acidic food as they
decrease its absorption
D. Nicotine chewing gum is better for constant use as it gives 25% higher blood level than lozenges
----------------------------------------
10. Which of the following statements accurately describes Korsakoff syndrome?
A. Triad of confusion, ophthalmoplegia, and ataxia
B. Loss of long-term memory but intact personality
C. Anterograde amnesia with loss of recent memory
D. Loss of implicit memory and immediate recall
----------------------------------------
11. Which drug listed below is utilized as an agent for smoking cessation?
Page 2
175
A. Busulfan
B. Acamprosate
C. Varenicline
D. Gabapentin
----------------------------------------
12. A known alcoholic patient is brought to the emergency in a disoriented state. He complains of
seeing snakes and insects crawling on his skin. Which of the following should be given to this patient?
A. Haloperidol + thiamine
B. Lorazepam only
C. Lorazepam + thiamine
D. Haloperidol only
----------------------------------------
13. What is the initial indication of alcohol withdrawal?
A. Tremors
B. Delirium tremens
C. Seizures
D. Hallucinations
----------------------------------------
14. What is the probable diagnosis for a 40-year-old male patient who was admitted to the ICU three
days ago due to breathing difficulty and is currently experiencing restlessness and widespread
tremors? The patient has reported seeing insects crawling on his bed and body and has had one
seizure, which was successfully managed with lorazepam. Further investigation reveals that the patient
is a chronic alcoholic.
A. ICU psychosis
B. Alcohol intoxication
C. Wernicke-Korsakoff psychosis
D. Delirium tremens
----------------------------------------
15. What treatment should be administered to a patient, known to be an alcoholic, who is brought to the
emergency department in a confused state and complains of visual hallucinations involving snakes and
insects crawling on their skin?
A. Haloperidol + thiamine
B. Lorazepam only
C. Lorazepam+thiamine
D. Haloperidol only
----------------------------------------
16. Jet black pigmentation of the tongue with tactile hallucinations is a feature of:
A. Heroin
Page 3
176
B. Opium
C. Alcohol
D. Cocaine
----------------------------------------
17. What is the probable diagnosis for a 30-year-old male who underwent appendicectomy 7 hours ago
and is currently experiencing auditory hallucinations and confusion about the source of the voices? The
patient maintains orientation, and his vital signs are stable. Upon inquiry, his wife reveals that he
occasionally consumes alcohol.
A. Delirium tremens
B. A/c transient psychotic disorder
C. Delirium
D. Alcoholic hallucinosis
----------------------------------------
18. What is the etiology of delirium tremens in a 48-year-old male who presents to the emergency room
with symptoms of confusion and generalized body tremors, and has a known history of heavy alcohol
use?
A. Acute infection
B. Fatty liver
C. Gradual withdrawal of alcohol
D. Small does of regular consumption of alcohol
----------------------------------------
19. Which of the following adverse effects is the least probable in a patient prescribed with
escitalopram?
A. Nausea
B. Vivid dreams
C. Anorgasmia
D. Sialorrhea
----------------------------------------
Correct Answers
Question Correct Answer
Question 1 4
Question 2 3
Question 3 1
Question 4 4
Question 5 1
Question 6 1
Page 4
177
Question 7 2
Question 8 1
Question 9 3
Question 10 3
Question 11 3
Question 12 3
Question 13 1
Question 14 4
Question 15 3
Question 16 4
Question 17 4
Question 18 3
Question 19 4
Page 5
178
• Option a. Anxiety: While some people may experience anxiety or paranoia after using cannabis,
this is not the most common symptom. In fact, some strains of cannabis are known for their calming
and anxiety-reducing effects.
• Option b. Depressed mood: Cannabis can have mood-altering effects, but a depressed mood is
not the most common symptom. In some cases, cannabis may actually improve mood and alleviate
symptoms of depression.
• Option d. Decreased appetite: Cannabis is known for causing "the munchies," or an increase in
appetite rather than a decrease. While some strains may have more appetite-suppressing effects,
this is not the most common symptom.
Page 6
179
Option C - 1, 3, 4: Naltrexone is commonly used to treat alcohol dependence. It blocks the effects
of opioids in the brain, reducing the desire to consume opioids and alcohol.
Page 7
180
"cocaine bugs." Cocaine is a powerful stimulant drug that affects the central nervous system. It
increases the levels of dopamine, a neurotransmitter associated with pleasure and reward, in the
brain.
• The sensation of formication is a common side effect of cocaine abuse and is believed to be
related to the drug's stimulant effects on the nervous system. The exact mechanism of how cocaine
causes this sensation is not fully understood, but it is thought to involve alterations in the brain's
perception and processing of sensory information.
Incorrect options:
Option A: Cannabis (marijuana) is a different class of drug that primarily acts as a depressant and
hallucinogen. It is not typically associated with the symptom of feeling insects crawling under the sk
in.
Option C: Amphetamine is a stimulant drug similar to cocaine, but it is not specifically known to cau
se the sensation of formication.
Option D: Alcohol abuse can lead to various neurological and psychological symptoms, but the feel
ing of insects crawling under the skin is not a typical manifestation of alcohol abuse.
Page 8
181
Solution for Question 10:
• Korsakoff syndrome is a chronic condition associated with severe thiamine deficiency due to alcohol
misuse. Characteristic features are loss of recent memory and anterograde amnesia. They also present
with learning difficulties and confabulation. It is treated by Thiamine supplementation of 100 mg per oral
twice or thrice daily for 3-12 months.
Incorrect Choices:
Page 9
182
Option A and D: Haloperidol + thiamine: Haloperidol only
• Haloperidol is indicated only when the symptoms and agitation are severe to control high doses of
Benzodiazepines and thiamine may not suffice. In those cases, we can add antipsychotics such as
Olanzapine, or Haloperidol.
Option B: Lorazepam only
• Here the patient also exhibits symptoms of thiamine deficiency so Lorazepam does not suffice.
Thiamine should be given along with lorazepam.
• After 6-8 hours of stopping alcohol patients develop mild symptoms where the tremor is the
classical or the most common sign. The coarse tremors include hand and body shaking. Other
early symptoms include GI symptoms (Nausea, Vomiting) , Autonomic Hyperactivity (Anxiety
arousal, Sweating, Mydriasis, Tachycardia, Hypertension)
Incorrect choices:
Option B: Delirium tremens :
The patient experiences the condition within 72 hours of stopping alcohol.This is a
life-threatening symptom.
Option C: Seizures : Occurs after 12-24 hours of stopping alcohol. The withdrawal symptoms are s
eizures. If the seizures are jerking movements of the body. These seizures are Generalized Tonic-
Clonic Seizures (GTCS).
Option D: Hallucinations: Occurs after 8-12 hours of stopping alcohol (Moderate Symptoms): The
withdrawal symptom is alcoholic hallucinosis (Hallucinations, auditory hallucinations more common
, visual hallucinations may also be seen). This hallucination occurs in clear consciousness. No imp
airment of consciousness.
Page 10
183
Solution for Question 14:
Correct Option: D
• A person who has been drinking heavily for a long period may have delirium tremens (DTs), a
severe type of alcohol withdrawal. DTs can happen if the individual stops drinking suddenly. DTs
are a medical emergency that poses a risk to life. Alcohol users may get delirium tremens, a severe
withdrawal condition. Some of its symptoms include trembling, anxiety, disorientation, and
hallucinations.
• Delirium tremens is a serious medical condition that requires immediate attention. Minor liquor
withdrawal is described by quakes, nervousness, queasiness, spewing, and sleep deprivation.
Significant liquor withdrawal signs and side effects incorporate visual mind flights and hearable
pipedreams, entire body quake, heaving, diaphoresis, and hypertension (hypertension).
Exaggerated autonomic hyperactivity, a hyperadrenergic condition, and central nervous system
excitability distinguish it.
• Delirium tremens is a condition that can cause seizures as well. In this case, the patient
experienced a seizure that was managed with lorazepam, a drug that is frequently used to treat
alcohol withdrawal. Typically, DTs symptoms appear 2 to 3 days after someone quits drinking. Here
the patient starts hallucinating three days post-admission to ICU. This is definitely a symptom of
Delirium Tremens. The history of chronic alcoholism further supports this diagnosis, as Delirium
tremens are most commonly associated with alcohol withdrawal.
Incorrect Choices:
Option A. ICU psychosis: ICU psychosis is a mental health condition that can develop in people wh
o are hospitalized in an intensive care unit (ICU). It is often caused by the stress of being in the IC
U, as well as the use of sedatives and other medications. ICU psychosis can cause various sympto
ms, including hallucinations, delusions, and agitation. DTs are caused by alcohol withdrawal, while
ICU psychosis is caused by the stress of being in the ICU.
Option B. Alcohol Intoxication: The patient is not having alcohol intoxication because he has been
sober for three days. Alcohol intoxication is a state of intoxication that occurs when a
person drinks too much alcohol. Alcohol intoxication can cause a variety of symptoms, including sl
urred speech, impaired coordination, and loss of consciousness. In severe cases, alcohol intoxicati
on can lead to death. The signs and symptoms associated with alcohol toxicity depend on the bloo
d alcohol concentration (BAC). As the BAC increases, so does the severity of the symptoms. At a
BAC of 0 to 50%, one would expect to see relaxation, increased talkativeness, and decreased fine
motor control. At a BAC of 50% to 100%, patients develop impaired judgment and coordination. Fr
om 100% to 200%, one sees gait instability, slurred speech, and mood and behavior changes. At a
BAC of 200% to 400%, patients develop nausea and vomiting, hypothermia, dysarthria, amnesia, d
iplopia, and nystagmus. With a BAC of greater than 400%, patients can develop respiratory depres
sion followed by coma and even death.
Option C. Wernicke-Korsakoff psychosis: Wernicke-Korsakoff psychosis is a
neurological disorder that is caused by a
deficiency of thiamine (vitamin B1). Wernicke-Korsakoff syndrome can cause a variety of symptom
s, including confusion, memory loss, and ataxia (loss of coordination). Wernicke's encephalopathy
represents the "acute" phase of the disorder, and Korsakoff's amnesic syndrome represents the di
sorder progressing to a "chronic" or long-lasting stage. The disorder's main features are problems i
n acquiring new information or establishing new memories and in retrieving previous memories.
Page 11
184
Solution for Question 15:
The correct answer is C
• Lorazepam + thiamine. In an alcoholic patient in a disoriented state with hallucinations, the
management involves treating alcohol withdrawal syndrome and preventing Wernicke-Korsakoff
syndrome.
Incorrect options:
Option A. Haloperidol + thiamine: Haloperidol is an antipsychotic medication commonly used to ma
nage acute psychosis, but it is not the first-line treatment for alcohol withdrawal syndrome.
Option B. Lorazepam only: Lorazepam alone may provide symptomatic relief for anxiety or agitatio
n but does not address the underlying alcohol withdrawal syndrome or thiamine deficiency.
Option D. Haloperidol only: Haloperidol alone does not address the underlying alcohol withdrawal s
yndrome or thiamine deficiency and may not be suitable as the sole treatment in this case.
Page 12
185
he is now having auditory hallucinations. The threatening and accusatory hallucinations cause the
patient's confusion. Alcoholic hallucinations are consistent with these symptoms. A particular kind
of alcohol-induced psychotic disorder known as alcoholic hallucinosis can occur during or shortly
after alcohol withdrawal. The way that the patient is a periodic alcoholic further backs this
determination, as liquor withdrawal is a typical trigger for the improvement of alcoholic hallucinosis.
Incorrect Options:
Option A: Delirium tremens A more severe withdrawal syndrome known as delirium tremens can af
fect alcohol users. Tremors, agitation, confusion, and hallucinations are some of its symptoms. Deli
rium tremens is a medical emergency that must be treated right away. Minor liquor withdrawal is de
scribed by the quake, nervousness, queasiness, spewing, and sleep deprivation. Significant liquor
withdrawal signs and side effects incorporate visual mind flights and hearable pipedreams, entire b
ody quake, heaving, diaphoresis, and hypertension (hypertension).
Option B: A/c transient psychotic disorder: Psychological stress can lead to the sudden onset of ps
ychotic symptoms in Acute and Transient Psychotic Disorder (ATPD). In this, episodes are present
moments, enduring from days to 90 days, trailed by complete abatement. Delusions, hallucination
s, and mood swings are their hallmarks. Most of the time, transient psychotic disorder goes away o
n its own after a few weeks or a month.
Option C: Delirium: Delirium is a mental state in which you can't think clearly or remember anything
, are lost, and are confused. There are many things that can cause delirium, such as an infection, t
aking medication, or going through a medical procedure. It usually appears out of nowhere. It is fre
quently brief and treatable. There are three types of delirium:
• Hypoactive, where one is not active and seems sleepy, tired, or depressed.
• Hyperactive, where one is restless or agitated.
• Mixed, where one changes back and forth between being hypoactive and hyperactive.
Page 13
186
Correct Option:
Option D (Sialorrhea). Sialorrhea refers to excessive salivation or drooling, and it is not commonly
associated with the use of escitalopram. Sialorrhea, or excessive salivation, is not typically associa
ted with the use of escitalopram. Other medications or medical conditions may be more commonly
linked to this side effect.
Incorrect Options:
Option A. Nausea: Nausea is a common side effect of escitalopram and can occur during the initial
stages of treatment. It usually subsides over time.
Option B. Vivid dreams: Escitalopram can cause changes in dream patterns, including vivid dream
s. This is a relatively common side effect.
OptionC. Anorgasmia: Anorgasmia refers to the difficulty or inability to achieve orgasm. It is a
known side effect of selective serotonin reuptake inhibitors (SSRIs) such as escitalopram.
Page 14
187
Child Psychiatry
1. A 10-year-old boy named Michael has been struggling in school. He frequently makes careless
mistakes in his assignments and has difficulty completing tasks. He often daydreams during class and
is easily distracted by sounds and movements around him. Additionally, he has trouble staying seated
and is constantly fidgeting, tapping his hands and feet. Michael frequently interrupts his classmates
during group activities and has difficulty waiting for his turn. Based on the information provided, what is
the most likely diagnosis for Michael?
(or)
What is the most likely diagnosis for a 10-year-old boy named Michael who struggles in school, makes
careless mistakes, is easily distracted, fidgets, and interrupts others frequently during group activities?
A. Generalized Anxiety Disorder (GAD)
B. Oppositional Defiant Disorder (ODD)
C. Autism Spectrum Disorder (ASD)
D. Attention Deficit Hyperactivity Disorder (ADHD)
----------------------------------------
2. A 10-year-old child is frequently having difficulty paying attention, is easily distracted, and often fails
to complete tasks both at home and school. These difficulties have been present for the past 9 months.
Which of the following criteria should be considered for a diagnostic evaluation?
(or)
Which criteria should be considered for a diagnostic evaluation of a 10-year-old child who frequently
exhibits difficulty paying attention, being easily distracted, and failing to complete tasks for the past 9
months?
A. Onset of symptoms before 7 years of age, symptoms present in two or more settings, symptoms
present for more than 6 months.
B. Onset of symptoms before 12 years of age, symptoms present in one setting only, symptoms
present for less than 6 months.
C. Onset of symptoms before 7 years of age, symptoms present in one setting only, symptoms present
for more than 6 months.
D. Onset of symptoms before 12 years of age, symptoms present in two or more settings, symptoms
present for more than 6 months.
----------------------------------------
3. What is the drug of choice for a 9-year-old child exhibiting symptoms of inattention, hyperactivity, and
impulsivity, with no known cardiac issues, and the symptoms have been present for the past 8 months?
(or)
A 9-year-old child is exhibiting symptoms of inattention, hyperactivity, and impulsivity. The symptoms
have been present for the past 8 months and are observed both at school and at home. The parents
are seeking advice regarding the most appropriate medication for their child. The child does not have
any known cardiac issues, and the family history is negative for cardiac problems. What is the drug of
choice for the patient in this case?
A. Atomoxetine
B. Methylphenidate
188
C. Clonidine
D. Amphetamine
----------------------------------------
4. What is the most likely diagnosis for a 4-year-old boy named Alex, who exhibits social
communication deficits, repetitive behaviors, and difficulty adapting to changes in routine?
(or)
A 4-year-old boy named Alex is brought to the clinic by his parents. They express concerns about his
social skills and behavior. During the assessment, you observe that Alex does not make eye contact,
seems disinterested in interacting with others, and does not respond when his name is called. He
engages in repetitive hand flapping and shows a strong attachment to a particular toy, which he plays
with in a repetitive and ritualistic manner. Alex becomes upset when there are changes in his routine
and displays difficulty in adapting to new situations. Based on the presented scenario, what is the most
likely diagnosis for Alex?
A. Attention Deficit Hyperactivity Disorder (ADHD)
B. Oppositional Defiant Disorder (ODD)
C. Autism Spectrum Disorder (ASD)
D. Generalized Anxiety Disorder (GAD)
----------------------------------------
5. A 6-year-old boy named Ethan is brought to the clinic by his parents due to concerns about his
development. During the assessment, you notice minor physical anomalies in his ears. His parents
mention that Ethan has difficulty with language, often reverses pronouns, and struggles to form
meaningful sentences. Additionally, Ethan displays aggressive behavior and frequently engages in
self-injurious actions like hand banging. He shows heightened interest in certain sounds, but seems
less responsive to other sounds. Despite these challenges, Ethan demonstrates exceptional reading
skills, reading at a level well beyond his peers. Based on the presented scenario, what is the most likely
diagnosis for Ethan?
(or)
What is the most likely diagnosis for a 6-year-old boy named Ethan, who exhibits language difficulties,
self-injurious behavior, exceptional reading skills, and minor physical anomalies in his ears?
A. Attention Deficit Hyperactivity Disorder (ADHD)
B. Specific Language Impairment (SLI)
C. Autism Spectrum Disorder (ASD)
D. Conduct Disorder (CD)
----------------------------------------
6. Which genetic disorder is most commonly associated with Autism Spectrum Disorder (ASD)?
A. Fragile X syndrome
B. Down syndrome
C. Turner syndrome
D. Klinefelter syndrome
----------------------------------------
Page 2
189
7. A 3-year-old boy named Jacob is brought to the pediatrician by his parents due to concerns about his
development. Jacob's parents mention that he had been developing typically in the first 2 years of his
life. However, over the past year, Jacob has lost the ability to communicate verbally, interact socially,
and perform previously acquired skills such as using the toilet and engaging in play activities. His
parents have also observed that Jacob experiences occasional seizures. Based on the presented
scenario, what is the most likely diagnosis for Jacob?
(or)
What is the most likely diagnosis for 3-year-old Jacob having normal development for first 2 years of
life, who has experienced a significant regression in communication, social interaction, and previously
acquired skills, along with occasional seizures?
A. Rett syndrome
B. Heller's syndrome
C. Intellectual Disability
D. Specific Language Impairment (SLI)
----------------------------------------
8. Sarah, a 9-year-old girl, is brought to a developmental clinic by her parents. They have noticed some
difficulties in her social interactions and behavior. Sarah has good language skills and has never
experienced any language delay. She tends to struggle with non-verbal communicative gestures and
often engages in repetitive behaviors. The parents are concerned about her condition and seek a
diagnosis. Based on the presented scenario, what is the most likely diagnosis for Sarah?
(or)
What is the most likely diagnosis for 9-year-old Sarah, who exhibits difficulties in social interactions,
struggles with non-verbal communicative gestures, and engages in repetitive behaviors, but has good
language skills and no history of language delay?
A. Intellectual Disability Disorder
B. Asperger syndrome
C. Specific Language Impairment (SLI)
D. Attention-Deficit/Hyperactivity Disorder (ADHD)
----------------------------------------
9. Emma, a 9-year-old student, has difficulty recognizing words, exhibits slow and inaccurate reading,
experiences poor comprehension, and struggles with spelling. Which specific learning disorder is she
likely to have?
(or)
What specific learning disorder is likely present in 9-year-old Emma, who has difficulty recognizing
words, exhibits slow and inaccurate reading, experiences poor comprehension, and struggles with
spelling?
A. Dysgraphia
B. Dyscalculia
C. Dyslexia
D. Dyspraxia
----------------------------------------
Page 3
190
10. What is the typical range of IQ in individuals with intellectual disability (mental retardation)?
A. IQ between 100-119
B. IQ between 90-109
C. IQ between 80-89
D. IQ below 70
----------------------------------------
11. What is the calculated IQ for an individual with the following information? Chronological age: 12
years Mental age: 8 years
A. 33.33
B. 50
C. 66.67
D. 75
----------------------------------------
12. Which of the following characteristics is consistent with profound retardation?
A. IQ of 40
B. Independent performance of daily tasks
C. Mental age of 8 years
D. Need for continuous supervision
----------------------------------------
13. What is the most common genetic cause of intellectual disability?
A. Down syndrome
B. Fragile X syndrome
C. Prader-Willi syndrome
D. Williams syndrome
----------------------------------------
14. Sarah, a 12-year-old girl, frequently argues with her parents, becomes easily annoyed by others,
and often puts the blame on others for her mistakes. She exhibits patterns of negativistic, disobedient,
and hostile behavior towards authority figures. However, there is no physical aggression or destructive
behavior. Which of the following best describes Sarah's condition?
(or)
What best describes the condition of 12-year-old Sarah, who frequently argues with her parents,
becomes easily annoyed by others, and exhibits patterns of negativistic, disobedient, and hostile
behavior towards authority figures without physical aggression or destructive behavior?
A. Conduct Disorder
B. Oppositional Defiant Disorder (ODD)
C. Attention-Deficit/Hyperactivity Disorder (ADHD)
D. Generalized Anxiety Disorder (GAD)
Page 4
191
----------------------------------------
15. What disorder is 15-year-old Maria most likely experiencing, given her history of physical fights,
threatening behavior, bullying, stealing, property destruction, breaking rules, staying out late at night,
and skipping school?
(or)
Maria, a 15-year-old girl, frequently engages in physical fights, threatens others, and bullies her
classmates. She has been caught stealing from stores and has a history of destroying property. Despite
her parents' rules, she often stays out late at night. She has also been skipping school for the past year.
Which disorder is Maria most likely experiencing?
A. Oppositional Defiant Disorder
B. Conduct Disorder
C. Antisocial Personality Disorder
D. Generalized Anxiety Disorder
----------------------------------------
16. Sarah, a 10-year-old girl, has been displaying severe recurrent temper outbursts. Her parents are
concerned about her behavior and seek professional help. After assessment, Sarah is diagnosed with
Disruptive Mood Dysregulation Disorder (DMDD). What are the key features of Disruptive Mood
Dysregulation Disorder (DMDD) exhibited by Sarah?
(or)
What are the key features of Disruptive Mood Dysregulation Disorder (DMDD)?
A. Verbal and behavioral aggression towards others, occurring at least three times per week.
B. Episodes of euphoria and manic behavior.
C. Symptoms of annoyance and defiance in one setting.
D. Symptoms resembling anxiety disorder with excessive worry.
----------------------------------------
17. Which of the following statements accurately describes complex motor tics?
A. Complex motor tics involve repetitive contraction of similar muscle groups.
B. Complex motor tics appear more purposeful and involve grooming behaviors.
C. Complex motor tics are characterized by repeating words or phrases out of context.
D. Complex motor tics primarily manifest as eye blinking and shoulder shrugging.
----------------------------------------
18. What is the most likely diagnosis for a 7-year-old boy with sudden, rapid, and recurrent motor
movements and vocalizations, lasting for several months, causing impairment, and initially presenting
as rapid eye blinking?
(or)
A 7-year-old boy presents with sudden, rapid, and recurrent motor movements and vocalizations. The
symptoms have been present for several months and are causing impairment in his daily functioning.
The parents report that the symptoms wax and wane but have not completely resolved. The initial tic
was observed as rapid eye blinking. What is the most likely diagnosis?
A. Attention-deficit/hyperactivity disorder (ADHD)
Page 5
192
B. Obsessive-compulsive disorder (OCD)
C. Tourette syndrome (Gilles de la Tourette Syndrome)
D. Major depressive disorder (MDD)
----------------------------------------
19. What is the most likely diagnosis for 2-year-old Emily, who experienced a sudden loss of motor
skills and speech, displays stereotypic hand movements, has decelerated head circumference growth,
and exhibits poor muscle coordination along with seizures and abnormal EEG findings?
(or)
A 2-year-old girl named Emily is brought to the clinic by her parents due to concerns about her
development. Her parents mention that Emily had been developing normally in the first few months
after birth. However, they noticed a sudden loss of acquired motor skills and speech. Emily now
frequently engages in stereotypic hand movements, such as hand wringing. Her head circumference
growth has decelerated, and she appears to have poor muscle coordination and an unsteady gait.
Additionally, Emily experiences seizures, and an EEG has revealed abnormal findings. Based on the
presented scenario, what is the most likely diagnosis for Emily?
A. Intellectual disability syndrome
B. Down syndrome
C. Rett syndrome
D. Cerebral palsy
----------------------------------------
Correct Answers
Question Correct Answer
Question 1 4
Question 2 4
Question 3 2
Question 4 3
Question 5 3
Question 6 1
Question 7 2
Question 8 2
Question 9 3
Question 10 4
Question 11 3
Question 12 4
Question 13 1
Question 14 2
Question 15 2
Page 6
193
Question 16 1
Question 17 2
Question 18 3
Question 19 3
Page 7
194
Solution for Question 3:
Correct Option B - Methylphenidate (ADHD): Methylphenidate is the drug of choice for the treatment of
ADHD, as mentioned in the provided information. It is a CNS stimulant and is commonly used as the fir
st-line pharmacotherapy for ADHD in children. It helps improve attention, reduce hyperactivity, and ma
nage impulsivity.
Incorrect Options:
Option A - Atomoxetine (ADHD): Atomoxetine is a non-stimulant medication used in the treatment
of ADHD. While it can be an alternative for individuals who cannot tolerate stimulant medications or
have specific contraindications, such as cardiac issues, it is not the first-line drug of choice accordi
ng to the information provided.
Option C - Clonidine (ADHD): Clonidine is an α-2 agonist that can be used as an adjunctive treatm
ent for ADHD, particularly in cases where stimulant medications exacerbate tics or in individuals wi
th comorbid tic disorders. However, it is not the first-line drug of choice for ADHD, as mentioned in
the information provided.
Option D - Amphetamine (ADHD): Amphetamine, similar to methylphenidate, is a CNS stimulant a
nd can be used in the treatment of ADHD. However, it is not the first-line drug of choice, as mentio
ned in the provided information. It may be considered as an alternative when methylphenidate fails
to provide an adequate response.
Page 8
195
Solution for Question 5:
Correct Option C - Autism Spectrum Disorder (ASD): This is the most likely diagnosis for Ethan based
on the presented scenario. He displays deficits in social communication (not explicitly mentioned in the
scenario but implied) and exhibits restricted, repetitive behaviors (aggression, self-injury). The languag
e impairment, atypical sensory responses, and exceptional reading skills align with the associated char
acteristics of ASD.
Incorrect Options:
Option A - Attention Deficit Hyperactivity Disorder (ADHD): ADHD is characterized by persistent pa
tterns of inattention, hyperactivity, and impulsivity. Although Ethan displays some challenging beha
viors like aggression, the core symptoms of ADHD related to inattention and hyperactivity are not p
rominent in the scenario.
Option B - Specific Language Impairment (SLI): SLI refers to difficulties with language developmen
t that are not attributable to intellectual disability, hearing loss, or neurological impairments. While
Ethan demonstrates language impairment, the presence of minor physical anomalies, self-injurious
behaviors, atypical sensory responses, and exceptional reading skills suggest a
broader diagnosis beyond SLI.
Option D - Conduct Disorder (CD): CD is characterized by a pattern of persistent and repetitive beh
aviors that violate the basic rights of others or societal norms. While aggression and self-injurious b
ehaviors are mentioned in the scenario, the presence of language impairment, atypical sensory res
ponses, and exceptional reading skills are not consistent with the diagnosis of CD.
Page 9
196
Solution for Question 7:
Correct Option B - Heller's syndrome:
• This is the most likely diagnosis for Jacob based on the presented scenario. Childhood
disintegrative disorder(Heller's syndrome) is characterized by normal development in the first 2
years of life, followed by a marked regression in language, social or adaptive behavior, bowel and
bladder control, play skills, and motor skills. Seizures may also be observed. The progressive
course and likelihood of moderate to severe mental retardation align with the symptoms described
for Jacob.
Incorrect Options:
Option A - Rett syndrome: Rett syndrome is a rare genetic disorder that primarily affects girls and l
eads to severe neurological and developmental problems. The scenario describes regression in m
ultiple areas, making rett syndrome less likely as the primary diagnosis.
Option C - Intellectual Disability: Intellectual disability refers to significantly below-average intellectu
al functioning and limitations in adaptive behaviors. While Jacob may have intellectual disability as
a result of his condition, the specific pattern of normal development followed by regression in multi
ple areas suggests a more specific diagnosis.
Option D - Specific Language Impairment (SLI): SLI is a disorder characterized by difficulties in lan
guage acquisition and use, with no significant deficits in other developmental areas. The scenario d
escribes regression in multiple areas beyond language skills, making SLI less likely as the primary
diagnosis.
Page 10
197
Solution for Question 9:
Correct Option C - Dyslexia:
• Dyslexia is a specific learning disorder characterized by difficulty in recognizing words, slow and
inaccurate reading, poor comprehension, and spelling difficulties. This aligns with Emma's
symptoms and is the correct answer.
Incorrect Options:
Option A - Dysgraphia: Dysgraphia is a specific learning disorder characterized by difficulties in ha
ndwriting and written expression. It does not involve reading difficulties.
Option B - Dyscalculia: Dyscalculia is a specific learning disorder related to mathematical abilities,
such as difficulty with number recognition, counting, and solving mathematical problems. It is not a
ssociated with reading impairments.
Option D - Dyspraxia: Dyspraxia, also known as developmental coordination disorder, involves diffi
culties with coordination, motor skills, and physical movements. It does not directly affect reading a
bilities.
Incorrect Options:
Option A - IQ of 40: An IQ of 40 falls within the range of moderate mental retardation, not profound
retardation. Profound retardation is characterized by an IQ below 20.
Page 11
198
Option B - Independent performance of daily tasks: Individuals with profound retardation are unabl
e to perform basic tasks independently and require continuous supervision for their daily activities.
Option C - Mental age of 8 years: Profound retardation is associated with a
mental age estimated to be less than 3 years, not 8 years.
Incorrect Options:
Option B - Fragile X syndrome: Fragile X syndrome is the second most common known genetic ca
use of intellectual disability. It is caused by a mutation in the FMR1 gene on the X chromosome.
Option C - Prader-Willi syndrome: Prader-Willi syndrome is a rare genetic disorder characterized b
y intellectual disability, hyperphagia (excessive appetite), and other physical and behavioral feature
s. It is not the most common genetic cause of intellectual disability.
Option D - Williams syndrome: Williams syndrome is a rare genetic disorder that results in intellect
ual disability, unique facial features, and cardiovascular problems. It is not the most common genet
ic cause of intellectual disability.
Incorrect Options:
Option A - Conduct Disorder: Conduct Disorder is characterized by a pattern of violating the rights
of others and societal norms. It involves aggression towards people or animals, destruction of prop
erty, and a disregard for rules. Sarah's lack of physical aggression or destructive behavior is not co
nsistent with Conduct Disorder.
Option C - Attention-Deficit/Hyperactivity Disorder (ADHD): ADHD involves symptoms of hyperactiv
ity, impulsivity, and inattention. While some individuals with ADHD may exhibit oppositional behavi
or, the absence of hyperactivity and impulsivity in Sarah's case makes ADHD less likely.
Option D - Generalized Anxiety Disorder (GAD): GAD is characterized by excessive worry and anxi
ety about various aspects of life. Sarah's symptoms, which primarily involve behavioral issues and
not excessive worry or anxiety, are not consistent with GAD.
Page 12
199
Solution for Question 15:
Correct Option B: Conduct Disorder
• Maria's behavior aligns with the symptoms of conduct disorder, which include aggression,
violation of others' rights, destruction of property, theft, and truancy. Conduct disorder is a more
severe form of disruptive behavior disorder.
Incorrect Options:
Option A - Oppositional Defiant Disorder: This disorder is characterized by negativistic, disobedient
, and hostile behavior towards authority figures. While Maria's behavior displays opposition, her act
ions go beyond the scope of oppositional defiant disorder.
Option C - Antisocial Personality Disorder: Antisocial personality disorder is a long-term pattern of
disregard for others' rights and violation of societal norms. While conduct disorder is a risk factor fo
r developing antisocial personality disorder in adulthood, Maria's age and the duration of symptoms
suggest conduct disorder rather than a full-blown personality disorder.
Option D - Generalized Anxiety Disorder: Generalized anxiety disorder is characterized by excessi
ve and persistent worry and anxiety about various aspects of life. It does not account for the aggre
ssive and disruptive behaviors exhibited by Maria.
Incorrect Options:
Option B - Episodes of euphoria and manic behavior: This option is incorrect as it describes episod
es of euphoria and manic behavior, which are more characteristic of bipolar disorder.
Option C - Symptoms of annoyance and defiance in one setting: This option is incorrect as it refers
to symptoms of annoyance and defiance in one setting, which is indicative of oppositional defiant
disorder (ODD), not DMDD.
Option D - Symptoms resembling anxiety disorder with excessive worry: This option is incorrect as
it mentions symptoms resembling anxiety disorder with excessive worry, which are not specific to
DMDD but may require a separate diagnosis and treatment approach.
Page 13
200
Incorrect Options:
Options A - Complex motor tics involve repetitive contraction of similar muscle groups: This option i
s incorrect as it describes simple motor tics involving repetitive contraction of similar muscle groups
. Complex motor tics, on the other hand, involve more elaborate and purposeful movements.
Option C - Complex motor tics are characterized by repeating words or phrases out of context: Thi
s option is incorrect as it describes vocal tics, which is the repetition of words or phrases, but does
not specifically capture the concept of complex motor tics.
Option D - Complex motor tics primarily manifest as eye blinking and shoulder shrugging: This opti
on is incorrect as it only mentions specific motor movements (eye blinking and shoulder shrugging)
and does not encompass the broader range of complex motor tics.
Incorrect Options:
Option A - Attention-deficit/hyperactivity disorder (ADHD): While ADHD is a common comorbidity in
individuals with Tourette syndrome, it primarily involves symptoms of inattention, hyperactivity, an
d impulsivity rather than sudden motor movements and vocalizations.
Option B - Obsessive-compulsive disorder (OCD): OCD is another common comorbidity in Tourett
e syndrome. However, it is characterized by intrusive thoughts and repetitive behaviors driven by a
nxiety, rather than the sudden motor and vocal tics observed in this case.
Option D - Major depressive disorder (MDD): MDD is a mood disorder characterized by persistent f
eelings of sadness, loss of interest or pleasure, and other depressive symptoms. It does not prese
nt with sudden motor movements and vocalizations as seen in this case.
Page 14
201
fests during childhood and results in difficulties with learning, problem-solving, and independent dai
ly living.
Option B - Down syndrome: Down syndrome is a genetic disorder caused by an extra copy of chro
mosome 21. It is characterized by distinct physical features, intellectual disability, and delayed dev
elopment. The symptoms described in the scenario, such as loss of acquired skills and hand move
ments, are not consistent with Down syndrome.
Option D - Cerebral palsy: Cerebral palsy is a group of neurological disorders that affect movement
, muscle tone, and coordination. While some symptoms, such as poor muscle coordination and an
unsteady gait, overlap with the scenario, the sudden loss of acquired skills, stereotypic hand move
ments, and decelerated head circumference growth are more characteristic of Rett syndrome.
Page 15
202
Previous Year Questions
1. Which of the subsequent diseases is solely observed in females?
A. Rett syndrome
B. Asperger syndrome
C. Selective mutism
D. Atypical autism
----------------------------------------
2. In which condition is La belle indifference observed?
A. Conversion disorder
B. Panic disorder
C. Schizophrenia
D. Depression
----------------------------------------
3. Which of the following is a new drug approved for Rett syndrome?
A. Trofinetide
B. Ambien
C. Lunesta
D. GHB opioids
----------------------------------------
4. According to the revision of DSM-5 by the American Psychiatric Association, what is the current term
used to refer to 'mental retardation'?
A. Feeble mindedness
B. Mental subnormality
C. Intellectual disability
D. Intellectually challenged
----------------------------------------
5. What is the previous name for minimal brain dysfunction among these disorders?
A. Dyslexia
B. Attention deficit/hyperactivity disorder
C. Mental subnormality
D. Oligophrenia
----------------------------------------
6. The parents of a 13-year-old boy have sought the assistance of a child psychiatrist due to their
concerns. They have observed that the child frequently engages in dishonesty both at home and in
school, displays disruptive behavior towards his teacher, often intimidates younger children, and has
been caught stealing money multiple times. What is the probable diagnosis for this child?
203
A. Oppositional defiant disorder
B. Conduct disorder
C. Antisocial personality disorder
D. Disruptive mood dysregulation
----------------------------------------
7. What could be the diagnosis for a child who exhibits below-average mathematical skills despite
having normal IQ and age-appropriate learning abilities, and remains unaffected by a year-long private
mathematics tutoring?
A. ADHD
B. Mathematics learning disorder
C. Aspergers syndrome
D. Specific learning disorder
----------------------------------------
Correct Answers
Question Correct Answer
Question 1 1
Question 2 1
Question 3 1
Question 4 3
Question 5 2
Question 6 2
Question 7 4
Page 2
204
Option D - Atypical autism: Impaired development becomes evident by 3 years or after 3 years. Th
ere are abnormalities in social interaction and restricted, repetitive patterns of behavior or activities.
It is common among both men and women. But women with atypical autism exhibit different sympt
om patterns, making diagnosis extremely challenging.
Page 3
205
Incorrect Options:
Option B: Ambien: Ambien (generic name: zolpidem) is a
sedative medication primarily used for the treatment of insomnia.
Option C: Lunesta: Lunesta (generic name: eszopiclone) is another sedative medication used for th
e treatment of insomnia. Similar to Ambien,
Option D: GHB opioids: GHB (gamma-hydroxybutyrate) and opioids. GHB is a central nervous syst
em depressant that has been used recreationally and has sedative effects. Opioids, on the other h
and, are a class of medications used for pain relief. Neither GHB nor opioids are approved drugs s
pecifically indicated for the treatment of Rett syndrome.
Page 4
206
Solution for Question 6:
Correct Option: B
The above case is that of a conduct disorder.
Patterns of behavior characterized by aggression and violation of the rights of others, such as:
• Physical aggression, a threat to harm, and bullying others.
• Destruction of property, theft, and harming animals.
• Frequent violation of age-appropriate roles.
• Staying out at night despite parent prohibition.
Incorrect Choices:
Option A: In oppositional defiant disorder, they do not resort to physically aggressive or disruptive b
ehavior.
Option C: In this case, the individual has no regard for others and is violent and aggressive.
Option D: Severe anger outbursts are not seen therefore, this is not the correct answer.
Page 5
207
essions.
Option C: Syndrome of Asperger's: For the most part, youngsters and adolescents with Asperger's
Condition can talk with others and can perform genuinely well in their school work. However, they s
truggle to comprehend social situations and subtle forms of communication, such as humor, sarcas
m, and body language. They might also only want to participate in a
limited number of activities or think and talk a lot about a single subject or interest.
Page 6
208
Organic Mental Disorder
1. Which of the following statements best describes a key difference between delirium and dementia?
A. Delirium has a short onset, while dementia has a gradual and insidious onset. `
B. Delirium is characterized by impaired consciousness, while dementia is associated with intact
consciousness.
C. Symptoms of delirium fluctuate, whereas symptoms of dementia remain stable over time.
D. All of the above
----------------------------------------
2. A 68-year-old male patient is admitted to the hospital with complaints of sudden onset confusion,
disorientation, and fluctuating levels of consciousness. His family reports that he was previously alert
and oriented. The patient appears restless and exhibits inattention during the examination. The
healthcare provider notices diffuse slowing of background activity on the patient's EEG. What is the
likely diagnosis of this patient, and what would be the appropriate treatment choice?
(or)
What is the likely diagnosis and appropriate treatment for a 68-year-old male patient with sudden onset
confusion, disorientation, fluctuating consciousness, restlessness, inattention, and diffuse slowing of
background activity on EEG?
A. Delirium; Antipsychotics
B. Dementia; Benzodiazepines
C. Delirium; Acetylcholinesterase inhibitors
D. Dementia; Acetylcholinesterase inhibitors
----------------------------------------
3. What is the most likely diagnosis for a 70-year-old male patient with memory loss, difficulty in
completing daily tasks, and word-finding difficulties, while remaining fully conscious during
examination?
(or)
A 70-year-old male patient presents to the clinic with complaints of memory loss and difficulty in
completing daily tasks. He has noticed a gradual decline in his ability to remember recent events, and
he often struggles to find the right words during conversations. On examination, the patient appears to
be fully conscious. Which of the following is the most likely diagnosis for this patient?
A. Delirium
B. Depression
C. Dementia
D. Anxiety disorder
----------------------------------------
4. A 72-year-old patient presents with a progressive decline in memory and language skills over the
past year. The family reports that the patient often forgets recent events and struggles to find the right
words during conversations. On examination, the patient appears unaware of his memory deficits.
There are no motor abnormalities observed. Based on the provided information, what is the most likely
diagnosis?
209
(or)
What is the most likely diagnosis for a 72-year-old patient with progressive decline in memory and
language skills, unawareness of memory deficits, and no motor abnormalities?
A. Alzheimer's disease
B. Vascular dementia
C. Lewy body dementia
D. Parkinson's disease
----------------------------------------
5. Which of the following factors is implicated in the self-aggregation of amyloid beta (Aβ) protein,
leading to synaptotoxicity in Alzheimer's disease?
A. Mutation in the APP gene
B. Decreased metabolism of APP by beta-secretase
C. Mutation of PS1 and PS2 linked with beta-secretase
D. None of the above
----------------------------------------
6. A 65-year-old male presents to the clinic with complaints of memory problems and difficulty with
concentration. He has a history of hypertension and high cholesterol. On examination, focal
neurological signs are observed. Which type of dementia is most likely present in this patient?
(or)
What type of dementia is most likely present in a 65-year-old male with memory problems, difficulty with
concentration, a history of hypertension and high cholesterol, and focal neurological signs observed on
examination?
A. Alzheimer's disease
B. Vascular dementia
C. Parkinson's disease dementia
D. Lewy body dementia
----------------------------------------
7. Which variant of frontotemporal dementia (Pick disease) is most likely in a 60-year-old female with
difficulty in recognising familiar faces and objects, and has normal verbal output (fluency), and atrophy
in the temporal lobe observed on imaging studies?
(or)
A 60-year-old female presents with difficulty in recognising familiar faces and objects, and has normal
verbal output (fluency). There are no significant changes in behavior or personality. Imaging studies
reveal atrophy in the temporal lobe. Which variant of frontotemporal dementia (Pick disease) is most
likely?
A. Behavioral variant
B. Semantic variant
C. Progressive non-fluent aphasia
D. Posterior cortical atrophy
Page 2
210
----------------------------------------
8. A 75-year-old female presents to the clinic with a history of progressive cognitive decline. Her family
reports that she frequently experiences visual hallucinations and has been having difficulty with balance
and coordination. On examination, she demonstrates fluctuating cognition and motor features of
parkinsonism. Additionally, her family mentions that she thrashes around during sleep. Which of the
following is the most likely diagnosis for this patient?
(or)
What is the most likely diagnosis for a 75-year-old female with progressive cognitive decline, visual
hallucinations, difficulty with balance and coordination, fluctuating cognition, thrashing around during
sleep and parkinsonism features on examination?
A. Alzheimer's disease
B. Vascular dementia
C. Lewy body disease
D. Frontotemporal dementia
----------------------------------------
9. A 42-year-old male with a known diagnosis of HIV infection presents to the clinic with complaints of
cognitive difficulties, motor abnormalities, and personality changes. Laboratory tests confirm the
presence of systematic HIV infection. Based on the clinical presentation and diagnostic criteria, what is
the most likely diagnosis for this patient?
(or)
What is the most likely diagnosis for a 42-year-old male with HIV infection, presenting with cognitive
difficulties, motor abnormalities, and personality changes, and confirmed systemic HIV infection on
laboratory tests?
A. HIV-associated psychosis
B. AIDS dementia complex
C. HIV-associated neurocognitive disorder
D. HIV encephalopathy
----------------------------------------
10. A 45-year-old former professional boxer presents to the clinic with complaints of emotional lability,
dysarthria, and impulsivity. He reports a history of repeated head trauma over many years during his
boxing career. Based on the clinical presentation and history of head trauma, what is the most likely
diagnosis for this patient?
(or)
What is the most likely diagnosis for a 45-year-old former professional boxer presenting with emotional
lability, dysarthria, and impulsivity, and a history of repeated head trauma during his boxing career?
A. Alzheimer's disease
B. Parkinson's disease
C. Dementia pugilistica
D. Vascular dementia
----------------------------------------
Page 3
211
11. What is the most likely cognitive screening tool which includes an assessment of orientation,
registration, recall, concentration, and language and what is the cutoff score for the diagnosis of the
dementia?
(or)
A 65-year-old patient presents to the clinic with progressive memory loss, difficulty with language, and
impaired judgment. A screening tool was administered to assess the patient's cognitive function. The
screening tool included assessment of orientation, registration, recall, concentration, and language.
Based on the provided clinical features and the screening conducted, what is the most likely screening
tool used and what is the score required for the diagnosis of the patient's condition?
A. Mini-Mental State Examination (MMSE), score less than 24
B. Montreal Cognitive Assessment (MoCA), score less than 28
C. Clock Drawing Test (CDT), score less than 3
D. Clinical Dementia Rating (CDR), score of 1 or above
----------------------------------------
12. A 75-year-old patient with moderate Alzheimer's disease was prescribed an NMDA antagonist for
cognitive symptoms. Which of the following statements is incorrect about the prescribed drug?
(or)
Which of the following statements is incorrect about the NMDA antagonist?
A. It is used in moderate to severe Alzheimer's disease.
B. The prescribed drug is Donepezil.
C. It is used both as monotherapy and in combination with other medications.
D. It is effective for cognitive symptoms in other dementias like Lewy body disease and vascular
dementia.
----------------------------------------
13. Which of the following is a key difference between delirium and schizophrenia?
A. Consciousness is intact in schizophrenia while it is impaired in delirium.
B. Delusions and hallucinations are transient and less organized in schizophrenia , while they are
constant and better organized in delirium.
C. Schizophrenia has a sudden onset of acute psychosis, while delirium is not associated with such
sudden onset.
D. Schizophrenia may be associated with fever, while fever is not commonly observed in delirium.
----------------------------------------
14. Which of the following is cortical dementia?
A. Alzheimer's disease
B. Vascular dementia
C. Huntington's disease
D. Parkinson's disease
----------------------------------------
Page 4
212
15. A 70-year-old individual presents with memory loss, difficulty finding words, and gradually
worsening cognitive impairment. Family history reveals a relative with a similar condition. Which genetic
factor is commonly associated with late-onset Alzheimer's disease in this scenario?
(or)
Which genetic factor is commonly associated with late-onset Alzheimer's disease?
A. APOE4
B. Presenilin-1 mutation
C. Presenilin-2 mutation
D. APP gene mutation
----------------------------------------
16. Which neurotransmitter is primarily decreased in Alzheimer's disease?
A. Acetylcholine
B. Norepinephrine
C. Serotonin
D. Glutamate
----------------------------------------
17. Which of the following is not a part of the clinical presentation seen in normal pressure
hydrocephalus?
A. Dementia
B. Urinary incontinence
C. Ataxia
D. Delirium
----------------------------------------
18. A 70-year-old male presents with progressive memory loss, difficulty with daily tasks, and changes
in behavior. A screening test reveals cognitive impairment. After further evaluation, the physician
prescribes a medication that is a human IgG1 monoclonal antibody given as an IV infusion. What is the
drug prescribed to the patient?
(or)
Which drug is prescribed for cognitive impairment in a 70-year-old male, which is a human IgG1
monoclonal antibody administered via IV infusion?
A. Donepezil
B. Rivastigmine
C. Galantamine
D. Aducanumab
----------------------------------------
19. A 45-year-old patient is brought to the psychiatry outpatient department due to recent changes in
behavior. The individual has been engaging in socially inappropriate actions, such as making loud and
inappropriate comments in public settings. Family members report instances of impulsive
decision-making, such as overspending and risky behaviors. During the assessment, the patient
Page 5
213
appears unaware of the consequences of these actions and struggles to recognize the impact on
personal and social relationships. Based on these clinical findings, which syndrome is most likely
associated with the observed behavior changes?
(or)
A 45-year-old patient presents to the psychiatry OPD with disinhibited behavior, impulsiveness, limited
insight, and poor judgment. Which syndrome is most likely associated with these clinical findings?
A. Orbitofrontal syndrome
B. Dorsolateral syndrome
C. Medial frontal syndrome
D. Occipitoparietal syndrome
----------------------------------------
20. What is the most likely condition causing sudden violent acts and emotional outbursts with
interrupted consciousness in a 32-year-old patient with epilepsy?
(or)
A 32-year-old patient with a history of epilepsy presents to the emergency department with episodes of
sudden violent acts and emotional outbursts, during which they appear to be in a state of interrupted
consciousness. These episodes have an abrupt onset and end. What is the most likely condition
causing these symptoms?
A. Schizophrenia
B. Bipolar disorder
C. Twilight state
D. Oneiroid state
----------------------------------------
21. What is the most likely diagnosis for a 78-year-old male with COPD, UTI, acute confusion,
hallucinations, and aimless picking movements on bedsheets and clothes?
(or)
A 78-year-old male with a history of chronic obstructive pulmonary disease (COPD) and recent urinary
tract infection (UTI) is admitted to the hospital. Over the past two days, he has developed acute
confusion, disorientation, and hallucinations. His symptoms worsen in the evening and he becomes
agitated. During the physical examination, the patient is observed to engage in aimless picking
movements on his bedsheets and clothes. What is the most likely diagnosis for this patient's
presentation?
A. Delirium
B. Dementia
C. Depression
D. Psychosis
----------------------------------------
22. Which neurotransmitter is typically decreased in delirium?
A. Acetylcholine
B. Serotonin
Page 6
214
C. Dopamine
D. GABA
----------------------------------------
Correct Answers
Question Correct Answer
Question 1 4
Question 2 1
Question 3 3
Question 4 1
Question 5 1
Question 6 2
Question 7 2
Question 8 3
Question 9 2
Question 10 3
Question 11 1
Question 12 2
Question 13 1
Question 14 1
Question 15 1
Question 16 1
Question 17 4
Question 18 4
Question 19 1
Question 20 3
Question 21 1
Question 22 1
Page 7
215
Solution for Question 2:
Correct Option A - Delirium; Antipsychotics:
• Delirium is characterized by acute onset confusion, disorientation, and fluctuating levels of
consciousness. Antipsychotic medications, such as haloperidol or olanzapine, are commonly used
to manage delirium symptoms, including delusions, hallucinations, and agitation.
Incorrect Options:
Option B - Dementia; Benzodiazepines: This option is incorrect. Dementia is characterized by grad
ual and insidious onset cognitive decline, not sudden onset confusion and fluctuating levels of cons
ciousness. Benzodiazepines are not the treatment of choice for dementia and may even worsen co
gnitive function and increase the risk of falls and other adverse effects.
Option C
- Delirium; Acetylcholinesterase inhibitors: Pharmacotherapy, specifically antipsychotics, is
necessary to manage the symptoms of delirium effectively and not acetylcholinesterase inhibitors
Option D - Dementia; Acetylcholinesterase inhibitors: This option is incorrect. Dementia is characte
rized by gradual and insidious cognitive decline, whereas the clinical scenario describes a sudden
onset of confusion and fluctuating consciousness. Acetylcholinesterase inhibitors, such as donepe
zil or rivastigmine, are commonly used in the treatment of dementia to improve cognitive function, b
ut they are not the appropriate choice for managing delirium.
Page 8
216
Option D- Anxiety disorder: Anxiety disorders are characterized by excessive and persistent worry
or fear that is out of proportion to the actual threat. While anxiety can cause cognitive symptoms su
ch as difficulty concentrating or racing thoughts, it generally does not lead to significant cognitive i
mpairment. In the given scenario, the patient's symptoms of memory loss and difficulty in completin
g tasks are not consistent with the primary features of an anxiety disorder.
Incorrect Options:
Option B - Vascular dementia: Vascular dementia is characterized by cognitive impairment caused
by cerebrovascular disease, typically resulting from stroke or reduced blood flow to the brain. It ma
y present with a sudden onset of symptoms rather than a
gradual decline seen in Alzheimer's disease.
Option C - Lewy body dementia: Lewy body dementia is characterized by cognitive decline, visual
hallucinations, and fluctuating attention. Parkinsonism features such as tremors and rigidity are oft
en present. In the given scenario, there are no motor abnormalities mentioned, making Lewy body
dementia less likely.
Option D - Parkinson's disease: Parkinson's disease primarily presents with motor symptoms such
as tremors, rigidity, and bradykinesia. Cognitive impairment, including memory deficits and languag
e disturbance, can occur in later stages of Parkinson's disease, but it is not the primary feature me
ntioned in the scenario.
Incorrect Options:
Option B: Decreased metabolism of APP by beta-secretase: Beta secretase increase the metabolis
m of APP rather than decreasing it.
Option C: Mutation of PS1 and PS2 linked with Beta secretase: Mutation of presenilin-1 (PS1) and
presenilin-2 (PS2), which are components of the gamma-secretase complex, is linked with aberran
t processing of APP and the accumulation of Ab, leading to self-aggregation and synaptotoxicity.
Option D: None of the above: The correct choice is A, indicating that not all of the options are wron
g.
Page 9
217
Solution for Question 6:
Correct Option B - Vascular dementia:
• Based on the clinical scenario provided, the patient is a 65-year-old male with a history of
hypertension and high cholesterol, presenting with memory problems, concentration difficulties,
and focal neurological signs. These findings are suggestive of vascular dementia.
Incorrect Options:
Option A
- Alzheimer's disease: This is the most common type of dementia but is not associated with a step l
adder pattern or focal neurological signs. It typically presents with gradual memory loss and other c
ognitive impairments.
Option C - Parkinson's disease dementia: Parkinson's disease primarily affects movement and is a
ssociated with motor symptoms such as tremors and rigidity. Dementia may develop in later stages
but is not typically associated with focal neurological signs.
Option D - Lewy body dementia: This type of dementia is characterized by the presence of Lewy b
odies, abnormal protein deposits in the brain. It is associated with fluctuations in cognition, visual h
allucinations, and parkinsonism. While focal neurological signs can be present, they are not the pri
mary feature of Lewy body dementia.
Incorrect Options:
Option A - Behavioral variant: The behavioral variant of frontotemporal dementia is characterized b
y changes in behavior and personality, such as disinhibition, apathy, and hyperorality. It is not cons
istent with the described clinical presentation.
Option C - Progressive non-fluent aphasia: Progressive non-fluent aphasia is a variant of frontotem
poral dementia characterized by difficulties in speech production and grammar. It is not consistent
with the described clinical presentation.
Option D - Posterior cortical atrophy: Posterior cortical atrophy is associated with degeneration of t
he posterior regions of the brain, primarily affecting the occipital and parietal lobes. It is not consist
ent with the described clinical presentation.
Page 10
218
Solution for Question 8:
Correct Option C - Lewy body disease :
• The clinical scenario described is suggestive of Lewy body disease, also known as Dementia with
Lewy bodies. This condition is characterized by progressive cognitive decline, fluctuating cognition,
visual hallucinations, and motor features of parkinsonism.
• Key features of REM sleep behavior disorder in the Lewy body dementia include: Dream
"Enactment": Individuals with REM sleep behavior disorder in Lewy body dementia may physically
act out the content of their dreams. This can involve complex and sometimes violent movements
during REM sleep. Loss of Muscle Atonia: Normally, the muscles are atonic (paralyzed) during
REM sleep to prevent physical movements associated with dreaming. In REM sleep behavior
disorder, this paralysis is impaired.
Key features of REM sleep behavior disorder in the Lewy body dementia include:
Dream "Enactment": Individuals with REM sleep behavior disorder in Lewy body dementia may ph
ysically act out the content of their dreams. This can involve complex and sometimes violent move
ments during REM sleep.
Loss of Muscle Atonia: Normally, the muscles are atonic (paralyzed) during REM sleep to prevent
physical movements associated with dreaming. In REM sleep behavior disorder, this paralysis is i
mpaired.
Incorrect Options:
Option A -Alzheimer's disease: While Alzheimer's disease is a common cause of dementia, it typic
ally presents with prominent memory impairment, which is not the case in this patient. Additionally,
visual hallucinations and motor features of parkinsonism are less commonly seen in Alzheimer's di
sease.
Option B - Vascular dementia: Vascular dementia presents with step-wise progression of cognitive
decline, focal neurological deficits, and evidence of cerebrovascular lesions on imaging, the most li
kely diagnosis for this patient is vascular dementia. The poorly controlled vascular risk factors, suc
h as hypertension and diabetes, contribute to the vascular pathology leading to the cognitive and n
eurological symptoms. However, the presence of visual hallucinations and motor features of parkin
sonism in this patient suggests a different diagnosis.
Option D - Frontotemporal dementia: Frontotemporal dementia (FTD) is a group of rare brain disor
ders that primarily affect behavior, personality, language, and executive functions.
Incorrect Options:
Option A - HIV-associated psychosis: HIV infection can sometimes lead to psychiatric manifestatio
ns, including psychosis. However, the clinical presentation described in the scenario, such as cogni
Page 11
219
tive difficulties and motor abnormalities, is more indicative of AIDS dementia complex rather than is
olated psychosis.
Option C - HIV-associated neurocognitive disorder: This term is used to describe a spectrum of co
gnitive impairments associated with HIV infection, ranging from mild to severe. However, in the giv
en scenario, the patient's clinical presentation with motor abnormalities and personality changes su
ggests a more advanced form of cognitive impairment seen in AIDS dementia complex.
Option D - HIV encephalopathy: HIV encephalopathy refers to the direct effects of HIV infection on
the brain, leading to cognitive and neurological dysfunction. While it shares some similarities with A
IDS dementia complex, the term ADC is specifically used to describe the clinical syndrome seen in
advanced HIV infection, whereas HIV encephalopathy is a
broader term encompassing various HIV-related brain abnormalities.
Incorrect Options:
Option A - Alzheimer's disease: Alzheimer's disease presents with progressive memory loss, cogni
tive decline affecting executive function and language skills, disorientation, impaired judgment, per
sonality changes, and loss of initiative. Individuals may experience mood swings, agitation, and diff
iculty learning new information. Motor function can be affected in advanced stages, and the diseas
e is characterized by a gradual decline in overall cognitive and functional abilities. Early diagnosis i
s essential for effective management. The clinical presentation in this patient, along with the history
of head trauma, suggests a different diagnosis.
Option B - Parkinson's disease: Parkinson's disease primarily presents with motor symptoms such
as tremors, rigidity, and bradykinesia. While cognitive impairment can occur in advanced stages, th
e prominent features in this patient are emotional lability, dysarthria, and impulsivity, which are not
typically associated with Parkinson's disease.
Option D - Vascular dementia: Vascular dementia presents with step-wise progression of cognitive
decline, focal neurological deficits, and evidence of cerebrovascular lesions on imaging, the most li
kely diagnosis for this patient is vascular dementia. The poorly controlled vascular risk factors, suc
h as hypertension and diabetes, contribute to the vascular pathology leading to the cognitive and n
eurological symptoms. However, the specific clinical features and history of head trauma described
in this patient are more indicative of head trauma-related dementia.
Page 12
220
Incorrect Options:
Option B - Montreal Cognitive Assessment (MoCA), score less than 28: MoCA typically assesses c
ognitive impairment emphasizing on cognitive domains (Broad range, including detailed assessme
nts of executive functions, visuospatial abilities, and language), not on (Orientation, registration, att
ention, calculation, recall, language). (MMSE :
Score <24/30; MoCA<26/30 is suggestive of cognitive impairment)
Option C - Clock Drawing Test (CDT), score less than 3: The Clock Drawing Test (CDT), assesses
visuospatial abilities and executive functions, which are not specifically mentioned in the scenario.
Option D - Clinical Dementia Rating (CDR), score of 1
or above: The Clinical Dementia Rating (CDR), is a staging system rather than a
screening tool and is not typically used for initial cognitive assessment.
Incorrect Options:
Option A - It is used in moderate to severe Alzheimer's disease: This statement is correct based on
the information provided.
Option C - It is used both as monotherapy and in combination with other medications: This stateme
nt is correct based on the information provided. NMDA antagonists, including Memantine, can be u
sed as monotherapy or in combination with other medications for the treatment of moderate to sev
ere Alzheimer's disease.
Option D - It is effective for cognitive symptoms in other dementias like Lewy body disease and vas
cular dementia: This statement is correct based on the information provided. NMDA antagonists, in
cluding Memantine, can be used for cognitive symptoms in other dementias such as Lewy body dis
ease and vascular dementia.
Page 13
221
Option B. Delusions and hallucinations are transient and less organized in schizophrenia, while the
y are constant and better organized in delirium: In schizophrenia, delusions and hallucinations can
be persistent and highly organized. While the content and nature of delusions and hallucinations c
an vary, they are not necessarily transient or less organized in schizophrenia.
Option C. Schizophrenia has a
sudden onset of acute psychosis, while delirium is not associated with such sudden onset: Schizo
phrenia typically has an insidious onset, with symptoms developing gradually over time. Delirium, o
n the other hand, often has a relatively sudden onset, often triggered by an underlying medical con
dition, medication, or substance use.
Option D. Schizophrenia may be associated with fever, while fever is not commonly observed in de
lirium: Schizophrenia is not typically associated with fever. Fever is more likely to be associated wit
h infectious or inflammatory conditions. In contrast, delirium may indeed be associated with fever,
especially if it is caused by an underlying medical illness or infection.
Incorrect Options:
Option B - Vascular dementia: Vascular dementia, on the other hand, is caused by impaired blood f
low to the brain, often resulting from stroke or small vessel disease. It may also affect both cortical
and subcortical regions of the brain.
Option C
- Huntington's disease: Huntington's disease primarily affects the basal ganglia, which is a subcorti
cal structure involved in motor control. It leads to motor symptoms, cognitive decline, and psychiatri
c disturbances.
Option D - Parkinson's disease: Parkinson's disease is also a subcortical dementia that affects the
basal ganglia and presents with motor symptoms, such as tremors, rigidity, and bradykinesia. Altho
ugh Parkinson's disease can have associated cognitive impairments, it is primarily classified as a
movement disorder rather than a primary cortical dementia.
Incorrect Options:
Page 14
222
Option B - Presenilin-1 mutation: Presenilin-1 mutations are more commonly associated with early-
onset familial Alzheimer's disease.
Option C - Presenilin-2 mutation: Presenilin-2 mutations are also more commonly associated with
early-onset familial Alzheimer's disease.
Option D - APP gene mutation: Mutations in the Amyloid Precursor Protein (APP) gene are rare an
d typically associated with early-onset familial Alzheimer's disease.
Page 15
223
These three symptoms (dementia, urinary incontinence, and ataxia) are collectively known as the c
lassic triad of NPH. They are considered characteristic clinical features of the condition.
Incorrect Options:
Option A - Donepezil: Donepezil is a
cholinesterase inhibitor used in the treatment of Alzheimer's disease but not a
monoclonal antibody.
Option B - Rivastigmine: Rivastigmine is also a
cholinesterase inhibitor used in the treatment of Alzheimer's disease but not a
monoclonal antibody.
Option C - Galantamine: Galantamine is another cholinesterase inhibitor used in the treatment of A
lzheimer's disease but not a monoclonal antibody.
Incorrect Options:
Option B - Dorsolateral syndrome: This syndrome is associated with deficits in recall of previously l
earned information, impaired planning, and impaired attention and concentration.
Option C - Medial frontal syndrome: This syndrome is characterized by apathy, poor motivation, de
creased interest, and decreased emotional concern. While some symptoms of depression are men
tioned in the scenario, the presence of disinhibited behavior, impulsiveness, and limited insight poi
nts more toward orbitofrontal syndrome.
Option D
- Occipitoparietal syndrome:Occipitoparietal syndrome, also known as Gerstmann syndrome, is a
rare neurological disorder that is characterized by a specific set of symptoms related to brain dama
ge or injury to the left hemisphere of the brain, particularly the angular gyrus, which is located in th
e parietal lobe.
Page 16
224
Solution for Question 20:
Correct Option C - Twilight state:
• The given scenario describes a patient experiencing episodes of interrupted consciousness,
abrupt onset and end, and unexpected violent acts/emotional outbursts. These symptoms are
indicative of a twilight state. A twilight state refers to a condition where there is a temporary
interruption of continuity of consciousness. It is often seen in individuals with underlying medical
conditions such as epilepsy or brain tumors.
• Schizophrenia and bipolar disorder do not typically present with episodes of interrupted
consciousness.
Incorrect choices:
Option A. Schizophrenia: Schizophrenia is a chronic mental disorder characterized by symptoms s
uch as hallucinations, delusions, disorganized thinking, and impaired cognitive and emotional functi
oning. It does not typically present with abrupt episodes of violent behavior and emotional outburst
s as described in the case.
Option B. Bipolar disorder: Bipolar disorder involves mood swings between episodes of depression
and mania. While individuals with bipolar disorder may experience emotional outbursts, it doesn't t
ypically manifest as sudden, impulsive, violent acts with an altered state of consciousness.
OptionD. Oneiroid state: Oneiroid state, also known as "dreamy state" or "dream-like state," is ass
ociated with a detachment from reality, and individuals in this state may experience vivid and unus
ual dream-like perceptions. However, this state is not typically associated with the abrupt episodes
of violent behavior and emotional outbursts described in the case.
Incorrect Options:
Option B - Dementia: Dementia is characterized by a
progressive decline in cognitive function, but it typically has a more gradual onset and does not ma
nifest with acute changes in mental status, sundowning, or floccillation.
Option C - Depression: Depression can cause cognitive impairments, but it is primarily characterize
d by persistent sadness, loss of interest, and other mood-related symptoms. It does not typically pr
esent with acute confusion, hallucinations, or picking movements.
Option D - Psychosis: Psychosis involves a loss of contact with reality, often accompanied by hallu
cinations and delusions. However, the acute onset, fluctuating course, sundowning, and floccillatio
n described in this scenario are more indicative of delirium.
Page 17
225
Solution for Question 22:
Correct Option A - Acetylcholine:
• Delirium is a state of acute confusion and altered consciousness characterized by decrease in the
activity of the neurotransmitter acetylcholine. Reduced levels of acetylcholine in the brain are
thought to contribute to the cognitive impairments and disorientation observed in delirium.
Delirium is a state of acute confusion and altered consciousness characterized by decrease in the
activity of the neurotransmitter acetylcholine. Reduced levels of acetylcholine in the brain are thou
ght to contribute to the cognitive impairments and disorientation observed in delirium.
Incorrect choices:
b. Serotonin: While serotonin is involved in mood regulation and has effects on various cognitive pr
ocesses, it is not typically associated with delirium. A
decrease in serotonin is more commonly linked to mood disorders like depression.
c. Dopamine: Dopamine is a neurotransmitter associated with a range of functions, including move
ment, reward, and motivation. Changes in dopamine activity are more commonly associated with c
onditions like Parkinson's disease, schizophrenia, and addiction. While delirium can manifest with s
ymptoms that might be confused with psychosis, dopamine is not typically decreased in delirium.
d. GABA (Gamma-Aminobutyric Acid): GABA is the primary inhibitory neurotransmitter in the brain.
Changes in GABA levels can be associated with anxiety and sleep disorders, but it is not typically i
mplicated as a primary factor in delirium. Delirium is more often associated with imbalances in acet
ylcholine and other neurotransmitters related to cognitive function.
Page 18
226
Previous Year Questions
1. Which mental functions are examined in the Mental State Examination (100-7)?
A. Judgment
B. Language
C. Orientation
D. Concentration
----------------------------------------
2. Confusion assessment test is used in:
A. Delirium
B. Dementia
C. Schizophrenia
D. Depression
----------------------------------------
3. Which of the following is not considered a possible factor contributing to SIADH?
A. Demeclocycline
B. Vincristine
C. Carbamazepine
D. Nicotine
----------------------------------------
4. Which of the following characteristics provides stronger evidence in support of delirium?
A. Occurs gradually over a period of time
B. Fluctuating course
C. Preserved consciousness
D. Commonly associated with auditory hallucinations
----------------------------------------
5. What is the purpose of utilizing the confusion assessment test?
A. Dementia
B. Depression
C. Delirium
D. Schizophrenia
----------------------------------------
6. Which of the following statements is false regarding Alzheimer's disease?
A. Intracellular neurofibrillary tangles are seen
B. Neuritic plaques made of beta amyloid are found
C. Nucleus of Meynert is not affected
227
D. Short term memory is affected
----------------------------------------
7. True for lithium:-
A. Delayed absorption
B. Narrow therapeutic index
C. Protein-bound
D. Can be given safely in renal dysfunction
----------------------------------------
8. In the physiology viva, the examiner inquires about the individual who initially recorded the first EEG.
You respond by admitting that you are unaware of the answer. Sympathetic to your lack of knowledge,
the examiner provides a hint indicating that the name resembles a common item found at McDonald's.
With this clue, you are able to recall the name and provide the correct response. Which form of memory
is involved in this situation?
A. Semantic memory
B. Implicit memory
C. Associative learning
D. Priming
----------------------------------------
9. Which one of the options below is not classified as a form of subcortical dementia?
A. Pick’s disease
B. Parkinson’s disease
C. Huntington’s disease
D. Wilson’s disease
----------------------------------------
10. What should be the treatment approach for a 7-year-old child who has been experiencing
bed-wetting about twice a week for the past year, following a thorough investigation to exclude any
underlying organic cause?
A. Pharmacotherapy with imipramine
B. Psychodynamic psychotherapy
C. Bladder training, with reward for delaying micturition during daytime
D. Bell and pad based classification conditioning
----------------------------------------
11. What are the distinguishing characteristics between delirium and dementia in Alzheimer's disease?
A. Acuity of onset and agitation
B. Acuity of onset and loss of consciousness
C. Visual hallucinations and impaired memory
D. Agitation and irritation
Page 2
228
----------------------------------------
Correct Answers
Question Correct Answer
Question 1 4
Question 2 1
Question 3 1
Question 4 2
Question 5 3
Question 6 3
Question 7 2
Question 8 4
Question 9 1
Question 10 4
Question 11 2
Page 3
229
• Confusion Assessment Test (CAM) is primarily used for the assessment and diagnosis of
delirium. Delirium is a state of acute confusion and changes in cognition that often occurs in older
adults, particularly those who are hospitalized or have underlying medical conditions. The CAM is a
validated screening tool that helps healthcare professionals identify the presence of delirium and
distinguish it from other cognitive disorders or psychiatric conditions.
Incorrect options:
Option B. Dementia: While both delirium and dementia involve changes in cognition, they are distin
ct conditions. Dementia is a chronic and progressive disorder characterized by a decline in memor
y, thinking, and reasoning skills. The CAM is not typically used for diagnosing dementia.
Option C. Schizophrenia: Schizophrenia is a psychiatric disorder characterized by disturbances in
perception, thought processes, and emotions. It does not typically present with acute confusion or
cognitive impairment, which are hallmarks of delirium. The CAM is not used for assessing schizoph
renia.
Option D. Depression: Depression is a mood disorder that primarily affects a person's emotional st
ate, such as persistent sadness, loss of interest, and changes in appetite or sleep patterns. While d
epression can sometimes lead to cognitive impairments, the CAM is not specifically designed for a
ssessing depression or its cognitive symptoms.
Page 4
230
chronic cognitive disorder rather than delirium.
Option C: Preserved consciousness- Delirium is associated with an impaired level of consciousnes
s. However, it is important to note that consciousness in delirium can vary, ranging from hyperactiv
e (agitated and restless) to hypoactive (lethargic and withdrawn). While consciousness is impaired,
it is not preserved, making this option incorrect.
Option D: Commonly associated with auditory hallucinations- Delirium is more commonly associate
d with perceptual disturbances, such as visual hallucinations or illusions.
Page 5
231
Option B: Neuritic plaques made of beta-amyloid are found- Neuritic plaques, also known as senile
plaques, are another characteristic feature of Alzheimer's disease. These plaques consist of aggre
gates of beta-amyloid protein that accumulate between neurons in the brain. The presence of neuri
tic plaques is associated with inflammation and neuronal damage.
Option D: Short-term memory is affected- Short-term memory impairment is a hallmark symptom of
Alzheimer's disease. As the disease progresses, individuals may have difficulty retaining and recal
ling new information, experiencing deficits in short-term memory. This is often one of the initial cog
nitive symptoms noticed in Alzheimer's disease.
Page 6
232
more specific to the activation of related memories through cues or stimuli.
Page 7
233
Solution for Question 11:
Correct Option B:
Acuity of onset and loss of consciousness. Delirium and dementia are both cognitive disorders, but
they have distinct features that differentiate them. Delirium is characterized by a rapid onset of sy
mptoms, often within hours or days, and it is accompanied by fluctuations in attention, confusion, di
sorientation, and sometimes a reduced level of consciousness. On the other hand, dementia is a
progressive condition with a gradual decline in cognitive function, primarily affecting memory, langu
age, and problem-solving abilities. In dementia, there is usually no significant impairment of consci
ousness.
Incorrect Options:
Option A: Acuity of onset and agitation: While delirium can involve agitation, it is not a
specific feature that distinguishes it from dementia. Agitation can be present in both conditions.
Option C: Visual hallucinations and impaired memory: Visual hallucinations can occur in certain typ
es of dementia, such as dementia with Lewy bodies, but they are not exclusive to dementia and m
ay also be present in delirium.
Option D: Agitation and irritation: Agitation and irritation can be seen in both delirium and dementia.
These symptoms are not specific enough to differentiate between the two conditions.
Page 8
234
Sleep Disorders
1. What is the typical latency period of REM (Rapid Eye Movement) sleep?
A. 15 minutes
B. 30 minutes
C. 60 minutes
D. 90 minutes
----------------------------------------
2. A 35-year-old patient undergoes an EEG examination with their eyes closed. The EEG waveform
obtained during this recording is displayed below:
235
5. A 45-year-old patient visits a sleep clinic due to complaints of vivid dreaming and muscle paralysis
during sleep. The patient reports experiencing episodes of muscle twitches and complete immobility
while mentally awake. Polysomnography is performed to evaluate the sleep architecture. Which of the
following statements about the above mentioned and depicted sleep stage is incorrect?
A. The EEG activity shows increased beta activity similar to the awake state.
B. Sawtooth waves or Sawtooth appearance is a characteristic feature of the EEG in this sleep stage.
C. Rapid eye movements (REM) are observed during this sleep stage.
D. It involves large phasic potentials that originate from midbrain and go towards the medial geniculate
body and to the occipital spikes.
----------------------------------------
6. Which of the following statements regarding the differences between REM (Rapid Eye Movement)
and NREM (Non-Rapid Eye Movement) sleep is incorrect?
A. NREM sleep is characterized by lower brain and physiological activity compared to the awake state.
B. REM sleep is known as paradoxical sleep because the person is asleep, but the brain activity is
similar to the awake state.
C. Body movements are absent during REM sleep, whereas they are present during NREM sleep.
D. Dreams are remembered during NREM sleep, but not during REM sleep.
----------------------------------------
7. A 45-year-old individual is experiencing difficulty sleeping well at night. He frequently wakes up in the
middle of the night and has trouble falling back asleep. Additionally, he feels tired and unrefreshed
upon waking. There is no significant medical or psychiatric history. What type of insomnia is likely
affecting this individual?
(or)
What type of insomnia is likely present in a 45-year-old individual experiencing difficulty sleeping,
frequent awakenings, and feeling tired upon waking, without significant medical or psychiatric history?
A. Initial Insomnia
B. Middle Insomnia
C. Late Insomnia
D. Secondary insomnia
----------------------------------------
Page 2
236
8. A 45-year-old patient presents with an irresistible urge to move their legs when at rest or trying to fall
asleep. They describe an uncomfortable sensation in their legs, similar to insects crawling, which is
relieved by movement or walking around. The symptoms have been causing difficulty in initiating sleep.
Upon examination, the patient's ferritin levels are found to be within the normal range. Based on the
provided information, what is the most likely diagnosis for this patient, and which treatment would be
the most appropriate?
(or)
What is the most likely diagnosis and appropriate treatment for a 45-year-old patient with an irresistible
urge to move their legs when at rest and difficulty initiating sleep?
A. Restless Leg Syndrome (RLS) - Pramipexole
B. Restless Leg Syndrome (RLS) - Iron supplementation
C. Akathisia - Ropinirole
D. Akathisia – Rotigotine
----------------------------------------
9. A 45-year-old male presents with complaints of restless sleep, frequent awakenings, and tiredness
during the day. His partner reports noticing his legs jerking during sleep but he is not aware of it.. The
symptoms are predominantly observed during the first half of the night. Which of the following is the
most likely diagnosis?
(or)
What is the most likely diagnosis for a 45-year-old male who experiences restless sleep, frequent
awakenings, and leg jerking during sleep, mainly in the first half of the night?
A. Obstructive Sleep Apnea (OSA)
B. Restless Legs Syndrome (RLS)
C. Periodic Limb Movement Disorder (PLMD)
D. Narcolepsy
----------------------------------------
10. A 25-year-old patient presents with a history of sudden episodes of muscle weakness, often
triggered by emotional outbursts such as laughter. He also reports excessive daytime sleepiness and
has experienced hallucinations during the transition between wakefulness and sleep. The symptoms
have been present since his teenage years. What is the most appropriate treatment for this patient's
condition?
(or)
What is the most appropriate treatment for a 25-year-old patient with sudden episodes of muscle
weakness triggered by emotions, excessive daytime sleepiness, and hallucinations during the transition
between wakefulness and sleep, which have been present since his teenage years?
A. Antihistamines
B. Benzodiazepines
C. Modafinil
D. Selective serotonin reuptake inhibitors (SSRIs)
----------------------------------------
Page 3
237
11. A 16-year-old male presents with a history of recurrent episodes of excessive sleepiness, increased
appetite, heightened sexual drive, and occasional aggressive behaviors. These episodes typically last
for several days to weeks, during which the patient sleeps excessively, eats significantly more than
usual, and engages in inappropriate sexual behaviors. In between the episodes, the patient appears
normal and asymptomatic. What is the most likely diagnosis for this patient?
(or)
What is the most likely diagnosis for a 16-year-old male with recurrent episodes of excessive
sleepiness, increased appetite, heightened sexual drive, and occasional aggressive behaviors lasting
for several days to weeks, with asymptomatic periods in between?
A. Kleine-Levin Syndrome
B. Sleep apnea
C. Insomnia
D. Restless legs syndrome
----------------------------------------
12. What is the most appropriate treatment for a 10-year-old child experiencing sleepwalking episodes
at night, where he engages in activities while asleep and appears confused when awakened?
(or)
A 10-year-old male child is frequently found walking around the house during the night. The parents
report that the child appears to be asleep during these episodes and engages in activities such as
opening doors or going downstairs. When the child is awakened during these episodes, he appears
confused and occasionally reacts with aggression. The episodes have been occurring for several
months. What is the most appropriate treatment for this child's condition?
A. Reassurance
B. Antihistamines
C. Cognitive-behavioral therapy (CBT)
D. Benzodiazepines
----------------------------------------
13. What is the most appropriate term for a sleep disorder in a 9-year-old child who experiences
sudden arousals during sleep, accompanied by fearfulness, screaming, and panic symptoms, occurring
during the slow wave sleep stage and with no dream recollection upon awakening?
(or)
A 9-year-old child frequently experiences sudden arousals during sleep, accompanied by fearfulness,
screaming, and panic symptoms. When awakened, the child appears confused and has no recollection
of any dream. These episodes usually occur during the slow wave sleep stage. What is the most
appropriate term for this sleep disorder?
A. Sleep paralysis
B. Insomnia
C. Night terrors
D. Narcolepsy
----------------------------------------
Page 4
238
14. A 7-year-old child experiences bed wetting during sleep. The parents report that this has been
ongoing since infancy. The child has no recollection of the bedwetting episodes. What is the most
appropriate treatment option for this child?
(or)
What is the most appropriate treatment for a 7-year-old child experiencing bedwetting during sleep,
which has been ongoing since infancy and occurs with no recollection of the episodes?
A. Behavioral techniques, such as Bell and Pad alarms
B. Desmopressin
C. Imipramine
D. Oxybutynin
----------------------------------------
15. Which of the following components is not typically included in polysomnography?
A. Snoring observation
B. Oxygen saturation monitoring
C. Respiratory effort measurement
D. Blood pressure recording
----------------------------------------
Correct Answers
Question Correct Answer
Question 1 4
Question 2 3
Question 3 2
Question 4 4
Question 5 4
Question 6 4
Question 7 2
Question 8 1
Question 9 3
Question 10 3
Question 11 1
Question 12 1
Question 13 3
Question 14 1
Question 15 4
Page 5
239
Solution for Question 1:
Incorrect Options:
Option A
- Delta (δ) waves: These waves are typically seen in deep sleep or delta sleep, but they have a
much lower frequency (0.5-3Hz) compared to the waveform displayed in the EEG image.
Option B - Theta (θ) waves: While theta waves are observed during the transition from wakefulnes
s to sleep and are prominent in children, their frequency range (4-7Hz) does not match the wavefor
m displayed in the EEG image.
Option D - Beta (β) waves: Beta waves are typically present in a fully awake state with eyes open,
and their frequency range (14-30Hz) does not correspond to the waveform displayed in the EEG i
mage.
Incorrect Options:
Option A:Sleep stages are characterized by EEG, EOG, and EMG measurements: EEG measures
brainwave activity, EOG measures eye movements, and EMG measures muscle activity. These m
easurements are essential for distinguishing and classifying different sleep stages.
Option C: NREM sleep is divided into three stages: N1, N2, and N3: NREM sleep is indeed divided
into three stages: N1, N2, and N3. These stages represent different levels of sleep depth and are
characterized by specific EEG patterns and physiological changes. N1 is the lightest stage, N2 is d
eeper, and N3 is the deepest stage of NREM sleep, also known as slow-wave sleep.
Page 6
240
Option D: The majority of sleep time is spent in N2 stage: N2 is the longest sleep stage and constit
utes a substantial portion of the sleep cycle. It is a transitional stage between the lighter N1 and th
e deeper N3 stages, and it is characterized by specific EEG patterns, including sleep spindles and
K-complexes.
Incorrect Options:
Option A: NREM Stage 1 is associated with theta waves, so this option is correct.
Option B: NREM Stage 2 is characterized by the presence of sleep spindles and K
complexes in the EEG, so this option is correct.
Option C: NREM Stage 3 is characterized by the presence of high-amplitude delta waves, not alph
a waves, so this option is correct.
Incorrect Options:
Option A: The EEG activity shows increased beta activity similar to the awake state: This statemen
t is correct. During REM sleep, the EEG activity is increased and resembles the beta activity seen
during wakefulness.
Option B: Sawtooth waves or Sawtooth appearance is a characteristic feature of the EEG in this sl
eep stage: This statement is correct. Sawtooth waves or Sawtooth appearance is a
typical finding in the EEG during REM sleep.
Option C: Rapid eye movements (REM) are observed during this sleep stage: This statement is cor
rect. Rapid eye movements are a prominent feature of REM sleep, which give this stage its name.
Page 7
241
• In NREM sleep we do Not REMember our dreams.
Incorrect Options:
Option A: NREM sleep is associated with lower brain and physiological activity compared to the aw
ake state.
Option B: REM sleep is known as paradoxical sleep because the person is asleep, but the brain ac
tivity is similar to the awake state.
Option C: Body movements are absent during REM sleep, whereas they are present during NREM
sleep.
Page 8
242
Option D - Akathisia-Rotigotine: Rotigotine is a dopamine agonist primarily used for the treatment o
f Parkinson's disease and restless leg syndrome. It is not the preferred treatment for akathisia.
Page 9
243
Correct Option A: Kleine-Levin Syndrome
• The patient's clinical presentation, including recurrent episodes of excessive sleepiness,
hyperphagia (increased appetite), hypersexuality (increased sexual activities), and disinhibition
(aggressive behaviors), are consistent with Kleine-Levin Syndrome (KLS). KLS is a rare disorder
primarily seen in males during early adolescence. The episodes are characterized by hypersomnia
and other associated symptoms. Importantly, the patient is asymptomatic in between the episodes.
Incorrect Options:
Option B - Sleep apnea: Sleep apnea, is characterized by repetitive pauses in breathing during sle
ep, resulting in fragmented sleep and excessive daytime sleepiness. However, it does not present
with the hyperphagia, hypersexuality, and disinhibition observed in the given scenario.
Option C - Insomnia: Insomnia, is characterized by difficulty falling asleep, staying asleep, or experi
encing non-restorative sleep. It does not typically include symptoms such as hyperphagia, hyperse
xuality, and disinhibition.
Option D - Restless legs syndrome: Restless legs syndrome, is characterized by an uncontrollable
urge to move the legs, often accompanied by uncomfortable sensations. It does not include the co
nstellation of symptoms seen in the patient, such as excessive sleepiness, hyperphagia, hypersexu
ality, and disinhibition.
Page 10
244
characterized by intense episodes of fear or terror. The individual may exhibit behaviors such as
screaming, crying, or even appearing panicked.
Incorrect Options:
Option A - Sleep paralysis: Sleep paralysis, refers to a different sleep disorder characterized by the
temporary inability to move or speak while falling asleep or upon waking up. It does not involve the
sudden arousal with fearfulness seen in sleep terrors.
Option B - Insomnia: Insomnia, refers to difficulty initiating or maintaining sleep, and it does not enc
ompass the specific symptoms described in the scenario.
Option D - Narcolepsy: Narcolepsy, is a neurological disorder characterized by excessive daytime
sleepiness, sudden loss of muscle tone (cataplexy), sleep paralysis, and hallucinations. While slee
p terrors may occur in individuals with narcolepsy, the scenario does not provide sufficient informati
on to suggest narcolepsy as the primary diagnosis.
Page 11
245
ep-related breathing disorders, such as obstructive sleep apnea.
Option B. Oxygen saturation monitoring: Monitoring oxygen saturation levels, usually via a pulse ox
imeter, is an essential component of polysomnography. It helps identify episodes of desaturation,
which are common in sleep-related breathing disorders.
Option C. Respiratory effort measurement: Polysomnography includes the measurement of respira
tory effort, which involves monitoring chest and abdominal movements. This measurement is critic
al for identifying issues such as obstructive apneas and hypopneas during sleep.
Page 12
246
Previous Year Questions
1. What medication is typically prescribed to treat the condition experienced by a 25-year-old woman
who complains of a sensation resembling creepy crawling in her legs? This symptom is more
pronounced at night and hinders her ability to sleep. However, she finds relief by either engaging in
walking or moving her legs.
A. Gabapentin
B. Pramipexole
C. Vitamin B12
D. Iron tablets
----------------------------------------
2. Please determine the sleep stage based on the red-highlighted EEG findings.
A. Stage 1
B. Stage 2
C. Stage 3
D. Stage 4
----------------------------------------
3. Which of the following statements about somnambulism is incorrect?
A. Disorder of sleep arousal
B. Person will have full consciousness
C. Usually, terminates in awakening followed by confusion
D. There may be neurological condition associated
----------------------------------------
4. What does the given image represent?
247
A. Hypnogram
B. Electroencephalogram
C. Polysomnogram
D. Epworth sleep assessment
----------------------------------------
5. On performing polysomnography in a patient, the waves of EOG, EEG, and EMG, from above
downwards, are seen below. Which stage of sleep do the marked areas represent? EOG:
Electrooculography EEG: Electroencephalography EMG: Electromyography
A. REM sleep
B. NREM I sleep
C. NREM II sleep
D. NREM II sleep
----------------------------------------
Correct Answers
Question Correct Answer
Question 1 1
Question 2 4
Question 3 2
Page 2
248
Question 4 1
Question 5 1
Page 3
249
Solution for Question 3:
Correct Option b. Person will have full consciousness.
• Somnambulism, also known as sleepwalking, is a parasomnia disorder characterized by complex
motor behaviors and actions during sleep. During an episode of somnambulism, individuals are in a
state of partial arousal from sleep, and their consciousness is typically impaired or altered.
Incorrect options:
Option A. Disorder of sleep arousal: Somnambulism is classified as a disorder of sleep arousal bec
ause it involves partial awakenings from sleep with the occurrence of complex behaviors.
Option C. Usually terminates in awakening followed by confusion: Somnambulism episodes often e
nd with the sleepwalker waking up, and upon awakening, they may experience confusion and disor
ientation about their surroundings or actions during the episode.
Option B. There may be a neurological condition associated:While somnambulism can occur in oth
erwise healthy individuals, there can also be underlying neurological conditions or factors that contr
ibute to its occurrence. These may include conditions such as epilepsy, sleep disorders, or certain
medications.
Page 4
250
• The given polysomnography shows REM sleep. Rapid eye movements in REM sleep are
observed in the following graph. During REM sleep, there will be temporary muscle atonia.
• The brain is quite active during this stage and resembles a state of wakefulness. There will be
fluctuations in the heart rate and rate of breathing. Vivid dreams usually occur during REM sleep.
Incorrect Options:
Option B - NREM I: It occurs before wakefulness and sleep. Heart rate and respiration are reduced
during this stage. There is a reduction in muscle tone as well as core body temperature. Alpha and
theta waves are typically observed during the NREM I stage of sleep.
Option C - NREM II: K complexes and sleep spindles are observed. In this graph, K
complexes or sleep spindles are not visualized.
Option D
- NREM III: Characterized by low-frequency delta waves. Heart rate and respiratory rate are slow.
Page 5
251
Sexual Disorders
1. Which phase of the sexual response cycle is characterized by subjective feelings of sexual pleasure
and physiological changes such as increased heart rate, blood pressure, and sweating?
A. Desire Phase
B. Arousal Phase
C. Orgasm Phase
D. Resolution Phase
----------------------------------------
2. A 35-year-old man presents with difficulty in obtaining or maintaining an erection during sexual
activity. He reports experiencing early morning erections and nocturnal erections. Which of the
following treatment options is most appropriate for this patient and what is the likely diagnosis for his
condition?
(or)
What is the likely diagnosis and most appropriate treatment option for a 35-year-old man with difficulty
in obtaining or maintaining an erection during sexual activity, but still experiencing early morning
erections and nocturnal erections?
A. Selective serotonin reuptake inhibitors (SSRIs); Organic erectile dysfunction
B. Phosphodiesterase-5 (PDE-5) inhibitors; Psychogenic erectile dysfunction
C. Psychotherapy and counselling; Organic erectile dysfunction
D. Hormone replacement therapy; Psychogenic erectile dysfunction
----------------------------------------
3. A 28-year-old man complains of consistently ejaculating within one minute of vaginal penetration. He
reports that this has been occurring for the past six months. Which of the following treatment options is
most appropriate for this patient and what is the likely diagnosis for his condition?
(or)
What is the likely diagnosis and most appropriate treatment option for a 28-year-old man complaining of
consistently ejaculating within one minute of vaginal penetration for the past six months?
A. Phosphodiesterase-5 (PDE-5) inhibitors; Erectile dysfunction
B. Selective serotonin reuptake inhibitors (SSRIs); Premature ejaculation
C. Psychotherapy and counselling; Erectile dysfunction
D. Testosterone replacement therapy; Premature ejaculation
----------------------------------------
4. What is the most likely diagnosis for a 9-year-old child exhibiting marked incongruence between their
expressed gender and assigned gender, expressing a strong desire to be of the other gender, and
displaying a dislike for their own sexual anatomy?
(or)
A 9-year-old child is brought to the hospital due to expressing a strong desire to be a female,
demonstrating a preference for cross-dressing, playing with toys typically associated with girls, and
displaying a dislike for his own sexual anatomy. What is the most likely diagnosis for this child?
252
A. Transsexualism
B. Dual role transvestism
C. Gender identity disorder of childhood
D. Oppositional Defiant Disorder (ODD)
----------------------------------------
5. Which of the following conditions is characterized by involuntary muscle contraction in the outer third
of the vagina, making penile insertion difficult?
A. Vaginismus
B. Dyspareunia
C. Vulvodynia
D. Endometriosis
----------------------------------------
6. What is satyriasis?
A. Excessive sexual desire in males
B. Excessive sexual desire in females
C. Involuntary muscle contraction in the outer third of the vagina
D. Persistent genital pain during or after sexual intercourse in males
----------------------------------------
7. Sarah, a 35-year-old woman experiences intense and recurrent sexual fantasies involving observing
people undressing or engaging in sexual activities without their knowledge. Which of the following
conditions is most likely affecting Sarah?
(or)
What is the most likely diagnosis for a 35-year-old woman experiencing intense and recurrent sexual
fantasies involving observing people undressing or engaging in sexual activities without their
knowledge?
A. Vaginismus
B. Dyspareunia
C. Voyeurism
D. Satyriasis
----------------------------------------
8. Alex, a 25-year-old individual, has been experiencing a persistent and intense desire to live as a
member of the opposite sex. He feels discomfort and a sense of inappropriateness with his own
anatomical sex. Alex wishes to undergo hormonal treatment and surgery to align his body with his
preferred gender. He has been expressing the feeling of being "trapped in the body of the wrong
gender" for the past four years. Which of the following best describes Alex's condition?
(or)
What best describes the condition of a 25-year-old individual, Alex, who has a persistent and intense
desire to live as a member of the opposite sex, experiences discomfort with his own anatomical sex,
and wishes to undergo hormonal treatment and surgery to align his body with their preferred gender?
Page 2
253
A. Fetishism
B. Transsexualism
C. Exhibitionism
D. Voyeurism
----------------------------------------
9. Emma, a 30-year-old individual, occasionally wears clothes of the opposite gender to experience the
feeling of being the other gender but has no desire for a sex change. There is no sexual arousal
associated with cross-dressing. Which of the following best describes Emma's condition?
(or)
What best describes the condition of a 30-year-old individual, Emma, who occasionally wears clothes
of the opposite gender for the experience but has no desire for a sex change and does not experience
sexual arousal associated with cross-dressing?
A. Transvestism
B. Dual role transvestism
C. Exhibitionism
D. Voyeurism
----------------------------------------
Correct Answers
Question Correct Answer
Question 1 2
Question 2 2
Question 3 2
Question 4 3
Question 5 1
Question 6 1
Question 7 3
Question 8 2
Question 9 2
Page 3
254
al fantasies or thoughts, but it doesn't typically involve the physiological changes mentioned in the
question.
Option C: Orgasm Phase: This phase follows the Arousal Phase and is marked by the peak of sex
ual pleasure and the release of sexual tension. It involves intense pleasure and contractions of the
pelvic muscles. While there is a significant increase in subjective sexual pleasure in this phase but
other features such as ejaculation in males, vaginal and uterus contraction in females are seen.
Option D: Resolution Phase: The Resolution Phase is the final stage of the sexual response cycle.
In this phase, the body returns to its pre-arousal state. Heart rate, blood pressure, and other physio
logical responses return to baseline levels. It is not associated with increased physiological change
s or subjective sexual pleasure.
Page 4
255
Option A - Phosphodiesterase-5 (PDE-5) inhibitors; Erectile dysfunction: Phosphodiesterase-5 (PD
E-5) inhibitors are primarily used for the treatment of erectile dysfunction, not premature ejaculation
.
Option C - Psychotherapy and counselling; Erectile dysfunction: Psychotherapy and counselling m
ay be beneficial in addressing psychological factors contributing to sexual disorders, but SSRIs are
the first-line pharmacological treatment for premature ejaculation.
Option D - Testosterone replacement therapy; Premature ejaculation: Testosterone replacement th
erapy is indicated for hypogonadism or androgen deficiency, which is not the underlying cause of p
remature ejaculation in this case.
Incorrect Options:
Option B - Dyspareunia: Dyspareunia refers to persistent or recurrent pain during sexual intercours
e. It can have various causes, such as physical issues, psychological factors, or underlying medica
l conditions.
Page 5
256
Option C
- Vulvodynia: Vulvodynia is characterized by chronic vulvar pain or discomfort, often described as a
burning or stinging sensation. It can cause pain during sexual intercourse, but it is not specifically r
elated to muscle contraction in the outer third of the vagina.
Option D - Endometriosis: Endometriosis is a condition in which the tissue lining the uterus (endom
etrium) grows outside the uterus, leading to pain and possible fertility issues. It can cause pelvic pa
in and discomfort, but it is not directly related to involuntary muscle contraction in the outer third of t
he vagina.
Page 6
257
Solution for Question 8:
Correct Option B: Transsexualism
• Alex's persistent desire to live as a member of the opposite sex, discomfort with their anatomical
sex, and the wish for hormonal treatment and surgery align with the condition of transsexualism or
gender dysphoria.
Incorrect Options:
Option A - Fetishism: This option is incorrect. Fetishism involves sexual focus or arousal towards o
bjects or non-genital body parts. Alex's experience goes beyond sexual attraction and encompasse
s a strong desire for personal identity alignment.
Option C - Exhibitionism: This option is incorrect. Exhibitionism refers to the recurrent urge to expo
se one's genitals to strangers. It is not related to Alex's experience of wanting to transition to the op
posite gender.
Option D - Voyeurism: This option is incorrect. Voyeurism involves recurrently observing unaware i
ndividuals undressing or engaged in sexual behavior. It does not relate to Alex's desire to live as a
member of the opposite sex.
Page 7
258
Previous Year Questions
1. What non-pharmacological measures can be recommended to a middle-aged man who experiences
premature ejaculation during intercourse?
A. Cognitive behavioral therapy
B. Exposure and response prevention therapy
C. Squeeze technique
D. Sensate focus therapy
----------------------------------------
2. Match the following: Eonism Exhibitionism Frotteurism Necrophilia A. Exposure of one’s genitals to
an unsuspecting stranger B. Sexual gratification by intercourse with dead bodies C. Sexual pleasure by
wearing clothes of the opposite sex D. Sexual gratification by rubbing private parts against another
person
ls to an unsuspecting strangerB. Sexual gratification by intercourse with dead bodiesC. Sexual pleasure by wearing clothes of the opposite
259
----------------------------------------
Correct Answers
Question Correct Answer
Question 1 3
Question 2 2
Question 3 4
Question 4 3
Question 5 3
Page 2
260
Correct Option D:
Erotomanic type of delusion: Also known as “psychosis passionate”. This type of delusion involves
a belief that a person usually of higher status is in love with the patient. These patients are usually
socially withdrawn, dependent, and sexually inhibited with a poor level of social and/or occupationa
l functioning. “Paradoxical conduct” is an important characteristic wherein all denials of affection ar
e rationalized as affirmations. Males with this type of delusion can be more aggressive.
Incorrect choices:
Option A: megalomania or delusion of grandeur: The patient believes that he has some exceptional
identity or power.
Option B: delusion of reference: The patient believes that events happening around him are someh
ow related to him. For example, a
patient claimed that the tube light of his apartment was flickering as there was a
camera fitted inside through which his movements are being recorded.
Option C: delusion of control: the patient believes others are trying to control their thoughts or beha
viors. For example: ”My doctor is trying to plant thoughts inside my head.”
Page 3
261
Incorrect Options:
Option A: Retrograde ejaculation- Retrograde ejaculation is a condition in which semen flows back
ward into the bladder instead of being expelled through the penis during ejaculation. It can be caus
ed by certain medical conditions, medications, or surgery. Treatment options for retrograde ejacula
tion may include medications to improve bladder neck function or techniques to facilitate antegrade
ejaculation, but the Seman's squeeze technique is not typically used for this condition.
Option B: Erectile dysfunction- Erectile dysfunction (ED) refers to the inability to achieve or maintai
n an erection sufficient for sexual intercourse. It can have various causes, including physical, psych
ological, or a combination of both.
Option D: Painful ejaculation- Painful ejaculation, also known as dysorgasmia, refers to discomfort
or pain during ejaculation. It can be caused by various factors, including infections, inflammation, st
ructural abnormalities, or psychological factors. The approach to managing painful ejaculation invol
ves identifying and addressing the underlying cause. Treatment options may include medications, li
festyle modifications, physical therapy, or other interventions depending on the specific cause and i
ndividual needs.
Page 4
262
Eating Disorders and Personality Disorders
1. A 16-year-old female presents to her healthcare provider with the following symptoms: restricted
food intake, fear of weight gain, and distorted body image. She has a significantly low body weight and
has not had a menstrual period for the past six months. Additionally, she engages in behaviors such as
hiding food and spending excessive time rearranging food on her plate. Which of the following is the
most likely diagnosis for this patient?
(or)
What is the most likely diagnosis for a 16-year-old female patient who presents with restricted food
intake, fear of weight gain, distorted body image, significantly low body weight, and absence of
menstrual periods for the past six months, along with behaviors like hiding food and rearranging food
on her plate excessively?
A. Bulimia nervosa
B. Anorexia nervosa
C. Binge eating disorder
D. Avoidant/restrictive food intake disorder (ARFID)
----------------------------------------
2. What is the most appropriate treatment option for a 17-year-old female patient with significantly low
body weight, intense fear of weight gain, distorted body image, and engaging in restrictive eating and
excessive exercise?
(or)
A 17-year-old female presents to the clinic with significantly low body weight, intense fear of weight
gain, and distorted body image. She admits to restricting her food intake and engaging in excessive
exercise. The treatment plan is discussed with the patient. Which of the following treatment options is
most appropriate for this patient?
A. Psychotherapy alone
B. Nutritional rehabilitation and weight restoration only
C. Pharmacotherapy with SSRIs only
D. A combination of psychotherapy, nutritional rehabilitation, and pharmacotherapy
----------------------------------------
3. A 20-year-old female presents with a history of recurrent episodes of binge eating followed by
self-induced vomiting. She reports feeling a lack of control during these episodes and has a strong
desire to prevent weight gain. She maintains a normal weight and has experienced dental issues such
as enamel erosion and dental caries. What is the most likely diagnosis for this patient?
(or)
What is the most likely diagnosis for a 20-year-old female who presents with recurrent episodes of
binge eating followed by self-induced vomiting, a lack of control during these episodes, and a desire to
prevent weight gain despite maintaining a normal weight and experiencing dental issues?
A. Anorexia Nervosa
B. Bulimia Nervosa
C. Binge Eating Disorder
263
D. Orthorexia Nervosa
----------------------------------------
4. What is the first-line treatment for a 20-year-old female with a history of binge eating followed by
self-induced vomiting, a sense of lack of control over eating during these episodes, and calluses on the
knuckles of her hands?
(or)
A 20-year-old female presents with a history of recurrent episodes of binge eating followed by
self-induced vomiting. She reports feeling a lack of control over her eating during these episodes. On
examination, you notice callus on the knuckles of her hands. Which of the following is the first-line
treatment for this condition?
Page 2
264
those she perceives as enemies. Based on the given scenario, which personality disorder is most likely
to be diagnosed in Maria?
(or)
Which personality disorder is most likely to be diagnosed in a 28-year-old woman who exhibits constant
suspicion of others, difficulty trusting people, and interprets innocent actions as threatening, while
holding grudges and having trouble forgiving perceived enemies?
A. Borderline personality disorder
B. Paranoid personality disorder
C. Narcissistic personality disorder
D. Avoidant personality disorder
----------------------------------------
7. Sarah is a 30-year-old woman who avoids social interactions and prefers spending most of her time
alone. She rarely shows emotions, and when someone praises or criticizes her, she remains indifferent
and does not respond. Sarah has few close friends and little interest in engaging in sexual activities.
Based on the provided scenario, which personality disorder is most likely to be diagnosed in Sarah?
(or)
Which personality disorder is most likely to be diagnosed in a 30-year-old woman who avoids social
interactions, shows limited emotional expression, and lacks interest in close relationships or sexual
activities?
A. Schizoid personality disorder
B. Borderline personality disorder
C. Antisocial personality disorder
D. Avoidant personality disorder
----------------------------------------
8. John is a 35-year-old man who frequently engages in illegal activities such as theft, fraud, and deceit.
He shows no remorse or guilt for his actions and often blames others for his misdeeds. John has a
history of aggressive behaviour, including physical fights and assaults. He has a disregard for the rights
of others and has been diagnosed with a substance use disorder. Based on the provided scenario,
which personality disorder is most likely to be diagnosed in John?
(or)
What personality disorder is most likely to be diagnosed in a 35-year-old man who engages in illegal
activities, lacks remorse, shows aggressive behavior, and has a substance use disorder?
A. Dissocial personality disorder
B. Borderline personality disorder
C. Narcissistic personality disorder
D. Schizoid personality disorder
----------------------------------------
9. Which defense mechanism and treatment therapy are commonly associated with borderline
personality disorder?
A. Defense Mechanism: Splitting; Treatment Therapy: Dialectical Behaviour Therapy (DBT)
Page 3
265
B. Defense Mechanism: Projection; Treatment Therapy: Cognitive Behavioural Therapy (CBT)
C. Defense Mechanism: Denial; Treatment Therapy: Psychoanalysis
D. Defense Mechanism: Displacement; Treatment Therapy: Mindfulness-Based Therapy
----------------------------------------
10. Mr. Smith is a 40-year-old man who is extremely meticulous and rigid in his daily routine. He is
known for his perfectionism and excessive attention to detail. He spends long hours at work and often
neglects his personal life and relationships. Despite completing tasks, he constantly feels they could be
improved and delayed due to his fascination with perfection. Which personality disorder is Mr. Smith
most likely experiencing?
(or)
What personality disorder is most likely experienced by a 40-year-old man who exhibits perfectionism,
excessive attention to detail, and neglects personal life due to a need for constant improvement in
tasks?
A. Borderline Personality Disorder
B. Antisocial Personality Disorder
C. Obsessive-Compulsive Personality Disorder
D. Histrionic Personality Disorder
----------------------------------------
11. A 30-year-old woman has been experiencing recurrent episodes of stealing items that are not
needed for personal use or monetary gain. This behavior has caused significant distress and negative
consequences in her life. She feels an irresistible urge to steal, despite knowing it is wrong. What is the
most likely diagnosis for this individual?
(or)
What is the most likely diagnosis for a 30-year-old woman who experiences recurrent episodes of
stealing items she doesn't need, feels an irresistible urge to steal despite knowing it's wrong, and faces
distress and negative consequences due to this behavior?
A. Pyromania
B. Kleptomania
C. Intermittent Explosive Disorder
D. Compulsive Sexual Behaviour Disorder
----------------------------------------
12. A patient presents with a persistent pattern of attention-seeking behaviour, exaggerated speech,
and a desire to be the center of attention. She often behaves in a dramatic and flirtatious manner, using
their physical appearance to draw attention. Which personality disorder is most likely?
(or)
What personality disorder is most likely in a patient who exhibits attention-seeking behavior,
exaggerated speech, and a desire to be the center of attention, often behaving dramatically and
flirtatiously?
A. Narcissistic personality disorder
B. Histrionic personality disorder
Page 4
266
C. Borderline personality disorder
D. Antisocial personality disorder
----------------------------------------
Correct Answers
Question Correct Answer
Question 1 2
Question 2 4
Question 3 2
Question 4 1
Question 5 3
Question 6 2
Question 7 1
Question 8 1
Question 9 1
Question 10 3
Question 11 2
Question 12 2
Page 5
267
Solution for Question 2:
Correct Option D: A combination of psychotherapy, nutritional rehabilitation, and pharmacotherapy
• In the treatment of Anorexia Nervosa, a multidisciplinary approach is typically recommended. The
combination of psychotherapy, nutritional rehabilitation, and pharmacotherapy has been found to
be most effective in addressing the complex nature of the disorder.
Incorrect Options:
Option A - Psychotherapy alone: Is not sufficient for the comprehensive treatment of Anorexia Nerv
osa. Psychotherapy, such as cognitive-behavioral therapy (CBT) or family-based therapy (FBT), is
an important component but should be combined with other interventions.
Option B - Nutritional rehabilitation and weight restoration only: Addresses the physical aspect of th
e disorder but neglects the psychological and emotional components. Psychotherapy is crucial in h
elping the patient address distorted body image, fear of weight gain, and underlying psychological i
ssues.
Option C -Pharmacotherapy with SSRIs only: May be beneficial for co-occurring mood disorders, b
ut it is not the standalone treatment for Anorexia Nervosa. Medications alone do not address the co
re symptoms and psychological factors associated with the disorder.
Page 6
268
• The presence of calluses on the knuckles, known as Russell's sign, is a physical finding
commonly associated with self-induced vomiting in Bulimia Nervosa. This sign is a result of
repeated contact between the teeth and knuckles during the act of inducing vomiting.
Incorrect Options:
Option B - Selective Serotonin Reuptake Inhibitors (SSRIs): Selective Serotonin Reuptake Inhibitor
s (SSRIs) such as fluoxetine are commonly used as an adjunct to CBT in the treatment of Bulimia
Nervosa. SSRIs can help reduce the frequency of binge eating episodes and improve mood sympt
oms associated with the disorder. However, CBT is considered the first-line treatment due to its eff
ectiveness and focus on addressing the underlying psychological factors contributing to the conditi
on.
Option C - Family therapy: Family therapy can be beneficial in certain cases, particularly when ther
e are family dynamics that contribute to the maintenance of the disorder.
Option D - Electroconvulsive therapy (ECT): Electroconvulsive therapy (ECT) is not indicated for th
e treatment of Bulimia Nervosa and is reserved for severe cases of other psychiatric conditions.
In conclusion, based on the given information, the most likely diagnosis for this patient is Binge Eat
ing Disorder (Option C) due to the presence of recurrent episodes of binge eating without compens
atory behaviours and being in the obese range.
Page 7
269
Solution for Question 6:
Correct Option B: Paranoid personality disorder
• Paranoid personality disorder: Paranoid personality disorder is characterized by a long-standing
pattern of suspicion, distrust, and a tendency to interpret others' actions as malicious or harmful.
Individuals with this disorder may hold grudges against others without justification.
Incorrect Options:
Option A - Borderline personality disorder: Borderline personality disorder is characterized by unsta
ble emotions, impulsive behaviour, and difficulties in relationships. Individuals with this disorder oft
en have a fear of abandonment, engage in self-destructive behaviours, and experience intense mo
od swings. While there may be some overlap in symptoms with paranoid personality disorder, the k
ey feature of pervasive suspiciousness and distrust is more indicative of paranoid personality disor
der rather than borderline personality disorder.
Option C - Narcissistic personality disorder: Narcissistic personality disorder is characterized by an
inflated sense of self-importance, a constant need for admiration, and a
lack of empathy for others. Individuals with this disorder have a grandiose view of themselves and
often seek excessive attention and admiration. While there may be some overlap in terms of interp
ersonal difficulties, the primary characteristic of suspiciousness and distrust is not typically associat
ed with narcissistic personality disorder.
Option D - Avoidant personality disorder: Avoidant personality disorder is characterized by a perva
sive pattern of social inhibition, feelings of inadequacy, and hypersensitivity to criticism or rejection.
Individuals with this disorder tend to avoid social interactions and fear being judged or ridiculed. W
hile there may be some overlap in terms of social difficulties, the key feature of pervasive suspiciou
sness and distrust is not typically associated with avoidant personality disorder.
Page 8
270
Solution for Question 8:
Correct Option A: Dissocial personality disorder
• Antisocial personality disorder (Dissocial personality disorder): This is the correct answer. The
scenario describes John's lack of regard for the rights of others, frequent involvement in unlawful
activities, aggressive behaviour, lack of guilt or remorse, and substance use disorder. These are
typical features of antisocial personality disorder.
Incorrect Options:
Option B - Borderline personality disorder: Borderline personality disorder is characterized by emoti
onal instability, impulsivity, and difficulties in relationships. The scenario does not mention these sy
mptoms, making it less likely that John has borderline personality disorder.
Option C - Narcissistic personality disorder: Narcissistic personality disorder is characterized by an
inflated sense of self-importance, a need for admiration, and a lack of empathy. While John's beha
viour may involve manipulative tendencies, the core features of narcissistic personality disorder are
not evident in the scenario.
Option D - Schizoid personality disorder: Schizoid personality disorder is characterized by social wi
thdrawal, emotional coldness, and a preference for solitary activities. While John's behaviour may i
nvolve some emotional detachment, his involvement in unlawful activities and aggressive behaviou
r are not consistent with schizoid personality disorder.
Page 9
271
Defense Mechanism
Description
Denial
Refusing to accept reality or the truth of a situation.
Repression
Unconscious blocking of unacceptable thoughts, feelings, or memories.
Projection
Attributing one's own undesirable thoughts or feelings to others.
Displacement
Redirecting emotions from the original source to a substitute target.
Sublimation
Channeling socially unacceptable impulses or emotions into socially acceptable activities.
Rationalization
Creating logical or reasonable explanations for behaviors that are actually driven by irrational motiv
es.
Regression
Returning to earlier, more childlike stages of behavior in response to stress or frustration.
Intellectualization
Focusing on abstract or intellectual aspects of a
situation to avoid dealing with the emotional components.
Undoing
Trying to counteract or make amends for unacceptable thoughts or actions through symbolic acts.
Compensation
Overemphasizing or overachieving in one area to compensate for perceived deficiencies in another
area.
Reaction Formation
Adopting attitudes and behaviors that are the opposite of one's true feelings or impulses.
Identification
Adopting the characteristics or behaviors of someone else to avoid feelings of inadequacy or to gai
n a sense of belonging.
Suppression
Conscious and intentional pushing aside or postponing the acknowledgment of unwanted thoughts
or feelings.
Fantasy
Creating an imagined scenario or situation to escape from reality or cope with difficult emotions.
Humor
Page 10
272
Using humor to cope with stress or anxiety, often by finding amusement in situations that might oth
erwise be distressing.
Incorrect Options:
Option B - Defense Mechanism: Projection; Treatment Therapy: Cognitive Behavioural Therapy (C
BT): Projection is a defense mechanism commonly seen in various psychiatric disorders, such as p
aranoid personality disorder and narcissistic personality disorder. Cognitive Behavioural Therapy (
CBT) is a widely used therapeutic approach for various mental health conditions, but it is not specif
ically associated with borderline personality disorder.
Option C - Defense Mechanism: Denial; Treatment Therapy: Psychoanalysis: Denial is a defense
mechanism observed in different psychiatric disorders, including substance use disorders and som
e anxiety disorders. Psychoanalysis is a therapeutic approach primarily used in psychodynamic the
rapy to explore unconscious conflicts and is not the primary treatment modality for borderline perso
nality disorder.
Option D - Defense Mechanism: Displacement; Treatment Therapy: Mindfulness-Based Therapy:
Displacement is a defense mechanism commonly observed in various psychiatric conditions, such
as post-traumatic stress disorder and obsessive-compulsive disorder. Mindfulness-Based Therapy
emphasizes present-moment awareness and is used in the treatment of several mental health diso
rders, but it is not the primary treatment approach for borderline personality disorder.
Page 11
273
Obsessive-Compulsive Personality Disorder (OCPD)
Nature of the Disorder
Characterized by obsessions and compulsions.
Characterized by a pervasive pattern of preoccupation with orderliness, perfectionism, and control.
Primary Symptoms
Obsessions: Intrusive, distressing thoughts. Compulsions: Repetitive behaviors to reduce anxiety.
Preoccupation with order, perfectionism, and control; difficulty delegating tasks.
Diagnostic Criteria
Presence of obsessions and/or compulsions causing distress and impairment.
Pervasive pattern of preoccupation with orderliness, perfectionism, and control, impacting flexibility
.
Insight
Typically recognizes obsessions as irrational and compulsions as excessive.
Individuals often believe their perfectionism and control are justified and necessary.
Focus of Treatment
Cognitive-behavioral therapy (CBT), exposure and response prevention (ERP), medication (SSRIs)
.
Psychotherapy (e.g., CBT), addressing maladaptive thought patterns; medication may be consider
ed for associated symptoms.
Flexibility and Openness
Obsessive thoughts can be distressing; flexibility in thinking may be impaired.
Difficulty with flexibility and openness, rigid adherence to rules and routines.
Impact on Daily Functioning
Interferes with daily life due to time-consuming rituals and distressing thoughts.
May affect interpersonal relationships and work due to perfectionism and rigidity.
Insight into Condition
Typically recognizes the irrational nature of obsessions and compulsions.
Often lacks insight into the problematic nature of their perfectionism and control.
Examples of Behaviors
Excessive handwashing, checking, counting; driven by specific fears.
Overly focused on details, overworking, reluctance to delegate tasks, rigid adherence to rules.
Incorrect Options:
Option A- Borderline Personality Disorder: Borderline Personality Disorder is characterized by unst
able interpersonal relationships, impulsivity, and intense mood swings. This does not match Mr. S
mith's symptoms of perfectionism, attention to detail, and rigidity.
Option B- Antisocial Personality Disorder: Antisocial Personality Disorder is characterized by a disr
egard for the rights of others, involvement in unlawful activities, and lack of guilt or remorse. Mr. S
mith does not exhibit these behaviours or traits.
Page 12
274
Option D- Histrionic Personality Disorder: Histrionic Personality Disorder is characterized by attenti
on-seeking behaviour, dramatic emotions, and a
desire to be the centre of attention. Mr. Smith does not display these characteristics.
Page 13
275
Previous Year Questions
1. What is the likely diagnosis for a 39-year-old man who frequently washes his hands (15-20 times a
day), leading to a negative impact on his work performance? He attributes this behavior to persistent
thoughts that his hands are contaminated with dirt.
A. Bipolar disorder
B. Obsessive compulsive disorder
C. Schizophrenia
D. Mania
----------------------------------------
2. Match the following: 1. Kleptomania A. Intense desire to drink alcoholic drinks 2. Pyromania B.
Intense desire to mutilate 3. Mutilomania C. Intense desire to steal items of trivial value 4. Dipsomania
D. Intense desire to burn things
1. Kleptomania A. Intense desire to drink alcoholic drinks
2. Pyromania B. Intense desire to mutilate
3. Mutilomania C. Intense desire to steal items of trivial value
4. Dipsomania D. Intense desire to burn things
276
5. What will be your next course of action upon noticing multiple cigarette burns on the forearm of a
14-year-old girl who has presented to the emergency room with a fractured forearm and claims to have
tripped and fallen?
A. Conduct a thorough examination
B. Seek a social worker’s help
C. Inform higher authorities immediately
D. Inform your colleagues
----------------------------------------
6. The likely diagnosis for a girl who exhibits typical language milestones but primarily focuses on
observing her own hands and lacks interaction with others is:
A. ADHD
B. Autism
C. Asperger's syndrome
D. Rett's syndrome
----------------------------------------
7. A young male who is shy and prefers social isolation appears to be emotionally cold and resides
alone. There is no history of hallucinations or delusions. The most likely diagnosis is:
A. Paranoid personality disorder
B. Schizoid personality disorder
C. Antisocial personality disorder
D. Emotionally unstable personality disorder
----------------------------------------
8. What is a true statement regarding depersonalization disorder?
A. More common after life threatening trauma
B. Seen in seizure and migraine patients
C. More common in females
D. Mean age at onset is 25 years
----------------------------------------
Correct Answers
Question Correct Answer
Question 1 2
Question 2 3
Question 3 3
Question 4 3
Question 5 1
Page 2
277
Question 6 3
Question 7 2
Question 8 1
Page 3
278
• It generally has a poorer prognosis with exposure and response prevention (ERP) in
obsessive-compulsive disorder (OCD).
• ERP is a widely used and effective treatment for OCD that involves gradually exposing individuals
to their obsessions or feared situations and preventing the associated compulsive behaviors.
Incorrect options:
Option A. Pathological doubt: Pathological doubt refers to excessive and irrational doubts that indiv
iduals with OCD experience, often leading them to seek reassurance repeatedly. While pathologica
l doubt can be challenging to treat, it generally responds well to ERP and has a
more favorable prognosis compared to hoarding.
Option B. Magical thinking: Magical thinking involves beliefs that one's thoughts or actions can influ
ence events in unrealistic or supernatural ways. While it can be a
symptom of OCD, it doesn't necessarily indicate a poor prognosis with ERP. The effectiveness of E
RP treatment depends on the individual and the specific nature of their OCD symptoms.
Option D. Contamination obsession: Contamination obsessions involve an intense fear of contamin
ation or germs and often result in excessive cleaning or avoidance behaviors. Contamination obse
ssions tend to respond well to ERP treatment, and individuals with this symptom subtype generally
have a better prognosis compared to hoarding.
Page 4
279
o ensure the safety and well-being of the child. In many healthcare settings, there are designated p
rofessionals who can be called upon to assist in cases involving suspected abuse or neglect. Optio
n C. Inform higher authorities immediately: While it is important to address the situation promptly, i
nforming higher authorities immediately might not be the first step. The immediate focus should be
on providing medical care and conducting a thorough examination of the patient. However, if durin
g the examination there are clear signs of abuse or neglect, it is essential to report the situation to t
he appropriate authorities in accordance with the established protocols and legal requirements in y
our jurisdiction. This may include contacting child protective services or law enforcement. Option D.
Inform your colleagues: While it can be helpful to discuss the case with colleagues, especially if th
ey have experience or expertise in child protection, it should not be the first step in this scenario. T
he priority is to ensure the immediate safety and medical care of the patient through a
thorough examination and, if necessary, involving a social worker or appropriate authorities.
Page 5
280
avoid close relationships and may show a lack of desire for social interaction. The description of a
young male who is shy, prefers social isolation, and appears emotionally cold aligns more closely
with the features of schizoid personality disorder.
Incorrect Options:
Option A: Paranoid personality disorder: Paranoid personality disorder is characterized by a pervas
ive distrust and suspicion of others, interpreting their motives as malevolent. Individuals with paran
oid personality disorder often exhibit a pattern of suspicion, hyper-vigilance, and a reluctance to co
nfide in others. However, the scenario you described does not mention any paranoid ideation or sig
nificant suspiciousness, making paranoid personality disorder less likely.
Option C: Antisocial personality disorder: Antisocial personality disorder is characterized by a
disregard for the rights of others and a lack of empathy. Individuals with this disorder often engage
in impulsive and irresponsible behavior, have difficulties conforming to social norms, and may exhi
bit a pattern of deceit and manipulation. While the scenario does not provide information about har
mful or exploitative behavior towards others, it suggests more of a preference for social isolation ra
ther than active disregard for others, making antisocial personality disorder less likely.
Option D: Emotionally unstable personality disorder: Emotionally unstable personality disorder, als
o known as borderline personality disorder, is characterized by a pervasive pattern of instability in
mood, self-image, and interpersonal relationships. Individuals with this disorder often experience in
tense and rapidly shifting emotions, engage in impulsive behavior, and have difficulties maintaining
stable relationships. The description provided does not indicate significant emotional instability or i
mpulsive behavior, making emotionally unstable personality disorder less likely.
Page 6
281
Forensic Psychiatry
1. Who is eligible to make an advance directive?
A. Only minors
B. Only individuals with a mental illness
C. Every person who is not a minor
D. Only individuals with a physical illness
----------------------------------------
2. Which of the following components are necessary for establishing criminal responsibility?
A. Voluntary conduct without any intent
B. Socially harmful act and mental illness
C. Actus reus and mens rea
D. Delusion of persecution and intoxication
----------------------------------------
3. A 28-year-old man, John, is brought to the emergency department by the police after being found
wandering aimlessly on the streets, muttering to himself, and exhibiting erratic behavior. Upon
examination, he appears disoriented, disheveled, and is unable to maintain a coherent conversation.
John's family reports a history of schizophrenia and episodes where he becomes detached from reality.
When asked about his actions, John seems unaware of his surroundings and believes he is on a
mission to save the world from an alien invasion. He adamantly insists that his actions are necessary to
protect humanity. Which legal principle applies to John's situation?
(or)
Which rule states that an act is not an offense if it is done by a person who, at the time of doing it, is of
unsound mind and incapable of knowing the nature of the act or that it is wrong or contrary to the law?
A. Durham Rule
B. McNaughton's Rule
C. Curren's Rule
D. Section 85 IPC
----------------------------------------
4. Who is obligated to report cases under the POCSO Act for an offense committed or likely to be
committed against a child?
A. Parents or guardians of the child
B. School teachers or counselors
C. Special juvenile police units (SJPU) only
D. Any person who has knowledge of the offense
----------------------------------------
5. When does Aggravated Sexual Assault occur under the POCSO Act in terms of the age of the child?
A. When the victim is under 21 years old
282
B. When the victim is under 13 years old
C. When the victim is under 12 years old
D. When the victim is under 18 years old
----------------------------------------
6. According to the Kübler-Ross model, what is the correct order of the stages of grief?
A. Anger, Bargaining, Depression, Denial and shock, Acceptance
B. Acceptance, Denial and shock, Anger, Bargaining, Depression
C. Denial and shock, Anger, Bargaining, Depression, Acceptance
D. Depression, Acceptance, Denial and shock, Anger, Bargaining
----------------------------------------
7. Which of the following statements accurately distinguishes between bereavement and depression?
A. Bereavement involves the coexistence of positive and negative emotions, while depression is
predominantly characterized by negative emotions.
B. Both bereavement and depression often involve feelings of guilt, worthlessness, and suicidal
tendencies.
C. Bereavement typically occurs anytime, while depression has an onset within 2 months of the death.
D. Both bereavement and depression result in clinically significant functional impairment.
----------------------------------------
8. Which protocol is commonly used for breaking bad news to patients?
A. HOPE protocol
B. SPIKES protocol
C. SOAP protocol
D. FAST protocol
----------------------------------------
9. Which type of Electroconvulsive Therapy (ECT) configuration is increasingly used due to its better
cognitive side effects profile?
A. Unilateral ECT
B. Direct ECT
C. Bilateral ECT with bifrontotemporal placement
D. Modified ECT
----------------------------------------
10. Which of the following statements best illustrates a key difference between bereavement and
depression?
A. Bereavement is subjective feeling precipitated by loss of a loved one, while depression is considered
a pathological mental disorder.
B. Functional impairment is moderate in bereavement and severe in depression
Page 2
283
C. Negative and positive emotions coexist in bereavement but negative emotions are predominant in
depression
D. Feeling of worthlessness is more in bereavement than in depression
----------------------------------------
11. What is the most common indication for Electroconvulsive Therapy (ECT)?
A. Schizophrenia
B. Manic episodes
C. Major Depressive Disorder
D. Intractable seizure disorder
----------------------------------------
12. What is the recommended duration for an effective seizure during an ECT session?
A. At least 10 seconds
B. At least 60 seconds
C. At least 90 seconds
D. At least 25 seconds
----------------------------------------
13. In which configuration of ECT is cognitive side effects often more pronounced?
A. Bilateral ECT with bifrontal electrode placement
B. Bilateral ECT with bi-frontotemporal electrode placement
C. Unilateral ECT with electrodes placed on the vertex and right side
D. Bilateral ECT with electrodes placed on both sides of the scalp
----------------------------------------
Correct Answers
Question Correct Answer
Question 1 3
Question 2 3
Question 3 2
Question 4 4
Question 5 3
Question 6 3
Question 7 1
Question 8 2
Question 9 1
Question 10 3
Page 3
284
Question 11 3
Question 12 4
Question 13 2
Page 4
285
schizophrenia, such as disorientation, muttering, erratic behavior, and a belief in a delusional mission to
save the world, indicate a potential lack of capacity to understand the nature and wrongfulness of his
actions. This aligns with the criteria set by McNaughton's Rule for the legal defense of insanity.
Incorrect Options:
Option A - Durham Rule: This rule, also known as the "product rule," states that an accused person is
not criminally responsible if their unlawful act was a product of mental illness or mental defect. It focuse
s on the causal relationship between mental illness and the commission of the offense. However, this r
ule has been discarded in many jurisdictions due to its subjective nature and difficulties in determining t
he causal connection.
Option C - Curren's Rule: Curren's Rule states that an accused person will not be criminally responsibl
e for their act if, at the time of committing the act, they did not have the capacity to regulate their condu
ct to the requirement of the law due to mental disease or defect. It emphasizes the inability of the accu
sed to conform their behaviour to legal standards due to their mental condition.
Option D - Section 85 IPC: Section 85 of the Indian Penal Code deals with the criminal responsibility of
an intoxicated person. It states that if a person is so intoxicated at the time of committing the act that t
hey are incapable of judging the nature of the act or that it is wrong or contrary to the law, then they ha
ve not committed an offense. This section applies when the substance was administered against the p
erson's will or without their knowledge.
Incorrect Options:
Option A - Parents or guardians of the child: While parents and guardians play a crucial role in the safe
ty of the child, the responsibility to report cases under the POCSO Act extends beyond them to include
anyone who becomes aware of the offense.
Option B - School teachers or counselors: School teachers and counselors often have a heightened re
sponsibility to report child abuse or offenses against children, but the legal obligation to report under th
e POCSO Act is not limited to them alone. It includes a broader range of individuals.
Option C - Special juvenile police units (SJPU) only: Special juvenile police units are tasked with addre
ssing cases related to children, including those covered by the POCSO Act. However, the reporting obl
igation is not restricted to SJPU alone. It is the responsibility of any person who has knowledge of the o
ffense, regardless of their role or position.
Page 5
286
Correct Option C - When the victim is under 12 years old:
• In the context of the Protection of Children from Sexual Offenses Act (POCSO), Aggravated
(Penetrative) Sexual Assault occurs when the victim is under 12 years old. This age threshold defines a
particularly vulnerable group of children, and offenses against them are considered especially severe
under the law.
Incorrect Options:
Option A - When the victim is under 21 years old: This age range does not specifically define Aggravat
ed (Penetrative) Sexual Assault under the POCSO Act. The law sets a
lower age limit of under 12 years for such cases.
Option B - When the victim is under 13 years old: It does not represent the threshold for Aggravated (P
enetrative) Sexual Assault. Aggravated cases apply to children under 12 years old.
Option D - When the victim is under 18 years old: While the POCSO Act provides protection for all chil
dren under the age of 18, the specific category of Aggravated (Penetrative) Sexual Assault is related to
children under 12 years old.
Page 6
287
systematic method for organizing and documenting patient information in medical records.
Option D - FAST protocol: The FAST protocol is not specifically associated with breaking bad news. It i
s an acronym used to identify the signs and symptoms of a
stroke and to expedite the evaluation and treatment of a suspected stroke. FAST stands for Face, Arm
s, Speech, and Time, which are the key factors assessed to recognize a potential stroke.
Incorrect Options:
Option A - Bereavement is subjective feeling precipitated by the loss of a
loved one, while depression is considered a
pathological mental disorder: Grief reaction is the subjective feeling precipitated by the loss of a
loved one. Bereavement is the state of being deprived of someone due to death.
Option B - Functional impairment is moderate in bereavement and severe in depression: Bereavement
can lead to mild and transient functional impairment, not moderate while depression can lead to clinical
ly significant functional impairment.
Option D - Feeling of worthlessness is more in bereavement than in depression: Feelings of worthlessn
ess are more characteristic of depression, not bereavement. Depression often involves a pervasive se
nse of worthlessness or self-blame, while bereavement may involve grief and sadness but rarely feelin
gs of worthlessness.
Page 7
288
Solution for Question 11:
Correct Option C - Major Depressive Disorder (MDD):
• Major Depressive Disorder is the most common indication for Electroconvulsive Therapy (ECT). It is
particularly recommended for patients with major depressive disorder who have certain characteristics,
such as depression with suicidal risk, severe agitation, psychotic symptoms, a lack of response or
intolerance to medications, or for pregnant women with depression and suicidal tendencies who cannot
take medications.
Incorrect Options:
Option A - Schizophrenia: ECT is more commonly associated with the treatment of mood disorders like
major depressive disorder than schizophrenia.
Option B - Manic episodes: ECT can be considered for the treatment of manic episodes, but it is not th
e primary indication mentioned in the information.
Option D
- Intractable seizure disorder: ECT is not most common treatment for intractable seizure disorders.
Incorrect Options:
Option A - At least 10 seconds: A
10-second seizure may not provide sufficient therapeutic benefit in ECT.
Option B - At least 60 seconds: While a longer seizure duration can be beneficial, a
minimum of 60 seconds is not typically required.
Option C - At least 90 seconds: A minimum of 90 seconds is not the standard duration recommended f
or an effective ECT seizure, 25-second duration is more commonly accepted as effective.
Page 8
289
Incorrect Options:
Option A - Bilateral ECT with bifrontal electrode placement: This configuration is associated with fewer
cognitive side effects and is often used to minimize cognitive impairment in ECT.
Option C - Unilateral ECT with electrodes placed on the vertex and right side: Unilateral ECT, especiall
y with right-side electrode placement (d'ELIA placement), is known for its better cognitive side effect pr
ofile. It is often used when cognitive side effects need to be minimized.
Option D - Bilateral ECT with electrodes placed on both sides of the scalp: While Bilateral ECT, in gene
ral, may have cognitive side effects, the bifrontotemporal electrode placement specifically is associated
with more pronounced cognitive side effects.
Page 9
290
Psychology
1. What is the term used to describe the therapist's feelings towards the patient, which can be both
conscious or unconscious, and can be both negative and positive?
A. Transference
B. Countertransference
C. Empathy
D. Projection
----------------------------------------
2. Which therapeutic technique involves the patient speaking freely and uninterrupted, expressing
thoughts without censorship, with the therapist sitting nearby but not visible to the patient?
A. Transference
B. Free association
C. Parapraxis
D. Condensation
----------------------------------------
3. Which of the following facts about the Structural Theory of Mind is incorrect?
(or)
Which of the following facts about the Structural Theory of Mind is incorrect?
A. The Id operates on the pleasure principle and lacks impulse control.
B. The Ego is the executive organ of the mind and balances the demands of the Id and Superego.
C. The Superego develops between 5-6 years of age and is mostly present in the conscious part of the
mind.
D. The Structural Theory of Mind consists of the Id, Ego, and Superego, representing different aspects
of the mind.
----------------------------------------
4. Which of the following statements about defense mechanisms is incorrect?
A. Defense mechanisms are used by the ego to manage conflicts between the demands of the id and
superego.
B. Defense mechanisms help in preventing excessive anxiety.
C. Denial is a defense mechanism where one refuses to acknowledge or accept reality.
D. Defense mechanisms are conscious strategies used to cope with psychological conflicts.
----------------------------------------
5. A 35-year-old woman, Ms S, constantly accuses her husband of infidelity. Despite having no
evidence or suspicions, she strongly believes that he is cheating on her. On the other hand she is
having an extra marital affair. Which defense mechanism is primarily at play in this scenario?
A. Regression
B. Repression
291
C. Projection
D. Rationalization
----------------------------------------
6. John, a 35-year-old man, has a history of emotional neglect during his childhood, which has left him
with a deep sense of insecurity and feelings of unworthiness. In his relationships, John tends to neglect
his partners' needs and consistently ignores them. Despite thinking he is undeserving and unworthy of
love, he often idealizes his partner. Considering John's tendency to neglect his partners and his
deep-seated feelings of unworthiness, which defense mechanism is most likely at play in this case
scenario?
(or)
Which defense mechanism is demonstrated by John, a 35-year-old man with a history of childhood
emotional neglect, who responds to his feelings of insecurity and unworthiness by neglecting his
partners' needs and continuously ignoring them?
A. Blocking
B. Acting out
C. Regression
D. Introjection
----------------------------------------
7. John had a stressful day at work. He received negative feedback from his boss, which made him
angry and frustrated. When he returned home, he noticed that his favorite mug was missing. In a fit of
rage, he kicked the nearby table, causing it to topple over and break. Later, John suddenly felt a sense
of relief about his bad day at work with his boss. Which defense mechanism is shown by John in this
case scenario?
(or)
Which defense mechanism is exemplified when John redirects his anger from his bad day at work onto
a nearby table, resulting in the table breaking and him later feeling relieved about the work situation?
A. Regression
B. Intellectualization
C. Displacement
D. Reaction formation
----------------------------------------
8. John is a highly ambitious and competitive individual. He has always had a strong desire to succeed
in his career. However, he also experiences intense feelings of anger and aggression, which he finds
unacceptable. To channel these impulses in a socially acceptable way, John joins a boxing club and
starts participating in competitive matches. He finds that engaging in physical exercise and using his
aggression in a controlled environment helps him manage his emotions effectively. Which defense
mechanism is exemplified in the scenario described above?
(or)
What defense mechanism is illustrated when John, driven by his ambition and competitive nature,
channels his anger and aggression into boxing matches to manage his emotions in a controlled and
socially acceptable manner?
A. Altruism
Page 2
292
B. Anticipation
C. Asceticism
D. Sublimation
----------------------------------------
9. Sarah is a 35-year-old woman who spends several hours each day engaging in repetitive behaviors
such as handwashing and checking. She feels intense anxiety and fear that something terrible will
happen if she doesn't perform these rituals. Despite recognizing that her fears are irrational, she feels
compelled to continue these behaviors to alleviate her anxiety. However, she also engages in acts of
kindness and generosity towards others. For example, after performing her rituals, she donates money
to charities and volunteers at local shelters. Which defense mechanism is being used by Sarah in this
scenario?
(or)
What defense mechanism is demonstrated by Sarah, who engages in repetitive behaviors to alleviate
anxiety, yet counteracts her distress by performing acts of kindness and generosity towards others?
A. Inhibition
B. Isolation of affect
C. Displacement
D. Undoing
----------------------------------------
10. Which of the following is incorrectly matched?
A. Oral stage -Fixation: schizophrenia, substance dependence.
B. Anal stage - Major site of pleasure: Genital area
C. Phallic stage - Age range: 3-5 years
D. Latent stage -relative sexual quiescence (inactivity).
----------------------------------------
11. What does the Oedipus complex refer to?
A. A psychological disorder related to excessive attachment to one's mother.
B. An attraction of male child towards one's mother during phallic stage of psychosexual development
C. A fear of castration experienced by male children.
D. A stage of cognitive development characterized by conflict with parental figures.
----------------------------------------
12. Which option regarding the stages of cognitive development is incorrect based on the information
provided?
A. Sensorimotor stage: Children learn through sensory observations and gains motor control, and
develop object permanence.
B. Stage of Preoperational thought: Children use symbols and languages more extensively, exhibit
intuitive, egocentric, and animistic thinking.
C. Stage of Concrete operations: Children can understand perspectives, do not use logical thinking,
and demonstrate conservation and reversibility.
Page 3
293
D. Stage of Formal operations: Abstract thinking develops, and children can make hypotheses and test
them against reality.
----------------------------------------
13. What is the term used to describe the phenomenon where a neutral stimulus starts to produce a
response after being repeatedly paired with a stimulus that naturally elicits that response?
A. Operant conditioning
B. Extinction
C. Classical conditioning
D. Stimulus generalization
----------------------------------------
14. According to Permack's principle, which statement is true?
A. A less favorable behavior can reinforce a more favorable behavior.
B. A more favorable behavior can reinforce a less favorable behavior.
C. A behavior with no favorability can reinforce any other behavior.
D. The frequency of a behavior has no impact on reinforcement.
----------------------------------------
15. What is the goal of biofeedback?
A. To provide information about involuntary biological functions.
B. To measure muscle tension in the treatment of bruxism.
C. To bring involuntary functions of the autonomic nervous system under voluntary control.
D. To regulate symptoms related to blood pressure and heart rate.
----------------------------------------
16. Which of the following statements about the image shown below is incorrect?
Page 4
294
17. The Thematic Apperception Test (TAT) is a psychological assessment tool that involves:
A. Answering a set of true or false questions about oneself.
B. Drawing a picture representing one's emotions.
C. Interpreting and creating stories about pictures depicting individuals engaged in various activities.
D. Identifying hidden objects within abstract inkblot images.
----------------------------------------
Correct Answers
Question Correct Answer
Question 1 2
Question 2 2
Question 3 3
Question 4 4
Question 5 3
Question 6 4
Question 7 3
Question 8 4
Question 9 4
Question 10 2
Question 11 2
Question 12 3
Question 13 3
Question 14 2
Question 15 3
Question 16 2
Question 17 3
Page 5
295
Option A- Transference: Transference refers to the patient's emotions, thoughts, and feelings that are t
ransferred onto the therapist. It is a psychoanalytic concept where the patient unconsciously projects th
eir feelings and desires onto the therapist, often based on past relationships or experiences. Transfere
nce can provide valuable insights into the patient's internal world and can be explored in therapy.
Option C- Empathy: Empathy refers to the therapist's ability to understand and share the feelings and e
xperiences of the patient. It involves putting oneself in the patient's shoes and experiencing their emoti
ons from their perspective. Empathy is an essential component of effective therapeutic relationships an
d can help create a safe and supportive environment for the patient.
Option D- Projection: Projection is a defense mechanism where individuals attribute their own undesira
ble thoughts, feelings, or impulses onto someone else. It involves unconsciously displacing one's own
emotions or qualities onto another person. Projection is not specifically related to the therapist-patient r
elationship but is a broader psychological concept applicable to various interpersonal dynamics.
Page 6
296
Option B- The Ego is the executive organ of the mind and balances the demands of the Id and Supere
go: This statement accurately describes the role of the Ego as the executive organ of the mind that bal
ances the demands of the Id and Superego.
Option D- The Structural Theory of Mind consists of the Id, Ego, and Superego, representing different a
spects of the mind: This statement provides a general overview of the Structural Theory of Mind, which
involves the Id, Ego, and Superego as different components of the mind.
Page 7
297
he person blocks out memories of past flirtatious behavior and genuinely believes they have never acte
d inappropriately.
Option D - Rationalization: Rationalization involves providing logical-sounding explanations to justify on
e's behavior, even if they are not entirely accurate. While it can sometimes be related to accusations of
infidelity, it does not align as directly with the scenario as projection does. Eg- If the person justifies sp
ending excessive time with a
colleague by claiming it's work-related and necessary for career advancement.
Page 8
298
Option D- Reaction formation: Reaction formation is a defense mechanism where an individual suppre
sses or denies their true feelings or impulses by expressing or behaving in the opposite manner. It invo
lves adopting beliefs, attitudes, or behaviors that are contrary to one's genuine emotions or desires. Fo
r example, To conceal her true feelings, Sarah goes out of her way to be excessively friendly and supp
ortive towards Mark. She compliments him on his achievements, expresses admiration for his leadersh
ip, and actively seeks his company. In the workplace, she presents herself as Mark's biggest ally and s
upporter, despite her underlying resentment.
Page 9
299
actions to offset guilt or anxiety associated with undesirable thoughts or impulses.
In the scenario you provided:
• Sarah experiences intense anxiety and fear related to obsessive thoughts (e.g., something terrible will
happen).
• She engages in compulsive behaviors (e.g., handwashing, checking) to alleviate the anxiety.
• Following the rituals, she performs acts of kindness and generosity as a way of undoing the negative
thoughts and behaviors associated with her anxiety.
Incorrect options
Option A: Inhibition: Inhibition refers to holding back or suppressing certain thoughts, feelings, or behav
iors, which is not directly reflected in Sarah's actions.
Option B: Isolation of affect: Isolation of affect involves separating feelings from thoughts or ideas. This
doesn't fully apply to Sarah's situation as her actions are more focused on reducing anxiety and distre
ss.
Option C: Displacement: Displacement is the redirection of emotions from their original source to a
substitute target. While Sarah's acts of kindness may seem like a
redirection, they are more related to undoing her distress rather than displacing emotions.
Page 10
300
Solution for Question 11:
Correct Option B: An attraction of male child towards one's mother during phallic stage of psychosexua
l development:
• This option correctly defines the Oedipus complex as an attraction towards one's mother during the
phallic stage of psychosexual development. According to Freud, male children experience unconscious
feelings of desire for their mother and view their father as a rival for her affection.
Incorrect Options:
Option A- A psychological disorder related to excessive attachment to one's mother: This option descri
bes the Oedipus complex as a psychological disorder related to excessive attachment to one's mother.
However, it is important to note that the Oedipus complex is a psychoanalytic theory, not a
psychological disorder itself.
Option C- A fear of castration experienced by male children: This option refers to a fear of castration ex
perienced by male children. While the Oedipus complex involves castration anxiety, it is not limited to f
ear alone but encompasses the broader psychosexual dynamics between a
child, their mother, and their father.
Option D- A
stage of cognitive development characterized by conflict with parental figures: This option describes a
stage of cognitive development characterized by conflict with parental figures. However, the Oedipus c
omplex is not related to cognitive development but rather focuses on the psychosexual development of
a child.
Option A: Sensorimotor stage: Children learn through sensory observations and gains motor control, a
nd develop object permanence: Children in the sensorimotor stage learn through sensory observations
and gain motor control. They also develop the concept of "object permanence," which means they und
erstand that objects continue to exist even when they are out of sight.
Option B: Stage of Preoperational thought: Children use symbols and languages more extensively, exh
ibit intuitive, egocentric, and animistic thinking: Children in the preoperational stage use symbols and la
nguage more extensively, but they are known for their limitations in logical thinking. They often exhibit i
ntuitive, egocentric (self-centered), and animistic (attributing human qualities to inanimate objects) thin
king. Option D: Stage of Formal operations: Abstract thinking develops, and children can make hypoth
eses and test them against reality: In this stage, abstract thinking develops, and individuals can think h
ypothetically and test their hypotheses against reality. They can engage in more advanced problem-sol
ving and critical thinking.
Page 11
301
Solution for Question 13:
Correct Option C: Classical conditioning
• The phenomenon where a neutral stimulus starts to produce a response after being repeatedly paired
with a stimulus that naturally elicits that response is termed as Classical conditioning.
Incorrect Options:
Option A- Operant conditioning: Operant conditioning refers to learning based on consequences and b
ehavior modification through reinforcement or punishment. It is not the correct answer in this case.
Option B- Extinction: Extinction refers to the gradual disappearance of a conditioned response when th
e conditioned stimulus is presented repeatedly without the unconditioned stimulus. It is related to classi
cal conditioning but does not address the specific phenomenon mentioned in the question stem.
Option D- Stimulus generalization: Stimulus generalization refers to the tendency for a conditioned res
ponse to occur in the presence of stimuli that are similar to the conditioned stimulus. While it is a pheno
menon associated with classical conditioning, it does not directly answer the question about the neutral
stimulus acquiring a response through repeated pairings.
Page 12
302
Option B- To measure muscle tension in the treatment of bruxism: Measuring muscle tension, such as
through the use of electromyography (EMG), is one of the techniques used in biofeedback. However, it
is not the overarching goal of biofeedback.
Option D- To regulate symptoms related to blood pressure and heart rate: Regulating symptoms relate
d to blood pressure and heart rate is one of the potential applications of biofeedback, but it is not the pr
imary goal of biofeedback itself.
Page 13
303
Recent Updates
1. A 65-year-old man presents to a memory clinic with a one-year history of progressive memory
impairment and difficulties with daily tasks. He has been diagnosed with Alzheimer's disease based on
clinical evaluation and imaging findings showing hippocampal atrophy. The patient and his family are
eager to explore potential treatment options to slow the progression of his cognitive decline.
Considering recent developments in the field of Alzheimer's disease management, what is the most
appropriate course of action for this patient, particularly in exploring a novel treatment option?
A. Initiate treatment with cholinesterase inhibitors (e.g., donepezil)
B. Begin cognitive behavioral therapy (CBT) to address memory deficits.
C. Refer the patient to a support group for individuals with Alzheimer's disease.
D. Discuss the potential use of lecanemab, a novel investigational treatment for Alzheimer's disease.
E. Recommend dietary and lifestyle modifications to improve cognitive function.
----------------------------------------
2. A 28-year-old man with a history of schizophrenia presents to a mental health clinic. He has been on
various antipsychotic medications in the past but has experienced inadequate symptom control and
bothersome side effects, including weight gain and sedation. Despite these challenges, the patient is
motivated to find an effective and well-tolerated treatment for his schizophrenia. Considering recent
developments in the field of schizophrenia management, what is the most appropriate course of action
for this patient, particularly in exploring a potential alternative antipsychotic medication with a more
favorable side effect profile?
A. Continue the current antipsychotic medication regimen and consider psychotherapy
B. Increase the dosage of the current antipsychotic medication
C. Switch to brexpiprazole, a newer antipsychotic medication with a more favorable metabolic side
effect profile
D. Refer the patient for electroconvulsive therapy (ECT) consultation.
----------------------------------------
Correct Answers
Question Correct Answer
Question 1 4
Question 2 3
304
Incorrect Options:
Option A - Initiate treatment with cholinesterase inhibitors (e.g., donepezil): Cholinesterase inhibitors ar
e commonly used in Alzheimer's disease to improve cognitive function. However, they primarily provide
symptomatic relief and do not modify the course of the disease.
Option B - Begin cognitive behavioral therapy (CBT) to address memory deficits: CBT can be beneficial
for managing behavioral and psychological symptoms in Alzheimer's disease, but it is not considered
a primary treatment for slowing disease progression.
Option C - Refer the patient to a support group for individuals with Alzheimer's disease: Support groups
can provide emotional and social support for patients and families dealing with Alzheimer's disease. W
hile valuable, this option does not address the potential for disease-modifying treatment.
Option E - Recommend dietary and lifestyle modifications to improve cognitive function: Dietary and life
style modifications, such as a healthy diet and regular exercise, can be important components of Alzhei
mer's disease management. However, they are generally considered as part of a
comprehensive care plan and may not have a direct disease-modifying effect.
Incorrect Options:
Option A - Continue the current antipsychotic medication regimen and consider psychotherapy: This op
tion does not address the patient's concerns about inadequate symptom control and bothersome side
effects. Exploring alternative treatments with better tolerability is warranted.
Option B - Increase the dosage of the current antipsychotic medication: Increasing the dosage of the c
urrent medication may not necessarily lead to better symptom control and could potentially worsen side
effects. Considering a different medication with a more favorable profile is a reasonable approach.
Option D - Refer the patient for electroconvulsive therapy (ECT) consultation: ECT is typically consider
ed when other treatments have failed or in severe cases of schizophrenia. It is not typically a first-line o
ption for patients with treatment-resistant schizophrenia without exploring alternative antipsychotic med
ications.
Option E - Explore complementary and alternative therapies for symptom management: Complementar
y and alternative therapies may have a role in adjunctive symptom management but are not considere
d primary treatments for schizophrenia. Addressing the core symptoms of schizophrenia often requires
antipsychotic medications.
Page 2
305